X



スレッドを立てるまでもない質問スレッド Part 377
レス数が1000を超えています。これ以上書き込みはできません。
0001😉三年英太郎🌈 ◆3CZBjOt3.Y (ワッチョイW 9b86-Y4hg)
垢版 |
2022/12/30(金) 12:34:35.87ID:l/IGXK/s0
■ 質問する方へ
・常識的な内容なら何を質問してもOKです。
・できるだけ分りやすく、具体的に質問して下さい。ソースがあれば必ず書いて下さい。

■ 回答する方へ
・YAHOO!知恵袋のコピペを使った嫌がらせが後をたたないので、回答の前に、
 知恵袋に同じ質問が出ていないか、質問文の中のワードでググってみたほうが賢明です。
・回答する方は、質問者の立場になって答えてあげましょう。

■ 辞書
・単語や熟語の意味なら辞書で調べた方が速くて正確です。
 研究社 - ルミナス英和・和英辞典
 http://www.kenkyusha.co.jp/modules/08_luminous/index.php?content_id=1
 無料で引けるオンライン辞書(英英辞典) - おさんぽ英語 おすすめ散歩道
 https://osanpo-english.com/site-book/online-dictionary.html

■ ChMateなどのアプリを使っていて、音声入力で英語を書き込みたいときは:
・(Androidの場合) 書き込む→工具マーク→入力方式の切り替え→音声入力に切り替え 。
・(iOSの場合) キーボードを表示させて左下のマイクのアイコンをタップする 。

過去スレ

スレッドを立てるまでもない質問スレッド Part 376
https://lavender.5ch.net/test/read.cgi/english/1667925886/
0002😉三年英太郎🌈 ◆3CZBjOt3.Y (ワッチョイW 9b86-Y4hg)
垢版 |
2022/12/30(金) 12:35:52.48ID:l/IGXK/s0
>>1
テンプレではありませんが過去スレで問題提起がありましたので転載します
(このスレ内でレスしている様に見えない様、アンカーのみ削除させていただきました)

> 881 名無しさん@英語勉強中 (オッペケ Sr03-crGe)[sage] 2020/10/12(月) 18:48:11.75 ID:BiNVjerEr
> 例文貼りまくり厨はマジ自重すべき
> 今このスレ、モバイルで開けらんない時あるぞ
>
> 924 名無しさん@英語勉強中 (ワッチョイ 9fa1-AMwI)[] 2020/10/15(木) 07:54:22.61 ID:oZ2vtNXk0
> ・同じ用法に関する例文は2つまで
> ・文法的な解説以外の、英語の新聞や雑誌の記事などを「同じ単語や表現が出てくるから参考になる」という理由で貼るのは避ける
0003名無しさん@英語勉強中 (ワッチョイ cb08-QOCT)
垢版 |
2022/12/30(金) 13:12:38.13ID:cERRz8Pb0
米国人のSNSでのコメントなんですが

文法変ですよね?

He was my dream come true
0004名無しさん@英語勉強中 (ワッチョイ b384-76NA)
垢版 |
2022/12/30(金) 13:43:42.44ID:+HLLaW6x0
>>2
0007名無しさん@英語勉強中 (ササクッテロル Sp35-hpJb)
垢版 |
2022/12/30(金) 18:10:43.75ID:HFqG6Khhp
家族の銀行取引って照会でわかりますか?
本人じゃないと無理?
0011名無しさん@英語勉強中 (ササクッテロル Sp35-hpJb)
垢版 |
2022/12/30(金) 19:33:47.57ID:HFqG6Khhp
>>9
どうも板を間違えたらしいw
0012名無しさん@英語勉強中 (ワッチョイ 5f89-jXNG)
垢版 |
2022/12/30(金) 19:50:11.77ID:w9xuyFhJ0
>>7
>>家族の銀行取引って照会でわかりますか?本人じゃないと無理?
=====

もしこれを英訳するとしたら、次のようにでも書くかな。

I want to know what transactions one of my family members
has been doing with their banks. Would the banks let me know
the contents of the transactions? Or would they refuse?
0013名無しさん@英語勉強中 (ワッチョイ 5f89-jXNG)
垢版 |
2022/12/30(金) 20:08:16.71ID:w9xuyFhJ0
前スレで "as I write" が本当に「私がこの文章を書いている現在は」という
意味なのかどうかという議論があったけど、今たまたまよい例文が見つかった。

It is full of its golden flowers this July, ★as I write (1981)★.

出典:
Franklyn Grace Lyo, ?M.A. Lyons ・ 2008 ・ ?Biography & Autobiography
https://books.google.co.jp/books?id=4T6LAAAAQBAJ&pg=PA491&lpg=PA491&dq=%22as+i+write++1981%22&source=bl&ots=1dS_Dhm_mp&sig=ACfU3U30J4F4uvVz3t65IqKY_BASLx8WTQ&hl=en&sa=X&ved=2ahUKEwjirMvZmKH8AhVbpVYBHRY_BTgQ6AF6BAgJEAM#v=onepage&q=%22as%20i%20write%20%201981%22&f=false

上のリンクにて表示される p.491 というページの、上から 12 行目にこの一節が見える。
"as I write (1981)" と書いているのだから、当然のことながら
「私がこの文章を書いている現在(つまり 1981年)」
という意味に決まってるわな。
0014名無しさん@英語勉強中 (ワッチョイ 5f89-jXNG)
垢版 |
2022/12/30(金) 20:12:33.59ID:w9xuyFhJ0
ついでに、
"as I write 1984"
というフレーズも検索してみたら、
https://www.google.com/search?q=%22as+I+write+1984%22&newwindow=1&rlz=1C1FQRR_enJP949JP949&sxsrf=ALiCzsa8jhz2CqNTJxdhQpN0fodA6XgTAw%3A1672398603709&ei=C8euY-TzKsG5oAST876ADQ&ved=0ahUKEwikpcmlmqH8AhXBHIgKHZO5D9AQ4dUDCA8&uact=5&oq=%22as+I+write+1984%22&gs_lcp=Cgxnd3Mtd2l6LXNlcnAQAzIICCEQwwQQoAEyCAghEMMEEKABMggIIRDDBBCgAUoECEEYAUoECEYYAFD0BVjWCGDsCWgBcAB4AIABuwGIAaUCkgEDMC4ymAEAoAEBwAEB&sclient=gws-wiz-serp
上のリンク先でいくつも
"as I write (1984)"
という用例が見つかった。
0016The OED LOves Me Not (ワッチョイ 5f89-jXNG)
垢版 |
2022/12/30(金) 20:28:25.14ID:w9xuyFhJ0
"as I write" の他に、"at the time I now write" という言い回しも、同じような意味であるらしい。

John Denvir ・ 2019 ・ ?History
I see by the Catholic Directory there are ★at the time I now write★ thirty-nine churches and chapels in Liverpool. The schools have increased in a like ...

しかし上の言い回しに of をつけるだけで、意味が変わってしまうらしい。

The Adventures of Owen Evans - Page 221 - Google Books Resulthttps://books.google.co.jp ? books
Rev. W.H. Anderdon
... that when I came back to England (a thing I never thought to do ★at the time I now write of★), I was for ever coming out with Indian words unawares : so ...

つまり、"at the time I now write of" となると、「いま私が書き綴っている対象であるその当時には」
という意味であるらしいのだ。そうなると日本語では、そんなまどろっこしいことは言わず「その当時には」
とでも書くだろう。
0019名無しさん@英語勉強中 (JP 0He3-9yt5)
垢版 |
2022/12/31(土) 03:13:31.77ID:IG9DbE6KH
前スレの994はもしかしたらニュージーランドや南アフリカやアイルランドやらアメリカの一部のɪとəの違いが小さい話者の発音やったんかもしれん
0021名無しさん@英語勉強中 (ワッチョイ 4e84-0ejh)
垢版 |
2022/12/31(土) 11:54:40.27ID:jW+C6I3f0
光浦靖子がカナダ留学して1年半経過してるそうなんだけど、
「ホントに。もうびっくりするぐらい。英語だけがいまだに聞き取れない。
もうトシなのかな。難しい…」なんだってさ。
50過ぎるとそんなに難しいもんなの?
カナダって英語の発音がすごくきれいなんだよね?
それで聞き取れないなんて英語学習って厳しいな。
誰か感想をお願いします。できたら50過ぎて成功した方の話なんかあったら。
0027名無しさん@英語勉強中 (ワッチョイ 4e84-0ejh)
垢版 |
2022/12/31(土) 13:38:48.05ID:jW+C6I3f0
>>26
インドネシア語専攻だとしても英語がかなり高いレベルでないと外大は入れないでしょ?
入学時で準一レベルだとか。
0030名無しさん@英語勉強中 (ワッチョイ a35d-EAgS)
垢版 |
2022/12/31(土) 16:25:50.05ID:G9Cq7G+f0
和文英訳の修業に
491今日の大学生は自分の受ける教育を収入のよりよい仕事にあるつくための手段としか考えない.
Today's college student views his education simply as a means towards a better paying job.
という暗誦用例文があります.

主語に加算名詞を無冠詞単数形で使う例って,manとか以外にはあまりみないのですが,どういう用法なのでしょうか.
ご存じの方,教えていただけないでしょうか.
よろしくお願いします.
0031名無しさん@英語勉強中 (ワッチョイ 4e84-0ejh)
垢版 |
2022/12/31(土) 16:47:22.78ID:jW+C6I3f0
>>29
ちと古いが偏差値の資料あったよ。
確かにそれほど堪能ではないかも。
https://harubou-room.com/1984hensachi/
0032名無しさん@英語勉強中 (ワッチョイ 1af0-TwI4)
垢版 |
2022/12/31(土) 16:54:29.56ID:KHbgVDzQ0
>>30
Today's てあるじゃん
0033名無しさん@英語勉強中 (ワッチョイ 5b89-joYd)
垢版 |
2022/12/31(土) 16:55:14.99ID:kGPGP5JT0
>>30
ふつう複数形だな
003630 (ワッチョイ a35d-EAgS)
垢版 |
2022/12/31(土) 17:55:40.43ID:G9Cq7G+f0
>>32-34
レスありがとうございます.

>>32
ということは,Today'sを限定詞として扱う場合は,「today's college student」は,
the studentの場合と同じように「現代の大学生というもの」といった総称表現と
いったことになるのでしょうか?
0037The OED LOves Me Not (ワッチョイ 4e89-4uR0)
垢版 |
2022/12/31(土) 18:39:15.88ID:uz1Vr+8T0
>>24
それについては、僕は前スレで次のようにすでに書いている。

==========

948The OED Loves Me Not (中止 5f89-jXNG)2022/12/25(日) 22:14:45.51ID:HPt74Rs20XMAS?>>950
>>946
その通りだと僕は思っている。ただし僕は、
(1) as I write
(2) as I am writing this
などという言い回しが、本当に僕の言っているような意味合いなんだと
いうことを誰かに教わったわけではない。誰かに質問したこともない。
どこかに書いてあったわけでもないし、辞書をいま引いてみても、載っていないように
思える。どこか気の利いた辞書や参考書には載っているかもしれないけどね。
0038The OED LOves Me Not (ワッチョイ 4e89-4uR0)
垢版 |
2022/12/31(土) 18:47:31.28ID:uz1Vr+8T0
>>34
そのリンク先の記事には、少しだけ間違いがある。

>>次のセンテンスのように,それぞれの名詞を複数形にしたら無冠詞でも間違いにならない。

(1) We synthesized compounds and measured molecular weights.

厳密に言うと、これは間違いだ。
(2) We synthesized compounds and measured ★their★ molecular weights.
にしないといけない。もしも their がなくて (1) のままだったら、
「我々は(いくつかの)化合物を合成し、そのあと(どういう物質についてのものかはわからいけど)
ともかく分子量というものを測定した」
なんていう変な意味になる。

しかし普通、We synthesized compounds と言えば、そのあとの molecular weights は
「その化合物の分子量」であるはずだ。そうなると、当然のことながら their がつく。
もしもどうしても、すでに言った compounds ではなくて別の物質の molecular weights を
測定したのであれば
We synthesized compounds and measured the molecular weights of substances other than the above.
とでもなろうけど、そんな変な状況がありえるだろうか?

別の所で著者は次のように書いているが、これは正しい。

>>compound の前に冠詞の a を付けて, molecular weight の前に所有代名詞の its を付けたら文が正しくなる。
>>We synthesized a compound and measured its molecular weight.

何で上の一節ではちゃんと正しいことを書いているのに、そのずっと上の (1) では間違ったことを書いてしまっているのだろうか?
0039The OED LOves Me Not (ワッチョイ 4e89-4uR0)
垢版 |
2022/12/31(土) 18:54:36.07ID:uz1Vr+8T0
>>20
> I asked him to use the toilet

ask someone to do something は、「誰かに何かをしてくださいと頼む」
という意味だから、him が use the toilet することになる。しかし

I asked to use the toilet. は「トイレを使わせてください、と私は頼んだ」
という意味。

参考: この下の英文を端から端まで読めば、前後の関係からも
asked to use the toilet は「使わせて下さいと頼んだ」という意味だとはっきりわかる。

“In one instance, when ★I asked to use the toilet★, although they allowed me to do so,
I was accompanied by a guard and the guard did not untie me and did not remove by blindfold,” she said.
https://newsinfo.inquirer.net/186771/%E2%80%98morong-43%E2%80%99-charge-arroyo-with-torture
0041The OED LOves Me Not (ワッチョイ 4e89-4uR0)
垢版 |
2022/12/31(土) 22:11:42.04ID:uz1Vr+8T0
>>36
>>「today's college student」は,the studentの場合と同じように「現代の大学生というもの」といった総称表現
===

あなたの言う通りだと思うよ。チラッと調べてみたら、

(1) the college student of today
(2) the college students of today

これら2つともちゃんと使われていることが、検索すればわかる。両方とも総称表現だということになるわな。そしてこの2つは
(1-a) today's college student
(1-b) today's college students
と、それぞれ同じような意味になるんだろうと思う。そしてこの (1-a), (1-b) についても、
両方ともちゃんと使われていることも、やはり検索によって確かめることができる。

さらには、
(3) college students today
という表現もちゃんと使われていることが検索によってわかるが、これもやはり「今日の大学生たち」という意味の
総称表現として使われている。
0042名無しさん@英語勉強中 (オッペケ Srbb-aAef)
垢版 |
2023/01/01(日) 19:29:52.33ID:PIbsT6xwr
質問です。
This is a pen.ってどう訳したらいいのですか?
直訳すると「これはペンです」なのですが、
アメリカ人とか感動詞的にThis is a pen.って使いますが、
直訳するととてもシュールな事になりますよね。
「おお・・・」とか「なんと!」とかいう感じでしょうか?
0043名無しさん@英語勉強中 (ワッチョイ 8a69-9yt5)
垢版 |
2023/01/01(日) 23:54:17.95ID:fQxHFQIg0
The festival has gained international recognition for the talent it has attracted over the years.

この文の
recognition for thetalent it has attracted
の部分のtalent it has のところが特に意味不明なんですが、関係代名詞とかが省略されているのでしょうか?
0047名無しさん@英語勉強中 (ワッチョイ 4e84-0ejh)
垢版 |
2023/01/02(月) 08:04:57.78ID:MtQGM8z80
>>45
>>46
だとすると it が余計な気がする。
勢い余って関係代名詞に it をつかったんでは?
0048名無しさん@英語勉強中 (ワッチョイ 4e84-0ejh)
垢版 |
2023/01/02(月) 08:24:14.66ID:MtQGM8z80
>>47
あ〜、it はfestivalか。
間違えた。
俺ドンマイ!
0049名無しさん@英語勉強中 (ワッチョイW 27e3-VpgV)
垢版 |
2023/01/02(月) 18:37:42.28ID:jnJiMhuk0
TOEICでよく見かける文です

what does the man suggest the women do?

この文の言いたいことはわかりますがモヤモヤします
suggestのあとにまだSVが来ていて???となってしまい
意味がパッとわかりません
違和感を感じるというか

この文を文法的に説明していただければと思います

よろしくお願いいたします
0051名無しさん@英語勉強中 (ワイーワ2W FF12-buLv)
垢版 |
2023/01/02(月) 20:25:51.30ID:uOQqjBxAF
The Asawa 4 is ( ) any vehicle currently on the market.
の問題で正解のunlike ではなく、〜を上回って、を意味するaboveを選んでしまいました。
なぜaboveがダメで、
どういう形ならaboveも正解になり得たのでしょうか?
わかりやすい解説をお願いします。
Asawa 4は現在市販されている、いかなる乗り物とも似ていません。
0052名無しさん@英語勉強中 (ワッチョイW 76e3-vLkk)
垢版 |
2023/01/02(月) 21:53:25.49ID:dsEOk6X90
今年はアルクのTOEIC初心者コースから始めて、ゆくゆくはヒアリングマラソン頑張ろーと思ってたら
アルクからそっくり講座が無くなってるのを今日知りました。
代替で良い通信講座ありませんか?
0054名無しさん@英語勉強中 (ワッチョイ 9afb-9yt5)
垢版 |
2023/01/03(火) 00:03:02.48ID:kc7cS7MS0
>>49 アメリカ英語の特徴の一つの仮定法現在というもの
とはいえ単にtheの前のthatとwomenの後のshouldが省略されてるだけ
What does the man suggest (that)the woman (should) do?とするとわかりやすいか
0055名無しさん@英語勉強中 (ワッチョイ 9afb-9yt5)
垢版 |
2023/01/03(火) 00:06:21.27ID:kc7cS7MS0
thatまで省略するのは相当くだけた文か
0056The OED LOves Me Not (ワッチョイ 4e89-4uR0)
垢版 |
2023/01/03(火) 02:50:39.54ID:JwxJ4Pb20
>>51
(1) The Asawa 4 is ( ) any vehicle currently on the market.
(2) Asawa 4は現在市販されている、いかなる乗り物とも似ていません。

(1) が英文で、(2) がその和訳として提示されてるんだろ?それなら正しい答えは
unlike であって、above ではないのは明らかだろ?

「似ていない」ということは、Awasa 4 は他のものよりも「劣っている」から
似てないのかもしれない。しかし above は「優れて」いないといけない。
0057The OED Loves Me Not (ワッチョイ 4e89-4uR0)
垢版 |
2023/01/03(火) 03:04:59.30ID:JwxJ4Pb20
>>53
> 貴方は最近、誰のYoutubeチャンネルのコミュニティにコメントをしてましたっけ?

(1) Whose YouTube community have you lately been commenting in?
(2) I guess you've lately been commenting in someone's YouTube community. Whose was it, I wonder?
(3) Have you by any chance been commenting in someone's YouTube community? If so, whose was it?

commenting の代わりに making posts でもいいと思う。
0058名無しさん@英語勉強中 (ワッチョイ 4e4b-uhAF)
垢版 |
2023/01/03(火) 07:25:59.01ID:0nMAHFVK0
For crying out loud, why didn't you say something?
お願いだから何とか言ってくれ。

最後はanythingでなくて良いのでしょうか?
0059The OED Loves Me Not (ワッチョイ 4e89-4uR0)
垢版 |
2023/01/03(火) 07:37:50.17ID:JwxJ4Pb20
>>58
(1) For crying out loud, why didn't you say ★something★?
お願いだから ●何とか● 言ってくれ。

(2) For cryinbg out loud, why didn't you say ◆anything◆?
★ほんとにもう、なんだって何にも言わなかったんだよ?★

===

上のように、not something と not anything とでは、かなり意味が違ってきて、
和訳をするとそれ以外の部分まで訳し方を変え、一文全体の
和訳までが変わってしまった。

いちいち解説はしない。なぜかというと "not... anything" と "not... something" に
ついてはあっちこっちに解説があるからだ。辞書にも載ってるし。
ただ、辞書や文法書での解説を読んだからって、少なくとも僕の場合は
すぐにきちんとは理解できなかったし、したがってきちんと身にはつかなかったし、
当然のことながらそれを使った英語なんてなかなか書けなかった。

理解するのに5年や10年はかかったし、身に付くのには20年かかり、英語を書く時にも
自然とそれが出てくるには、どれくらいかかったかな?何十年も経っても、いまだに
not... something とか not... somebody などは(not... anything の場合とはちがって)
いまだに書けないし(しゃべることなら余計のことしゃべれない)かもしれない。
それくらいに日本人にとっては some が not とくっついたときや、
肯定文や if 構文の中に any がついたときなどは、
ネイティブ並みに使いこなすのは難しいと思っている。
0061名無しさん@英語勉強中 (ワッチョイ 4e84-0ejh)
垢版 |
2023/01/03(火) 08:35:42.31ID:UuVQA+KD0
>>49
平叙文に直すと
The man suggest (that) the women do (something).
somethingを問いたいのでwhatに置き換える。
The man suggest the women do what.
これをwhatを問う疑問文にするにはthat節の中で疑問文を作るのではなく
× The man suggests what do the women do? (疑問文にならない、意味が分からない)
また以下は提案するかどうかの疑問文になるのでダメ。
× Does the man suggest the women do what?
具体的な答えを問う疑問詞疑問文を作るのでwhatを文頭に持ってくる
○ What does the man suggest the women do?
that節の中の目的語を問うているのにthe manが主語の疑問文で良いかと少し混乱するかもだけど
主節の「提案」とその内容はthe manのものだから主語はthe manで良い。
0062The OED Loves Me Not (ワッチョイ 4e89-4uR0)
垢版 |
2023/01/03(火) 09:19:41.80ID:JwxJ4Pb20
>>60
それじゃ、最初っからそれを言ってくれ。偉そうに。

それはともかく、
You are above [something or someone].
He is above [something or someone].
という表現をあちこち探しまわって、片っ端から昨日、読んでみた。
その結果として僕が導き出した中間結論は、

[Someone or something] is above [something].
つまり「誰か又は何か] が何かの上にある(aobve)という
表現を使うときには、そのあとに抽象名詞のようなものが来ることが圧倒的に多い。
above のあとに、「人や団体や品物」が来るケースはきわめて少ない。

したがって、今回のように
[商品名] is above any [品物].
という言い回しはまれであるらしい。

その一方で、unlike ならそういう用例はいくらでもジャンジャン出てくる。

しかし今回の問題は TOEIC だ。TOEIC は初心者に毛の生えたような
試験なので、そんなに高度なことを尋ねるべき試験ではない。誰でも一瞬にして答えられる
ような問題ばかりでないといけないことになっている。したがって今回の TOEIC の問題は、
あまり適切でない問題なので、無視していいと思う。
0063The OED Loves Me Not (ワッチョイ 4e89-4uR0)
垢版 |
2023/01/03(火) 09:26:58.14ID:JwxJ4Pb20
>>60
ABC is above XYZ. の用例
以下の用例はすべて、ネット上にあるニュース記事から引き出した。

(1) Money means you are above any and all laws.
(CBC より)

この上の例では、「人 (you)」があらゆる法律を凌駕している、という
意味で above と言っている。つまり人が別の人よりも優れている
というような意味で使っているのではなくて、あらゆる法律という
抽象的なものを凌駕している、という意味を表している。

この他のいろんな例を自分で探して読んでみてほしい。ほとんどが
「自分と同類のもの(例えば今回の課題文で言えば主語の製品と
同類の別の製品)」を凌駕している(優れている)という意味で使っている
例文は、きわめて少ない。
0066名無しさん@英語勉強中 (ワッチョイW 4e84-SqYu)
垢版 |
2023/01/03(火) 12:09:48.93ID:UuVQA+KD0
>>62
爺様!
「偉そうに」はアカン。
相手はんはニュートラルのつもりでいてはるよ。
0067名無しさん@英語勉強中 (アウアウウー Sac7-KYZs)
垢版 |
2023/01/03(火) 15:06:27.56ID:lxfyZ81Xa
>>49
疑問と省略含めてこの形を何回もやれば、違和感を抱かなくなるよ
文法的な説明は上でしてくれてる人いるが
理屈で納得しても慣れなければ違和感は消えないし、処理スピードは上がらない
006836 (ワッチョイW a35d-Vgb4)
垢版 |
2023/01/03(火) 23:59:54.90ID:Gy1ZzFBu0
>>41

詳しいコメントありがとうございます.

勉強になりました.
0069名無しさん@英語勉強中 (ワッチョイW 27e3-VpgV)
垢版 |
2023/01/04(水) 00:27:43.39ID:vT2hfdfa0
>>67
同じような疑問文を抜き出してまとめて和文英訳して慣れていこうかなと思います
今はdo youに慣れすぎていてdoes the manですら違和感を覚えてしまう状況なので‥

>>61
文法的に理解できてモヤモヤは消えました
ありがとうございます
理解はできても見慣れない文なのでこれから慣れたいです

>>54
省略をなくした方がさっと理解できそうです、ありがとうございます
0070The OED Loves Me Not (ワッチョイ 4e89-4uR0)
垢版 |
2023/01/04(水) 13:04:58.04ID:C9NTLlyu0
前スレの質問

>>761名無しさん@英語勉強中 (アウアウウー Sa9f-78gd)2022/12/18(日) 05:08:27.76ID:jzOV7yK4a?>>765
>>次のsureはどういう意味ですか?
>>What do you think of using this cover photo on our Web site?
>>★Sure★, that looks great.
=======

この場合の sure の意味合いは、通常の意味として考えると、この文脈では不似合いに感じるわな。
でもこれは、相手が実は
Do you think tht this cover photo would look good on our Web site?
というふうに尋ねたがっているものと考えて、先回りして
Sure と答えていると考えられる。

こういうふうに、相手の質問に対して直接的に、文字通り、論理的に答えるのではなくて、相手の尋ねたいこと
(つまり相手の言外の質問)に対して答えるような場合が日本語にもよくあるが、英語にもあるってことだな。

そういうふうに答えたかったんだが、この質問が出た時には、僕はそういう用例を引き出すことができなかった。
そういう用例を普段から見ることはあっても、いきなりそういうものをネットから引き出そうとしても、うまくキーワードが
思いつかず、結果的にそういう用例をいきなり引き出すなんて無理だわな。

(続く)
0071The OED Loves Me Not (ワッチョイ 4e89-4uR0)
垢版 |
2023/01/04(水) 13:05:26.16ID:C9NTLlyu0
(続き)
諦めていたら、今さっき、たまたま読んでいた文章の中でそういう用例を見たので、紹介する。

“Who is Captain Ahab, sir?”

“★Aye, aye, I thought so. ★Captain Ahab is the Captain of this ship.”

出典:
Herman Melville, Moby Dick; Chapter 16; Project Gutenberg
https://www.gutenberg.org/cache/epub/2701/pg2701-images.html

上の会話は、例の有名な1851年の小説に出てくるのだが、古いものとは言え、基本的にこれに似た会話は
現代小説や映画にもよく出てくる。Aye, aye. は、スコットランド人が昔からよく Yes, yes. という意味で使う
言葉だが、船乗りが特によく使う。かなり最近では軍隊でよく使われる。そのあとに I thought so. と
言っているが、相手が「Captain Ahab って誰?」と言ったのに対して、それに対して直接に答える前に、
I thought so. (思った通りだな) と言っている。これは、文字通りに受け取ると意味が通じていないが、
もちろんのことながら I thought that you didn't know of Captain Ahab yet. という意味だ。

この会話では、質問者が提起している会話と同じく、相手の質問に対して直接には答えず、
相手が本当に尋ねていることや、相手の質問の裏にある背景に対して応答しているのであって、
表向きの質問に対して論理的・直接的に答えているわけではない。こんな用例は、他にもたくさん
英語にもあれば日本語にもある。
0072名無しさん@英語勉強中 (ワッチョイW 03fb-MdUm)
垢版 |
2023/01/04(水) 23:30:26.05ID:loJ+uATd0
名詞の目的格についての質問です。
He is a boy.
のboyは主格補語だから主格
The bird is deep blue.
のdeep blue は目的格
ということらしいですが、
どのように主格と目的格を区別すれば良いのでしょうか?
0077名無しさん@英語勉強中 (ワッチョイW 03fb-MdUm)
垢版 |
2023/01/05(木) 00:17:34.53ID:nHFDLPZ00
>>76
それが、目的格 形容詞に相当する用法
の補語に使われる例
というところで
When I was your age, I was independent of my parents.

という文が例として出てきます。

さらに追加すると
Her eyes, a deep blue, were quite impressive.
のa deep blueが
目的格 形容詞に相当する用法

名詞を修飾する例として出てきます。
0078名無しさん@英語勉強中 (アークセーT Sxbb-9j0N)
垢版 |
2023/01/05(木) 00:42:57.49ID:nV2wQr1Rx
ジーニアス英和大辞典をAndroidに入れたいのですがどこで買えるでしょうか?
0081名無しさん@英語勉強中 (ワッチョイW 03fb-MdUm)
垢版 |
2023/01/05(木) 01:11:00.90ID:nHFDLPZ00
そうなんだ。じゃあ、google play からロゴビスタの無料辞書アプリをダウンロードして、
そのアプリの中からジーニアス英和大(ジーニアス英和、和英という中辞典ではなく)
が買えないのかな?
0082名無しさん@英語勉強中 (アークセーT Sxbb-9j0N)
垢版 |
2023/01/05(木) 01:24:10.81ID:nV2wQr1Rx
ありがとうございます。
google play からロゴビスタの無料辞書アプリをダウンロードをしたのですが選べるのがジーニアス英和、和英でジーニアス英和大辞典がないんですよね…。
0086The OED Loves Me Not (ワッチョイ 4e89-4uR0)
垢版 |
2023/01/05(木) 06:58:10.46ID:br7ssZwk0
>>74 >>77
I was your age の場合は、
I was of your age っていう言い回しの of がだんだん省略されて出来上がった。

his eyes, a deep blue, were... の場合は、
his eyes, of a deep blue, were... の of がだんだん省略されて出来上がった。

しかし
He is a student. では、(*)He is of a student. なんていう言い回しは大昔からなくて、
最初から He is a student. だった。

そういう違いがあるんだとさえ覚えておけば、主格補語とか目的格だとかいう
不気味な文法用語なんて覚える必要もなければ使う必要もない。
ただ、文法用語を駆使した解説を読むときには役立つけど。
0088名無しさん@英語勉強中 (ワッチョイW 03fb-MdUm)
垢版 |
2023/01/05(木) 10:46:35.26ID:nHFDLPZ00
>>86
When I was your age
については英文法解説のp22の解説で、
記述の対格(Accusative of Description)と呼ばれる用法で、
When I was (of)your age…のように前置詞の脱落によるものかどうかで、
昔の文法学者の間に論争があった。
とあります。論争の経緯を知ることもできない書き方です。

目的格とはなんなのでしょうね。問題意識を持っていればそのうちわかる日も来るのかもしれません。
0090名無しさん@英語勉強中 (ワッチョイ 9ae2-U0ia)
垢版 |
2023/01/05(木) 11:24:41.13ID:189VZVjU0
God said, Let there be light: and there was light.

神は「光あれ」と言われた, すると光があった. - 研究社 新英和中辞典
0091名無しさん@英語勉強中 (ワッチョイW 03fb-MdUm)
垢版 |
2023/01/05(木) 11:31:01.76ID:nHFDLPZ00
>>88の続き
記述的対格とは
Onions1929の用語。
形状、色彩、年齢、価格、職業などをあらわす名詞で形容詞的に用いられる。通例ofを補うことができる。
と書いています。

主格補語として
The earth is ★the shape of an orange.★
★What price★ is that article.

目的格補語として
What colour shall I paint your door?

名詞句の後に置かれて、これを修飾する。
Why can’t you be like other girls your age?
0093名無しさん@英語勉強中 (ワッチョイ 4e84-0ejh)
垢版 |
2023/01/05(木) 12:25:34.85ID:ahe82Fw+0
>>89
ご期待に沿える回答ではないかもです。
私の考え(多少ネットで同様のイメージを語る方もいますが)
で多くの文法書(一部除く)で裏は取れません。

動詞の原形のイメージは「未確定」です。
過去形は「と言うことがあった」とか「した」と
現在より前に動詞の所作が確定したことを示します。
現在形は過去から現在、未来まで動詞の所作が継続していることを
示します。
ですので、There be sharks. を過去形、現在形で書くと、
・There were sharks.
・There are sharks.
となり、意味として各々
・(かつて)サメがいた。(今はいるかどうか分からない)
・(昔、現在、(当面の)未来)サメがいる。
過去形はさておき、ですので基本的には現在形を使っても構わないと思います。
ではなぜbe動詞の原形を使うかというと、
件のvtuberさんが、「以前見た」、「誰かにサメがいると聞いた」、
「こういう環境には普通サメがいる」など確信があり、
《今は目の前にいないけど》「サメがいる(はずだから、そのうち出てくるよ)。」
と言うことを示していると思います。
現在形の場合は目の前にサメがいるかもしれないし、いないかもしれない。
それは同席している人が分かるだろうし、いるならカメラでズームされるかも知れません。
とにかくいることは確定してるわけです。

そんな違いが原形と現在形にはあると思います。

ご批判もあるかもしれませんがその時は真摯に受け止めまた考えます。

Do that be~
はすみません。イメージが湧かず答えられません。
0094😉三年英太郎🌈 ◆3CZBjOt3.Y (ワッチョイW 9a86-fgND)
垢版 |
2023/01/05(木) 12:55:33.49ID:YnJKlPu80
>>88
of が省略されてるなら、my age は of の目的語だから目的格だろうけど、省略されてないからね

これは I listen to Chopin. と同じでメトニミーなんだよ。江川は認知文法知らない、ゴリゴリ伝統グラマリアン
0095The OED Loves Me Not (ワッチョイ 4e89-4uR0)
垢版 |
2023/01/05(木) 13:09:59.31ID:br7ssZwk0
>>89
意外なところに be がいきなり出てきたら、
There be sharks. の場合だったら
次の2つの場合である可能性が高い。

(1) There'd be (= There would be) sharks.
と言っているうちに 'd がそのあとの be によって
かき消されてしまって完全に消失し、ふだんから
最初から d' を発音しない人がどんどん出てきて、
そのうちにそれを書く時でさえ 'd を消したままで
There be sharks.
と書いてしまう連中がたくさんいる。

(2) 二つ目の可能性としては、Ebonics (African-American English)
である可能性がある。この方言においては、
There are sharks. という意味で最初から There are sharks.
と言っている場合がある。この黒人英語特有の文法については、
泉山 真奈美という人が黒人英語に詳しく、それについてずいぶん前から
流布している2冊の本の中で詳しく論じてくれている。もちろん、
その2冊の本の10倍くらい詳しい、英語で書かれた次のような本もある。

African American English: A Linguistic Introduction (English Edition)
by Lisa J. Green (著)
0097The OED Loves Me Not (ワッチョイ 4e89-4uR0)
垢版 |
2023/01/05(木) 13:14:14.35ID:br7ssZwk0
>>95 の (1) に関しては、現にネット上に
(1) There be sharks.
という英文と共に、
(2) There'd be sharks.
(3) There would be sharks.
という英文もちらほら見かける。おそらくこの3つは、
同じような系列の話題(ギャグ?)なんだろうと思う。
0099🐤三年英太郎🍓 ◆3CZBjOt3.Y (ワッチョイ 9a86-4uR0)
垢版 |
2023/01/05(木) 15:30:12.26ID:YnJKlPu80
英語で機能論的になんちゃら格だというのは古い
副詞的対格とか言うのもそう
そんなものに付き合ってる暇はない

いまどきの英語学で格といえば、
生成文法の格と、格文法の深層格だ!
0101名無しさん@英語勉強中 (テテンテンテン MMb6-9yt5)
垢版 |
2023/01/05(木) 16:55:59.44ID:nt2kxIbNM
>>94
my age がメトニミー?
ウソくせー

Chopin
なら
Chopinの音楽の比喩だけど
じゃあ
my ageで、何を意味してんだよ。

アホ太郎の話は話半分で聞いた方がいい。
0102名無しさん@英語勉強中 (テテンテンテン MMb6-9yt5)
垢版 |
2023/01/05(木) 17:05:00.61ID:nt2kxIbNM
あ、失礼
my age

a man of my age を意味してるってことか・・・

でも本当かな
このmy ageを記述の対格ではなく、メトニミーとして説明してる本あるの?
0103名無しさん@英語勉強中 (テテンテンテン MMb6-9yt5)
垢版 |
2023/01/05(木) 17:19:20.40ID:nt2kxIbNM
記述の対格は主格補語の位置に来る以外に
a car the size of a killer whale
のように名詞を後置修飾もする。

メトニミーと解釈するのは無理がある。
0104名無しさん@英語勉強中 (ワッチョイW 4e89-WAdv)
垢版 |
2023/01/05(木) 20:10:03.95ID:6kA/+va30
下記の訳で、only last summer が、つい昨夏も、となっていますが、このような解釈になるのはなぜでしょうか?

(EN) my dear mother used to say I should never have a comfortable home; and only last summer I proved myself perfectly unworthy of one.

(JP) 母上もなにかにつけ言っていたっけ。心安らぐ家庭なんて、おまえには一生もてやしないよ、と。なにをかいわんや、つい昨夏も、そうしたことにはてんで不向きであることを実証してしまったところだ。

嵐が丘対訳サイト
http://www.f.waseda.jp/guelberg/germanic_languages/faroese/fo_verb/quellen/bronte/bronte01_03.htm
0105The OED Loves Me Not (ワッチョイ 4e89-4uR0)
垢版 |
2023/01/05(木) 20:15:27.67ID:br7ssZwk0
>>104
> (EN) my dear mother used to say I should never have a comfortable home; and only last summer I proved myself perfectly unworthy of one.

only last summer を普通に訳せば、「つい昨夏には」ってことだろ?
だからそのように訳してもいいんだけど、文脈の中に、
「母親が~だと言っていたんだけど、私は自分で、母親の言う通りだということを自分で証明してしまった」
と言っている、だから「つい昨夏にも」と訳していいのだ、と言える。

それ以上に。どう説明してほしいのかな?
0106名無しさん@英語勉強中 (ワッチョイW 4e89-WAdv)
垢版 |
2023/01/05(木) 20:26:15.18ID:6kA/+va30
>>105
どうしても、つい昨夏にもの「も」が引っかかるのですが、
母親のエピソードは結婚できないという指摘で、
自分で結婚できないということ証明したことは
関連しないような気がしてひっかかりました。

母親が指摘して、つい昨夏に、母親の指摘を自分が証明した
というのが理解している範囲なのですが、
国語の問題ですかね、、、
0107The OED Loves Me Not (ワッチョイ 4e89-4uR0)
垢版 |
2023/01/05(木) 20:27:07.06ID:br7ssZwk0
>>104
例えばもっと単純な例を挙げると、

Tom was a very careless boy, and ★only last summer★ he lost his expensive wallet.

という例文があったとしたら、
Tom は~だった。それで ★つい昨夏には★ あいつは値段の張る財布をなくしてしまったんだ。

と訳してもいいけど、次のように訳した方が日本語らしくて気が利いている。

Tom は~だったけど、★つい昨夏にも★ あいつは値段の張る財布をなくしてしまったんだ。

要は、翻訳上の工夫に過ぎない。要は、今回の問題は英語の問題ではなくて、日本語の問題に過ぎないと思うけど。
0108The OED Loves Me Not (ワッチョイ 4e89-4uR0)
垢版 |
2023/01/05(木) 20:37:00.40ID:br7ssZwk0
>>106
その点が引っ掛かっているんだったら、最初からそう言ってくれれば、何度も回答を書く必要がなかったんだ。

> (EN) my dear mother used to say I should never have a comfortable home; and only last summer I proved myself perfectly unworthy of one.

母親は、「お前は居心地のいい家庭なんて絶対にもてないよ」と言っていた。
and only last summer
私は自分で、居心地のいい家庭なんて持つ資格もないということを賞めぢしてしまったのだった。

だから前半と後半は、同じようなことを言っているわけだ。
母親は ABC だといった。それに対して、自分は自分でそうなんだということを証明してしまっていた。
だから、only last summer は「つい昨夏にも」と訳していい、というよりも、その和訳は気が利いていると思う。

つまり、only last summer と言う限りは、ふだんから何度も何度も、普段から「私がそんな家庭なんて持てっこないということ」を
証明していたんだ。だからこそ only last summer という言葉が出てくる。

もしも母親がそんなことを予言していて、「実は母親の言う通りだということ」を私が自分で証明してしまったのが、
その数十年後である去年の一回だけなのであれば、別の言い方をするはず。
たとえば
my dear mother used to say I should never have a comfortable home.
●But I had long refused to believe her. Then, last summer I finally● proved myself perfectly unworthy of one.

上のようにでも書くだろうよ。
0109名無しさん@英語勉強中 (ワッチョイW 4e89-WAdv)
垢版 |
2023/01/05(木) 20:46:39.16ID:6kA/+va30
>>108
返信ありがとうございます。

確認させてください。

つまり、全文と関連していれば、「にも(過去に何回も起きていた)」

短文では(には(ようやく))という認識になるということでいいのでしょうか?


Only yesterday did he begin to realize what was going on.
昨日になってようやく彼はことの次第がわかり始めた。
0111The OED Loves Me Not (ワッチョイ 4e89-4uR0)
垢版 |
2023/01/05(木) 20:47:44.14ID:br7ssZwk0
>>106
さっきの解説はちょっとまずかったかな?only last summer の only そのものは、後者の場合でも問題ないか。

my dear mother used to say I should never have a comfortable home.
★But I had always refused to believe her. And it was only last summer that★ I finally proved myself perfectly unworthy of one.

もしも何十年ものあいだ「いやいや、母親の言う通りになんかならないぞ」と思っていて、つい去年の夏になって
初めて「やっぱり母親の言う通りだったな」ということを悟った、というのであれば、上のようにでも著者は書くんじゃないかな?
もちろん、僕の英文はまずいとは思うけど、考え方としてはこれでいいのではないだろうか?
0113The OED Loves Me Not (ワッチョイ 4e89-4uR0)
垢版 |
2023/01/05(木) 21:08:13.57ID:br7ssZwk0
>>109
> 短文では(には(ようやく))という認識になるということでいいのでしょうか?

> Only yesterday did he begin to realize what was going on.
> 昨日になってようやく彼はことの次第がわかり始めた。

===========

短文だからそうなるって言うわけじゃなくて、その短文の中に最初から、
その only yesterday が「ようやく」という意味になるための仕掛けがある。

begin to realize と書いてあるから、ここでは「昨日になってようやく」という
意味にしかならないんじゃないかな?
0115名無しさん@英語勉強中 (ワッチョイ 9a79-dayM)
垢版 |
2023/01/05(木) 21:15:19.04ID:oE/ub2Yg0
>>106
母親が「なにかにつけ言っていた」ってことは、心安らぐ家庭を持てないと母親に判断される
ようなイベントが色々あったんでしょ。今の借家の状況も安らげるものではない中、
昨夏の出来事を「累加」して安らげる家庭を持つ適性の無さを痛感してる感じの「も」。
0116The OED Loves Me Not (ワッチョイ 4e89-4uR0)
垢版 |
2023/01/05(木) 21:17:24.38ID:br7ssZwk0
>>114
確かに文脈だとしか、僕には説明できないなあ。他の人なら、もしかしたら
もっといい説明をしてくれるかもしれないから、諦めないで他の人の回答を待ってみてほしい。
僕自身も明日以降になって頭を冷やして休めたあとだったら、
もしかしたらもっといい解説ができるかもしれないし。
0117名無しさん@英語勉強中 (ワッチョイW 4e89-WAdv)
垢版 |
2023/01/05(木) 21:19:34.74ID:6kA/+va30
例えば

Only yesterday there was another terrible accident in New York.
Only yesterday, the Jihad fighters burnt down two housing estates in Ambon.

The evaluation report on the Daphne programme became available only yesterday.  

これらは過去に何回も起きている

Only yesterday, more than 40 demonstrators were arrested for attempting to commemorate the anniversary of the Chernobyl disaster. イベントが特異

これらは初めて起きた

という振り分けでしょうか
まだ、上手く振り分けられないですが、慣れて行きます
ありがとうございました!
0118The OED Loves Me Not (ワッチョイ 4e89-4uR0)
垢版 |
2023/01/05(木) 21:27:06.65ID:br7ssZwk0
>>114
>>115 のコメントを読んで、ここでは used to が使われていることを再確認した。
うっかりそれを見落としていた。My dear mother used to say と used to
が使われているんだから、 >>115 の言う通り、母親がいつもいつも
繰り返し繰り返しそれを言ってたわけだな。そのあとに
only last summer I proved myself perfectly unworthy of one となっていれば、
「去年の夏になってやっと」なんていう意味にはならず、「つい去年の夏に(も)」
という和訳をしたくなるような意味合いになるはずだよね。

もしも「去年のなつになってやっと」という和訳をつけたくなるような文脈なら、
and only last summer I ★proved my mother to have been completely wrong.★
とでもなるんじゃないだろうか?
0120名無しさん@英語勉強中 (ワッチョイW 4e89-WAdv)
垢版 |
2023/01/05(木) 21:43:27.12ID:6kA/+va30
>>118
つまり、
母親とのエピソード 複数
本人の自己による証明 単体
の総称として「にも」とした、という可能性もあるということですよね
(本人の自己による証明が複数の場合もある)
という認識ですね
0121名無しさん@英語勉強中 (アウアウウー Sac7-KYZs)
垢版 |
2023/01/05(木) 21:50:55.71ID:NKgMQSMka
動詞が何回かとかの、アスペクトの問題か
動詞に単発か複数というのが内包されてるのもあるし、ケースバイケースの動詞もある
ケースバイケースの動詞の場合、解釈が分かれることもあるのかな
相を限定するために何かをつけたり、逆につけてないなら相は~という消去法をネイティブはやってるのかな
0128名無しさん@英語勉強中 (ワッチョイ 9a79-dayM)
垢版 |
2023/01/05(木) 22:12:58.04ID:oE/ub2Yg0
ただ、"only last summer〜" 自体に助詞(も)の機能は無くて、文脈上、
昔の母の指摘と現在のごたごたに、同種の出来事として "only last summer〜"
の文の表す出来事が加えられたため累加・強調の「も」が付けられたとご理解くだされ。
0130名無しさん@英語勉強中 (ワッチョイW 03fb-MdUm)
垢版 |
2023/01/06(金) 00:10:01.06ID:6ozC1fkP0
何をか言わんや(→これ以上言うことはない)
I have nothing more to say about it.|What’s (there) left to say?

次のことを言えば十分だと思うが
というのが、何をかいわんや、ですね


only last summerは昨夏ーしたばかりだ
です


つい昨夏も、という訳は正しいのかな?
0131The OED Loves Me Not (ワッチョイ 4e89-4uR0)
垢版 |
2023/01/06(金) 08:00:28.83ID:NO/7EuvL0
>>130
>>つい昨夏も、という訳は正しいのかな?

日本人はとかく、「も」をつけることによって言葉を和らげようとしてしまうんだろうと思う。
だから、「も」が論理的には必要なくても、つければ日本語らしくなって、流れがよくなる。

わがままだね、君も。

この上の文は、ある日本のテレビドラマで登場人物の一人が別の人に向かって言った言葉。
別にこの文の前に何か別の文脈があったわけではない。誰か別の人がわがままだった
というわけではまったくない。それなのに、この登場人物のみならず、日本人はこういうときに

わがままだね、君 ★は★。

と言えばいいときにも「も」を使いたくなる。もちろん、この登場人物がふだんから
「人間っていうものは、とかくわがままだな。どいつもこいつも、わがままだよ」
と思っているのだろうな、と視聴者は想像することもできる。しかしそうでないかもしれない。
そんなことを匂わせる舞台背景(文脈)は、このドラマに置いては何もなかった。
それなのに、ともかく文脈も何もなしにいきなり「も」を日本人はつける。

それを英語に訳すときには、「も」を省いて英訳した方が英語らしい場合がほとんどだということになる。
したがって、日本語において「も」が現れたからと言って、すぐにそれにくっついている英文の
どこに「も」という意味合いが含まれているのかを論議することにはあまり意味がない場合も多い。
0132The OED Loves Me Not (ワッチョイ 4e89-4uR0)
垢版 |
2023/01/06(金) 10:25:38.11ID:NO/7EuvL0
>>104
> (1) my dear mother used to say I should never have a comfortable home;
(2) and ★only last summer★ I proved myself perfectly unworthy of one.

原文を上のように (1), (2) を分けてみる。執拗にこの問題に取り組んでみよう。
元の質問者さんは、僕のこれからのコメントが
うざければ、無視してくれたらいい。返事をしなくてもいい。

(1) 僕なりの試訳: 母親は、ふだんからよく "お前は絶対に居心地のいい家庭なんて持てっこないよ" と言っていた。

(2) 僕なりの試訳: そして、★つい去年の夏のことだったが、★ 私は(確かに、母親の言う通り)
そんな家庭は絶対に持てっこないと、自分で証明してしまうことになるのだった。

この only this summer を直訳すると「つい去年の夏のことだったが」とでもなるだろうけど、
このように主人公が「自分には居心地のいい家庭なんか絶対に持てっこない」ということを
再確認するような場面が何度も何度もあったんだろうな、と、この小説を読んでいると
読者には感じられてくる。少なくとも「僕には」そのように感じられてくる。

だから元の翻訳者はここで、「つい去年の夏のことだったが」とか
「つい昨夏のこと」とか「つい昨夏には」とは言わず、(こんなことはよくあったのだが)再び(またもや)
「つい昨夏にも」こんなことがあったのだ、と書いているわけだと僕は思う。

そう考えると、この翻訳者はやはり、この小説の筋をすべて深く知り、英語をも深く理解し、
そのうえで、最も日本人にとって日本語らしい言い回しでこういう雰囲気を表すときに最もふさわしい
表現として、「この昨夏にも」というふうに「も」をつけたんだろうと思う。

すでに僕がこのスレで言ったことを繰り返しているに過ぎないかもしれないが、
あえてこれを消さないでここに投稿する。同じことであったとしても、疲れていた夕べよりも、
疲れを休めたあとの今朝の方が、夕べよりもさらに丁寧に言葉を選んで説明できているかもしれない。
0133The OED Loves Me Not (ワッチョイ 4e89-4uR0)
垢版 |
2023/01/06(金) 10:38:38.15ID:NO/7EuvL0
>>132 で述べたような問題もあるので、英語を勉強するときに、特に古典的な文学小説を読むとき、
その日本語訳と対照させて読み進めるならそうしてもいいけど、いちいちこういう細かい点まで
指摘して、「なぜここはこういうふうに訳してあるのか」などと論議しても不毛だと僕は思う。

一般的に、そもそも日本語訳なんかと対照させて読むこと自体が邪道だ。
英語だけで読めばいいのだ。そしてもしも、英語だけで読んだらチンプンカンプンになるんだったら、
それはその人の英語力がまだまだ足りないからだ。だから5年、10年、あるいは30年ほど待って、
英語力が思いっきりついたころに、再びその作品に戻ればいい。

日本語訳なんてものを横に置かないといけないような作品はずっと後回しにして、まずは
絵本を読む。それに飽き足りなくなったら、100ページから300ページくらいの童話を読む。
それを50冊か100冊ほど読み切ったあと、それにつくづく嫌気が差したら、今度は
大人の娯楽小説または young adults 向けの本を数十冊ほど読む。それでも飽き足りなくなってきたら、
今度は大人のノンフィクションなど、少し真面目な大人向けの読み物、そしてそれを数十冊ほど読んで、
それでも飽き足りなくなったら、今度は歴史書や純文学や人生論めいたものを読む。

そのずっとあとで、今回の Wuthering Heights のような、19世紀の純文学小説を読むのだ。
そのとき、いっさい日本語訳は参照しない。もし参照するなら、自分がそういう小説を
日本語に翻訳して出版したいから読む、つまりプロの翻訳家による日本語の使い方や
翻訳の技巧を学び取るために邦訳を読む。しかし決して「英語学習」のために邦訳を読んではならない。

英語学習は、英語の超初心者がやることだ。邦訳を読むのは、プロの翻訳家たちの日本語の運用能力を
知るために、あるいは日本語に訳しにくい英語表現をどのように工夫して日本語に変えているかを
観察するために行う、プロ志向の人たち(今すぐにでも翻訳家として独立したい人)がやるべきことだ。

そのように僕は考える。
0134名無しさん@英語勉強中 (アークセーT Sxbb-9j0N)
垢版 |
2023/01/06(金) 10:58:41.67ID:+TGTPTXBx
>>83
大修館に問い合わせてみました。
ジーニアス英和大辞典のandroid版はないとのこと。
翻訳家の方はPCで使うし、iphoneでは昔から辞書を買う方は多いけれどandroidだと辞書を買う人が少ないため、
androidでは辞書全般が少ないと言われました…。
なんでですかね?
0135名無しさん@英語勉強中 (アウアウウー Sac7-KYZs)
垢版 |
2023/01/06(金) 11:03:12.06ID:tVF1BKuda
onlyは強調→強調の意味を経験則で直観する→訳す
これが理想的な流れだな

この形のonlyはこう訳す→?
ってのはやり方がそもそも間違ってると思う
onlyと一対一で対応する日本語はないので、新しく日本語を生み出しでもしないとできないアプローチ
0138名無しさん@英語勉強中 (ワッチョイ 33a1-GRDc)
垢版 |
2023/01/06(金) 15:25:41.83ID:dowh5rwa0
https://twitter.com/4chanGreen/status/1610694563328397316?s=20&t=41axIgAJLfg7xOeSx0g7Wg
この書き込み、日本人じゃないと思うのは俺だけ?
英語ネイティブが非ネイティブの片言で面白い喋り方を想像して書きましたという感じがする
日本人ならむしろ文法は正しいけど堅苦しい学校英語的な英文を書くと思う
まあだからなんだって話なんだけど
https://twitter.com/5chan_nel (5ch newer account)
0139名無しさん@英語勉強中 (アウアウウー Sac7-KYZs)
垢版 |
2023/01/06(金) 15:52:46.77ID:JtlvHetTa
良い辞書アプリがないからか、買う人少ないからか
何方が先だったか怪しいね
EPWING手に入れて使うことはできるんじゃないかな?
今、メルカリを見てみたら、一万円くらいでCDROM版が売ってたわ
0140名無しさん@英語勉強中 (ワッチョイ 1af0-TwI4)
垢版 |
2023/01/06(金) 15:52:57.80ID:EPZetn5b0
>>138
ひろゆきの英語もこんなんだぞ
0141😉三年英太郎🌈 ◆3CZBjOt3.Y (ワッチョイW 9a86-fgND)
垢版 |
2023/01/06(金) 16:02:37.97ID:DM6qQSDM0
Androidユーザーがそもそも少なかったからな
前はiOS用しか提供してなかった大手メーカーのアプリもいっぱいあった

ジーニアスはbiglobeが売ってたのに、6版でlogovistaに変わった。G大がAndroid 市場から消えたのもこれと関係があると思われる
0144名無しさん@英語勉強中 (テテンテンテン MMb6-9yt5)
垢版 |
2023/01/06(金) 17:47:50.02ID:hwkZzZfuM
iphone上の辞書と、androidのそれでは
プログラム的にそんなに違うものなのだろうか?
もし、ちょっとした差異しかないなら
両方とも売ればいいのに。
0145名無しさん@英語勉強中 (テテンテンテン MMb6-9yt5)
垢版 |
2023/01/06(金) 17:51:26.07ID:hwkZzZfuM
ゲームとかなら、OSによってプログラムの差異が大きそうだけど
辞書なんて基本テキストベースのデータでしかないのでは?
0147名無しさん@英語勉強中 (アウアウウー Sac7-eOZT)
垢版 |
2023/01/06(金) 20:18:25.46ID:S947gj12a
>>94
my age がメトニミー?
ウソくせー

Why can’t you be like other girls your age?
a car the size of a killer whale

とか「記述の対格」が名詞を後置修飾するケースはどうなんだよ 苦笑

アホ太郎の話は眉唾モノが多いなw
0150名無しさん@英語勉強中 (ワッチョイ 9af0-rE+U)
垢版 |
2023/01/06(金) 20:40:17.80ID:K4rtivCh0
去年英語のやり直しを始めて中学文法をおさらいできる本のあと
どんどんスラスラ英作文とターゲットで単語と一億人の英文法3周読みました(全部家にあったやつ)
同時にキンドルで児童向けの洋書を読んでいます
で、ふとおさらいしてない高校の文法が気になったのだが
過去完了とか仮定法とか一億人にちょっと載ってるのとか読んでなくて
洋書読んでると結構でてくるから一応認識はしてるんだけど
もっとちゃんと高校英語もやったほうがいいかな?と思いまして
その場合エバーグリーンと問題集とかでいいですかね?
0151名無しさん@英語勉強中 (アウアウウー Sac7-eOZT)
垢版 |
2023/01/06(金) 20:46:01.38ID:S947gj12a
>>149
お前の頭がやられてるんだろw低能
名詞を後置修飾する場合はメトニミーでは説明できないと言っている。

I was your age,
のyour ageと
other girls yourage
のyour ageとが全く機能の違うものと理解するなんて、何の利点まないし
アホとしか思えん
0156名無しさん@英語勉強中 (テテンテンテン MMb6-9yt5)
垢版 |
2023/01/06(金) 20:58:17.10ID:+iymUpxyM
>>155
ホモ太郎は楽天をアボーンしてるからね
で、この件についてはどう思いますか?
ホモ太郎の完全なインチキでしょ?
0157名無しさん@英語勉強中 (テテンテンテン MMb6-9yt5)
垢版 |
2023/01/06(金) 21:01:52.43ID:+iymUpxyM
Why can’t you be like other girls your age?
のyour ageがメトニミーではないなら
I was your ageのyour ageも同様に解するべき

すべての学問にオッカムの剃刀が指針となるとは限らないかもしれんが
英文法については従うべき。
0158😉三年英太郎🌈 ◆3CZBjOt3.Y (ワッチョイW 9a86-fgND)
垢版 |
2023/01/06(金) 21:11:24.18ID:DM6qQSDM0
I was your age
her eyes a deep blue

いずれもメトニミーで説明できると言ってるのにどうしちゃったのこの人?🤔

不幸にも「記述の対格」というおそらく古典語の文法に由来する仰々しい用語で便宜的に呼ばれてきたので、勘違いしちゃった人がyour ageとa deep blueは対格→今の目的格だ!と思ったのであろう。江川先生、そうでしょ?
0160名無しさん@英語勉強中 (テテンテンテン MMb6-9yt5)
垢版 |
2023/01/06(金) 21:21:04.24ID:+iymUpxyM
Why can’t you be like other girls your age?

your age がメトニミーで説明できるわけない

またホモ太郎、論破されて逃走w
0162😉三年英太郎🌈 ◆3CZBjOt3.Y (ワッチョイW 9a86-fgND)
垢版 |
2023/01/06(金) 21:24:53.97ID:DM6qQSDM0
I ≠ your age
her eyes ≠ a deep blue

論理的にはおかしいのである。「ショパンを聞く」も正しくはショパンの作曲した作品の演奏を聞く、が正しい

がしかし、言語の実際の運用は、このような緩やかなつながり=比喩に満ちている。こんなものをわざわざ of 対格の省略と説明する必要はない。20世紀前半で廃れた議論だろう。江川の文法がカビ臭いのは、参照してるのがそういう時代の文法書だから
0163名無しさん@英語勉強中 (テテンテンテン MMb6-9yt5)
垢版 |
2023/01/06(金) 21:30:57.07ID:+iymUpxyM
Why can’t you be like other girls your age?

your age がメトニミーであるとするなら
girls と your age が同格の関係であることになる。

しかし、your age は明らかにgirlsを限定修飾している。
your ageは、メトニミーではなく、形容詞的な働きをしている。

ネイティブの学者も、記述の対格には形容詞的な働きがあるとしている。


歴史的にofが省略されたものなのか?
目的格と呼んでいいのか?
という話にすり替えて、ホモ太郎は逃走する気だが

明らかに
I listen to Chopin.
は全く関係ない。
0164名無しさん@英語勉強中 (スップ Sd5a-sZaX)
垢版 |
2023/01/06(金) 21:32:35.98ID:2jk0LRjZd
I remain to be seen it.
でGrammarlyでエラーが出ないのですが、こちらも文法上正しいのでしょうか?正しいならば意味はどうなるでしょうか?
0165名無しさん@英語勉強中 (テテンテンテン MMb6-9yt5)
垢版 |
2023/01/06(金) 21:35:07.76ID:+iymUpxyM
>>164
文法的におかしいでしょ。
0166名無しさん@英語勉強中 (テテンテンテン MMb6-9yt5)
垢版 |
2023/01/06(金) 21:44:38.76ID:+iymUpxyM
ホモ太郎は知識があっても
地頭が悪いから
ときどきトンデモないことを言い出すのなwwww
0167名無しさん@英語勉強中 (テテンテンテン MMb6-9yt5)
垢版 |
2023/01/06(金) 21:50:29.65ID:+iymUpxyM
>>164
be seenという受動態の動詞の後にitがある。
ということは
(能動態に態変換したとき)第4文型か第5文型の可能性を
考える。
しかしseeは第4文型はつくらない。
そして第5文型の場合も目的格補語に(代)名詞をとることはない。

よって間違い。
0168名無しさん@英語勉強中 (テテンテンテン MMb6-9yt5)
垢版 |
2023/01/06(金) 21:56:46.05ID:+iymUpxyM
>>162
偉そうに言ってるけど
いざ江川先生と対面したらボコボコにされるだろうねw

江川「じゃあ君、Why can’t you be like other girls your age?
のyour age が本当にメトニミーで説明できるのかね?」
ホモ太郎「・・・・・」
0169名無しさん@英語勉強中 (アウアウウー Sac7-KYZs)
垢版 |
2023/01/06(金) 22:16:14.24ID:ds5foHR8a
そもそも格は名詞がどのような標識と意味を取るかという概念
名詞そのものでなく他の部分、例えば位置が標識のこともあるし、無標識も理論の内
メトニミーは意味に焦点あてた概念
別に対立するものではない
0170名無しさん@英語勉強中 (テテンテンテン MMb6-9yt5)
垢版 |
2023/01/06(金) 22:24:58.85ID:+iymUpxyM
>>169
名詞のメトニミーが形容詞的役割をする例はない。
ショパンという名詞は、ショパンの曲という名詞の比喩である。

繰り返す
Why can’t you be like other girls your age?

your age がメトニミーであるとするなら
girls と your age が同格の関係であることになる。
0171名無しさん@英語勉強中 (テテンテンテン MMb6-9yt5)
垢版 |
2023/01/06(金) 22:26:30.60ID:+iymUpxyM
>>170
そして
所謂記述的対格が形容詞的な働きをするということを
認めれば
メトニミーの話なんて全く介在する必要はない。
0172エキセントリックなソフィスト ◆BWpafMA3P8me (ワッチョイ 2796-ChaJ)
垢版 |
2023/01/06(金) 22:26:53.73ID:G54U/FLs0
3年ちゃんが逃げて、議論が終ったようなので、私の意見を。

your age は、The earth is the shape of an orange.と同じ用法でしょ。これは、メトニミーとは呼ばない。

「メトニミー」とは、「示したいもの」の代わりに、近接する「別のもの」を利用して伝える比喩表現だからだ。shapeが言いたいことの別物だとは思えない。
0174名無しさん@英語勉強中 (テテンテンテン MMb6-9yt5)
垢版 |
2023/01/06(金) 22:31:26.14ID:+iymUpxyM
>>172
アホ太郎は
the shape of an orange

a thing of the shape of an orange
みたいなモノを表す「メトニミー」とイイタイのだと思う。
0175名無しさん@英語勉強中 (テテンテンテン MMb6-9yt5)
垢版 |
2023/01/06(金) 22:33:45.60ID:+iymUpxyM
>>174
これは主格補語の場合は
なんとか通用する話だけど

名詞を後置修飾する場合については無理がある。
a car the size of a killer whale
0176名無しさん@英語勉強中 (テテンテンテン MMb6-9yt5)
垢版 |
2023/01/06(金) 22:39:53.68ID:+iymUpxyM
>>170
>>名詞のメトニミーが形容詞的役割をする例はない。

こう言い切ってしまったが、十分に検討したわけではないので
違うかもしれない。

とにかく
記述的対格に形容詞的働きを認めれば
メトニミーの話なんて介在する必要がない
というのが結論。
0177名無しさん@英語勉強中 (アークセーT Sxbb-9j0N)
垢版 |
2023/01/06(金) 22:43:49.17ID:+TGTPTXBx
>>174
>> そして第5文型の場合も目的格補語に(代)名詞をとることはない。
ココがちょっと気になったのですが、これはseeの場合は目的格補語に(代)名詞をとることはないという意味でしょうか?
(他の動詞だとある)
0180名無しさん@英語勉強中 (テテンテンテン MMb6-9yt5)
垢版 |
2023/01/06(金) 22:53:33.78ID:+iymUpxyM
>>177
This song made her a famous singer.
とか。
0181名無しさん@英語勉強中 (テテンテンテン MMb6-9yt5)
垢版 |
2023/01/06(金) 22:55:50.25ID:+iymUpxyM
>>179
イェスペルセン大先生説は色々反論があるので
彼の主張が絶対ではない。

そもそも
of 対格 を認めるかどうかの話をしているのではない。
形容詞的機能を認めれば
メトニミーの話なんて介在する必要がないと言っている。
0182😉三年英太郎🌈 ◆3CZBjOt3.Y (ワッチョイW 9a86-fgND)
垢版 |
2023/01/06(金) 22:56:09.41ID:DM6qQSDM0
θ role的ておかしいな

フィルモア大先生の格文法的な格はない
それに影響を受けたθ役割的にも役割担ってない(だって意味役割は動詞があたえるんでしょ。ここには動詞がないもの)
0184😉三年英太郎🌈 ◆3CZBjOt3.Y (ワッチョイW 9a86-fgND)
垢版 |
2023/01/06(金) 23:07:08.71ID:DM6qQSDM0
I → my age
her eyes → The color of her eyes

>メトニミー(換喩)表現とは近接性あるいは隣接性に基づく比喩表現である」

彼女の目は緑だ、というとき「彼女の目」は「彼女の目の色」のことを言ってる

メトニミーで問題なし
0187名無しさん@英語勉強中 (テテンテンテン MMb6-9yt5)
垢版 |
2023/01/06(金) 23:22:49.71ID:+iymUpxyM
Her eyes, a deep blue, were quite impressive.
へー、この場合は、a deep blue じゃなく、Her eyesがメトニミーなんだ。
そうすると
この文は「彼の眼の色はとっても印象的だった」ていう意味の文なんだ。
本当だろうか?勝手にそんな捻じ曲げた解釈していいのだろうかw

そういう解釈だろうが
a deep blue がそういう色のモノというメトニミーと解しても
まぁこの場合は
Her eyesとa deep blueが同格ということで、解釈できるけど

Why can’t you be like other girls your age?
はやはり無理だね。

いい加減あきらめろ、へなちょこ野郎。
0190😉三年英太郎🌈 ◆3CZBjOt3.Y (ワッチョイW 9a86-fgND)
垢版 |
2023/01/06(金) 23:56:30.15ID:DM6qQSDM0
比喩という意味論的解釈に、同格云々の機能論的視点を持ち込んで反論できた気になってるアホですか?🤔

残念ながら、アウアウアーもしばらくNGにさせていただきます🙇🏻‍♂
0191😉三年英太郎🌈 ◆3CZBjOt3.Y (ワッチョイW 9386-iXfb)
垢版 |
2023/01/07(土) 00:05:10.68ID:sh8U3J2W0
実際の英語の歴史の中で、もともと of 対格 という形で存在していたなら、そういう通時的理解もありえよう

あの副詞的対格と呼ばれてるものは、ホントに昔は対格だったのである

が、こっちはあんまりそんな証拠も見つかってないようなので🙄
0192名無しさん@英語勉強中 (アウアウウー Sa85-8xCC)
垢版 |
2023/01/07(土) 00:06:36.40ID:2J3KVpdBa
other girls your age
これなら、名詞の直後に素の名詞のままおかれるという標識で表されている格
当たり前のようだが、your ageはother girlsの前ではないし、他の単語を挟んだ後ろでもない
your age'sでもない

または、ofが省略されていて以下略

これは格でなく名詞の後ろに名詞があるくらいで説明した方が簡潔なだけで、格でも説明できること

この格(又は簡潔に名詞の後ろに名詞)の意味はまた別問題として
0193名無しさん@英語勉強中 (テテンテンテン MM4b-tb1I)
垢版 |
2023/01/07(土) 00:07:15.89ID:eNFIi7KdM
>>190
何言ってん基地外。
your ageがgirlsを限定修飾しているケースなんだから
your ageがメトニミーという説明では
解釈が無理だと言ってんだよ、低能。

Why can’t you be like other girls your age?
のyour ageが何のメトニミーか言ってみろ、馬鹿。
0194名無しさん@英語勉強中 (テテンテンテン MM4b-tb1I)
垢版 |
2023/01/07(土) 00:09:34.13ID:eNFIi7KdM
>>192
その意見に賛同しますよ。

で、この解釈にメトニミーがどうとかっていう話は必要だと思います?
0195名無しさん@英語勉強中 (テテンテンテン MM4b-tb1I)
垢版 |
2023/01/07(土) 00:16:05.78ID:eNFIi7KdM
>>192
質問2
other girls your age
のyour age と
I was your age
のyour age とは、機能的に同様にみなせませんか?
0197😉三年英太郎🌈 ◆3CZBjOt3.Y (ワッチョイW 9386-iXfb)
垢版 |
2023/01/07(土) 00:21:53.73ID:sh8U3J2W0
>>172
無学なソフィスト先生に教えてあげると、そのアースはアースそのものじゃなく、アースの形を言ってるのです

ソフィスト先生には申し訳ないけど、あなたが今日ウィキペを読んで理解出来てる範囲より、問題はもう少し先にあるので、どうか自重してください🙇🏻‍♂
0198名無しさん@英語勉強中 (アウアウウー Sa85-8xCC)
垢版 |
2023/01/07(土) 00:25:45.03ID:ro8/vvjsa
>>194
俺の言いたいこととズレてる
俺は格で分類できるし、無意識レベルでしてるという考え
無意識でやってる名詞の位置関係とかを、わざわざ格という理論で考え直すのは意味があると言いたい
んで標識による格で考えたあとに、その格の意味を考えるのはまた別のこと

158は名詞の働きをメトニミーと説明できるからと、格の説明に否定的だが(この説明が正しいと俺は書いてない)
標識による格の分類と名詞の働きを考えるのは別に相反するわけじゃないと言いたい
0199名無しさん@英語勉強中 (テテンテンテン MM4b-tb1I)
垢版 |
2023/01/07(土) 00:30:48.35ID:eNFIi7KdM
>>197
へー、そうすると
I bought a car the size of a killer whale.
は、
シャチくらいの車の重さを買った
ってことなんだな 苦笑
0200名無しさん@英語勉強中 (テテンテンテン MM4b-tb1I)
垢版 |
2023/01/07(土) 00:35:00.02ID:eNFIi7KdM
>>198
「その格の意味を考えるのはまた別のこと」
とありますが

その意味は考えていらっしゃるのですよね?
形容詞的な意味であるとするなら
メトニミーなんてことを考える必要なんてないですよね?

もっとずばり聞きます。
Why can’t you be like other girls your age?
のyour ageが何のメトニミーと考えれば
この文は解釈できますか?
0201名無しさん@英語勉強中 (テテンテンテン MM4b-tb1I)
垢版 |
2023/01/07(土) 00:44:03.76ID:eNFIi7KdM
when I am your age
のIがmy ageのメトニミーだとして解釈するのだから

Why can’t you be like other girls your age?
の場合もyour ageがメトニミーではないのだろう。

どれが、なんのメトニミーなのだろうか? 苦笑
0202名無しさん@英語勉強中 (テテンテンテン MM4b-tb1I)
垢版 |
2023/01/07(土) 00:47:25.16ID:eNFIi7KdM
>>198
>>158は名詞の働きをメトニミーと説明できるからと

それについて疑問点はないでしょうか?
Why can’t you be like other girls your age?
↑これ、メトニミーで説明できると思いますか?
0203名無しさん@英語勉強中 (テテンテンテン MM4b-tb1I)
垢版 |
2023/01/07(土) 01:03:58.85ID:eNFIi7KdM
>>169
繰り返しますが
対立するとは思わないけど
格によって形容詞的な意味合いが生じるなら
メトニミーなんていうことは考える必要がないと
私は言っている。

アホ太郎は、格によって形容詞的な意味合いが生じるなんてことはない
と判断してるから
メトニミーでなんとか説明しようとしてるのだから。
失敗してるけど。
0204名無しさん@英語勉強中 (テテンテンテン MM4b-tb1I)
垢版 |
2023/01/07(土) 01:18:55.33ID:eNFIi7KdM
はっきり言って「格」で考える問題でもないと思うんだよな。
サイズ、重さ、形、色、年齢等を表す名詞に限った語法の話と捉えればいいと思う。

根拠1
限られた名詞にのみ用いられる用法だから。

根拠2
主格補語、目的格補語として用いられる場合も
名詞の後置修飾として用いられる場合も
同様な機能だとみなせるから。
0205😉三年英太郎🌈 ◆3CZBjOt3.Y (ワッチョイW 9386-iXfb)
垢版 |
2023/01/07(土) 01:38:06.19ID:sh8U3J2W0
>>191
>実際の英語の歴史の中で、もともと of 対格 という形で存在していたなら

伝統文法が言う「記述の対格」なる用法が、格変化があった時代から (of) 対格 という形で存在していたなら、という意味です。念のため

その昔は、前置詞が格支配をしていたのは当然です。念のため🙇🏻‍♂
0206😉三年英太郎🌈 ◆3CZBjOt3.Y (ワッチョイW 9386-iXfb)
垢版 |
2023/01/07(土) 01:48:41.03ID:sh8U3J2W0
伝わってないかも…🤔 丁寧に書こう

When I was [your age] ...
her eyes, [a deep blue] ...

[...] 内が「記述の対格」と呼ばれるなら、格変化があったOEでは当然は対格に格変化させてたんですよね?🤔

そして現代英語においても、深層格において対格or 目的格なんですよね?🤔

そうでない限り、この名称はデタラメというわけです
0207名無しさん@英語勉強中 (テテンテンテン MM4b-tb1I)
垢版 |
2023/01/07(土) 07:57:28.91ID:M7SVOvNsM
記述の対格という名称がデタラメであろうが、なかろうが

まず確定している事実は
Why can’t you be like other girls your age?
をメトニミーで説明することは不可能

記述の対格という概念は不要、メトニミーで説明できるから
というアホ太郎の主張がデタラメであるという事実は覆せない。
0208名無しさん@英語勉強中 (ワッチョイ 1189-vxq+)
垢版 |
2023/01/07(土) 07:59:50.32ID:QdNTHptw0
ドイツ語の動詞の活用 過去時制に使う単語 preterite
なんだけど なんで past じゃないんだろっておもったら
過去を意味するドイツ語の単語にあわせてるんだろな
0210名無しさん@英語勉強中 (ワッチョイ 9379-EeTn)
垢版 |
2023/01/07(土) 10:21:20.31ID:qAEJ4Nr40
>>206
用語上、なんちゃらの対格とか与格とかの用語、古英語中英語で相当する格が明示された
時期がある必要はないんじゃないかな?
ググってみると19世紀に流行った省略語法って書いてものもあるし。
://iwate-u.repo.nii.ac.jp/?action=pages_view_main&active_action=repository_view_main_item_detail&item_id=12791&item_no=1&page_id=13&block_id=21
0214名無しさん@英語勉強中 (テテンテンテン MM4b-tb1I)
垢版 |
2023/01/07(土) 13:06:57.87ID:7zG2VFAoM
記述の対格という名称がデタラメであろうが、なかろうが

まず確定している事実は
Why can’t you be like other girls your age?
をメトニミーで説明することは不可能

記述の対格という概念は不要、メトニミーで説明できるから
というアホ太郎の主張がデタラメであるという事実は覆せない。
0216🐤三年英太郎🍓 ◆3CZBjOt3.Y (ワッチョイ 9386-K0Bq)
垢版 |
2023/01/07(土) 13:16:06.39ID:sh8U3J2W0
>>213を踏まえて整理しよう。

わたくしは、すでに英文法解説は売りはらったので、断片から想像するだけだが、江川せんせーは「記述の対格」という通称から二つの誤解をしてしまった。

a. When I was [your age] ...
b. Her eyes, [a deep blue] ...

1.せんせーは [...] 内が対格(現在の目的格)であると思ってしまった。

2.せんせーは、目的格という形に意味があると考えてしまった。
(だから「目的格 形容詞に相当する用法」というトンチンカンな分類をしたのである)
0217🐤三年英太郎🍓 ◆3CZBjOt3.Y (ワッチョイ 9386-K0Bq)
垢版 |
2023/01/07(土) 13:20:36.52ID:sh8U3J2W0
>>210
> When l was your age, I didn't do such a thing.において, I が your
age と同一であるとするのは非論理的である。 Sweet はこれは When I was
of your ageの ofが脱落したものであるとみる。This is sailor fashion.
も This is done sailor fashion.という表現から生れたものである

文法的に of を入れて補ったのは Sweet さんの想像であって、おそらく歴史的事実ではなかろう。
0218名無しさん@英語勉強中 (テテンテンテン MM4b-tb1I)
垢版 |
2023/01/07(土) 13:26:39.31ID:7zG2VFAoM
記述の対格という名称がデタラメであろうが、なかろうが

まず確定している事実は
Why can’t you be like other girls your age?
をメトニミーで説明することは不可能

記述の対格という概念は不要、メトニミーで説明できるから
というアホ太郎の主張がデタラメであるという事実は覆せない。
0219🐤三年英太郎🍓 ◆3CZBjOt3.Y (ワッチョイ 9386-K0Bq)
垢版 |
2023/01/07(土) 13:31:16.46ID:sh8U3J2W0
>>210
>言語表現は,誤解を防 ぐため、情報の確実な伝達をはかるために、重複的
であるといわれるが、一方、意味が通ずれば、できるだけ簡潔な表現が好ま
れる傾向がある。

要するに、伝わればいいじゃん!ということである。以下、日本語のメトニミーの例。

1.本を読む。(読んでいるのは、本に印刷された字である)
2.Youtubeを見る。(見ているのは、youtubeにアップロードされた動画である)
3.彼女の目は赤い。(赤いのは、彼女の白目の色である)

> メトニミー表現に関する近年の認知言語学的見方では、「言語表現のほとんどはメトニミー表現」だという認識が一般的になりつつある
李在鎬 『認知言語学への誘い』
0220名無しさん@英語勉強中 (テテンテンテン MM4b-tb1I)
垢版 |
2023/01/07(土) 13:50:28.75ID:7zG2VFAoM
>>219
早く
Why can’t you be like other girls your age?
をメトニミーで説明しろよ、馬鹿
0221名無しさん@英語勉強中 (テテンテンテン MM4b-tb1I)
垢版 |
2023/01/07(土) 14:00:17.69ID:7zG2VFAoM
I bought a car the size of a killer whale.
Why can’t you be like other girls your age?
こららの文をメトニミーで説明できるのか、どなたか彼に聞いてくれませんか?

私のことはアボーンしているそうなので。
どなたか宜しくお願いします。
楽天モバイルとAUは全部アボーンしているそうです。
0223名無しさん@英語勉強中 (ワッチョイ b308-/qOa)
垢版 |
2023/01/07(土) 17:59:02.69ID:4XR+gytI0
あなたがわたしに歌わせてくれる
みたいなのを
訳すとき普通に使役動詞でmake letで訳す以外にないでしょうか?
もうちょっと
宗教的に
賛美歌とかのように

「神(的な感謝すべき偉大な方)のおかげで」わたしは歌えてるんだ
みたいな意味をこめるときの
素敵な表現を教えてください
Queenのヒット曲(my best friend)だと
出だしが Oh you make me liveなんですが。
この場合は確かにletでない方がいいですよね。でも
聖なる神への捧げ物としての歌を歌わせてもらうなら?
どういう表現でしょう?
0226🐤三年英太郎🍓 ◆3CZBjOt3.Y (ワッチョイ 9386-K0Bq)
垢版 |
2023/01/07(土) 20:21:05.19ID:sh8U3J2W0
見直した

過去 past
過去時制 past tense
過去形 preterite

過去という時を、(助)動詞の変形によって表現するのが過去時制で、
過去時制のうち、1語の屈折によるもの(単純過去)が preterite
0227名無しさん@英語勉強中 (ワッチョイ d308-/qOa)
垢版 |
2023/01/07(土) 22:29:16.02ID:ittLMU370
>>224
ありがとうございます!
素敵な文ですね
ただ
「うたわせていただける」という謙遜の気持ちの分なので
他の人を喜ばせる声に恵まれたってのが
ほんの少しw エッヘンしてるかもってw

>>225

ありがとうございます

2番目の分の thatはなくてもいいんでしょうか?
入れることで関係代名詞としてどういう
意味というか用法になるんでしょうか?
0228エキセントリックなソフィスト ◆BWpafMA3P8me (ワッチョイ 7996-DWwd)
垢版 |
2023/01/07(土) 23:03:49.89ID:m7VJDUTV0
>>197
3年ちゃん、もうボロボロだね。(^O^)

いいか、私は、>>172で「The earth is the shape of an orange.のthe shapeは、メトニミーとは呼ばない」と言ってるわけ。それ理解できてる?まず、それを理解してね。
そして、>>172に反論するなら、「the shapeはメトニミーだ」と主張しなければ、論理的整合性がとれないんだよ。
「このアースの意味は・・・」では、反論にはならない。なぜなら、172では、「the shapeはメトニミーか否か」が問題の焦点なのだから。

それでも、まだ「The earth is the shape of an orange.のthe shapeは、メトニミーとは呼ばない」に反論するなら、the shapeがなんのメトニミー(比喩)なのか言ってみなよ。
それが言えないなら、>>172での私の結論になんの問題もないことを理解しようね。

いつも、私に論破されてばかりの3年ちゃんは、IQが自分が思ってるほど高くないことをもっと自覚したほうがいいよ。
私は、人の英作・和訳を見ただけで、その人のIQレベルがわかるほうだが、3年ちゃんのIQは私より高くない。(^-^)
3年ちゃんはメモリー(側頭葉)に知識がいっぱいあるけど、それを使うCPU(頭頂葉)がいまひとつなのだ。もったいない。
0229名無しさん@英語勉強中 (ワッチョイ b369-tb1I)
垢版 |
2023/01/08(日) 00:00:27.29ID:Obj7xa6F0
I'd like you all to do is think about another dish you can cook for Saturday evening.

I'd like you all to do is think about の部分ですがI'd like you all to do is (to)think about 〜

のようにthink の前に to が必要な気がするのですが、最初の文は文法的に間違いでしょうか?
0231エキセントリックなソフィスト ◆BWpafMA3P8me (ワッチョイ 7996-DWwd)
垢版 |
2023/01/08(日) 00:30:09.81ID:B4OT/7k00
>>229
その文は、頭にAllまたはWhatがついているはずです。
(正)All I'd like you all to do is think about another dish you can cook for Saturday evening.
   「私があなた方みんなにやってほしいことは、~することだけです」(直訳)

英語には、こんな決まりがあります。
『「All」や「What (こと)」で始まる主語のとき、SVCのCの不定詞は、toを省略できる』
0232名無しさん@英語勉強中 (テテンテンテン MM4b-tb1I)
垢版 |
2023/01/08(日) 00:43:16.68ID:XgjOP2mhM
>>230
omaemona
0233名無しさん@英語勉強中 (ワッチョイ b369-tb1I)
垢版 |
2023/01/08(日) 00:57:14.38ID:Obj7xa6F0
After the lease( ),customers have the option of purchasing the car or returning it to their local dealer.
1.expired
2.expires
答えはexpires 何ですが、expired でもいいような気がします
expires じゃなければいけない理由を教えてください
0234The OED Loves Me Not (ワッチョイ 2b89-K0Bq)
垢版 |
2023/01/08(日) 07:07:59.59ID:29+Qlza40
>>233
>>After the lease ( ), customers have the option of

後半の "customers have the option of..." が現在形だろ?
ということは、「lease が expire したときには、恒常的に
(原則として、普段は、一般的に) customers は
これこれこういう option を持つことになるのだ、と言っている。

つまりここでの単純現在形の have は、一般的、普遍的、
恒常的なことを述べるための形なんだ。

だからこそ、after のあとの expire も expires というふうに
「一般的、普遍的、恒常的な様子」を表す形である lease
が似合うわけだ。

expires の代わりに、has expired でもいいだろうけど、
少なくとも英語ではそこまでこだわらなくても、
after the lease expires で十分だということになっている。
0236名無しさん@英語勉強中 (ワッチョイW c1fb-S0bH)
垢版 |
2023/01/08(日) 07:11:22.18ID:E2eboVmb0
伸び代で思い出した。
原野が面積の多くを占める地域についてのことだ。
原野はこわい。

aさん:これからは観光で儲けよう
と役所の仕事を引き受けてたaさん

bさん:ほとんど観光する場所もないし、観光客も
入ってませんが。

aさん:だからこそ、伸び代はでかい。

aさん有名人だけど、ほんと適当で、産官学共同のベンチャーだ
と全国で講演して本まで出して、商品売れずベンチャーは
潰れてaさんは大学を辞めて、都会の大学の先生になって、
再びその地域で地域活性化の委員会座長とかやってます
0237The OED Loves Me Not (ワッチョイ 2b89-K0Bq)
垢版 |
2023/01/08(日) 07:14:42.57ID:29+Qlza40
>>234-235
というような説明をしてみたけど、こんな抽象的なことを言っても、
何を言われているのかわかりにくいだろうな。僕の説明の仕方も
悪いだろうけど、こんなことを明快に、たいていの人にすぐに
わかるように解説できるような人なんて、そんなに多くはいないと思う。

こういうことをクリアするために僕がどうしたのかというと、
教師や参考書や文法書が言っているこんな抽象的な解説は無視して、
ただひたすらたくさんの英文を読んでいったんだ。そして何十年も経った頃、
なんとなく「ああ、こういうことだったみたいだな」と感じるようになったんだ。

抽象的な思考力の発達した人なら、あるいは言語学一般に最初から
興味のある人なら、初心者のころ(つまりたとえば英検1級 TOEIC 990
に到達する以前)でも、こんなことを抽象的な言葉で議論して、
きちんと理解できるのかもしれんけどね。
0240The OED Loves Me Not (ワッチョイ 2b89-K0Bq)
垢版 |
2023/01/08(日) 07:35:16.49ID:29+Qlza40
それはともかくとして、

>>233
>>After the lease ( ), customers have the option of

このように、
After the lease expires, customers have the option of...
みたいな言い回しは、巷に溢れかえっている。そして
その expires が expired になっている用例を見ることは
ほとんどないか、あるいはまったくない。

それなら、「こういうときには expires になるんだな」と
素直にまずは覚える。そして 30 年jほどたくさんの
用例に出会い続けたあとに、改めて考えなおせばいいんだ。

そもそも文法って、そのようにして体得していくもんだろ?
文法家は最初から法則を身に付けたんじゃなくて、
彼らは彼らで、無規則に見える膨大な用例を何万通り、
何十万通りも見たり聞いたりしてきたわけだ。そのうえで
「どうやらこういう規則があるらしい」と思いつく。それを
文法書として発表するだけのことだ。
0241名無しさん@英語勉強中 (ワッチョイ 2b84-GrLB)
垢版 |
2023/01/08(日) 07:49:10.89ID:MMPmDtGe0
>>233
expireは辞書で調べると自動詞用法が多数を占めてます。
他動詞用法は「〈息〉を〔…から〕吐き出す〔from〕.」(ジーニアス英和辞典)
しか見当たりません。ですので過去分詞(受け身)による後置修飾ではないですね。

ではexpiredを過去形と見ることはできるでしょうか?仮定法になってしまう恐れもでてきます。
expiredといわれた瞬間、聞いてる人は今いる時点から離れた時空を感じてしまうことでしょう。
・時系列としての過去と感じた場合→確定した過去を感じ、聞いてる人がリース契約してると考えるのが
普通だが主節は一般論を述べておりチグハグな感じを受ける。
・仮定法すなわち本来の時系列からずれた世界を感じた場合→主節はwouldを使うべき。

一つ気になるのがexpiredを完了の形容詞と考えた場合にダメな理由が思いつきませんでした。
あとafterを副詞として考えるアプローチができません。

以上、ご批判は甘受しますのでよろしくお願いします。
0243エキセントリックなソフィスト ◆BWpafMA3P8me (ワッチョイ 7996-DWwd)
垢版 |
2023/01/08(日) 08:10:20.55ID:B4OT/7k00
追記。3年ちゃんは、>>197で、The earth is the shape of an orange.のthe earthは、「地球」ではなく「地球の形」なのだと言っているが、そんな屁理屈通るなら、『I was your ageの「I」は、「私」ではなく「私の年齢」なのだ』という屁理屈も通る。3年ちゃんの反論って、根拠のない簡単に論破できる稚拙な屁理屈であることが多いよね。

結論
この「shape」はメトニミー(何かの代用語)として使われてるわけではない。同様に、your ageも、The earth is the shape of an orange.のケースと同様、メトニミーではない。
0246😉三年英太郎🌈 ◆3CZBjOt3.Y (ワッチョイW 9386-iXfb)
垢版 |
2023/01/08(日) 13:03:15.47ID:deDv6gLA0
>>243
>>184

ソフィスト先生は無理しなくてもいいよ

伸びしろない人の特徴

👉そもそも興味の幅が狭い
👉ちょこちょこ小ネタは追加・アップデートできるが、二十歳前後で身につけたパラダイムからシフト出来ない
0247名無しさん@英語勉強中 (テテンテンテン MM4b-tb1I)
垢版 |
2023/01/08(日) 15:26:20.33ID:rKxkZV8zM
>>243
>>『I was your ageの「I」は、「私」ではなく「私の年齢」なのだ』という屁理屈も通る。

アホ太郎は最初からそう主張していたようだ。

>>94
>of が省略されてるなら、my age は of の目的語だから目的格だろうけど、省略されてないからね
>これは I listen to Chopin. と同じでメトニミーなんだよ。江川は認知文法知らない、ゴリゴリ伝統グラマリアン

とだけ書いてあったので
my age の部分がofの省略ではなく、Chopinと同様なメトニミーだと
主張しているのだと思った。
つまり、my ageがa man of my age のような意味のメトニミーという主張だと思った。

しかしそうではなく
Iがmy ageのメトニミーという主張のようだ。
たしかに、記述の対格の用法と言われる語句が主格補語の位置にあるときには、その主張も論理的には破綻していない。

だがそれはインチキな説明である。
それは↓これらのように、名詞を後置修飾されている例を説明できないから。
I bought a car the size of a killer whale.
Why can’t you be like other girls your age?

これらはメトニミーで説明できないだろうと論破されて
アホ太郎はいつものようにアボーンして逃げ回っているわけだよ。

卑怯なホモ野郎w
0248名無しさん@英語勉強中 (テテンテンテン MM4b-tb1I)
垢版 |
2023/01/08(日) 15:34:33.47ID:rKxkZV8zM
Sweetはofが欠落したものだと解した。
Onions(5文型の元になる理論を提唱した人)が「記述の対格」と命名した。

歴史的に、本当にofが欠落して生じた用法かはわからない。
しかし、意味的にはそのような意味だとネイティブの学者は主張しているわけ。
実際、主格補語や目的格補語だけでなく、後置修飾として使われている場合も
きれいにそれで理解できる。

これを、メトニミーだ!なんて言ってるアホ太郎はやはりオツムガ足りないのだろう。
0249名無しさん@英語勉強中 (テテンテンテン MM4b-tb1I)
垢版 |
2023/01/08(日) 15:46:52.94ID:rKxkZV8zM
後置修飾のケースでは説明できないだろうと反論すると
Her eyes, a deep blue, were quite impressive.
についてだけ、言及した。
このケースでは、記述の対格である a deep blue が非制限的用法であるため、
The color of her eyes, a deep blue, were quite impressive.
と、同格の名詞句として、たまたま説明できたに過ぎない。
(このような場合でも、述部が例えば、"were getting weaker" だとしたら、勝手に
Her eyesをThe color of her eyesの意味だと強弁すると、明らかにおかしなことになる。)
0250名無しさん@英語勉強中 (テテンテンテン MM4b-tb1I)
垢版 |
2023/01/08(日) 15:53:18.22ID:rKxkZV8zM
ofの消失だから「対格」(→(目的格))という説明は
確かに歴史的な裏付けがないのかもしれない。
しかし、意味の理解としてはそれで十分である。

少なくも、一部の用例でしか説明できない、出来の悪いメトニミー説なんかを採用するより
はるかにマシであることは確実である。
0251名無しさん@英語勉強中 (テテンテンテン MM4b-tb1I)
垢版 |
2023/01/08(日) 15:55:05.96ID:rKxkZV8zM
>>246
お前のようにドアホな糞バグ理論にアップデートするより遥かにマシだろ 苦笑
0252名無しさん@英語勉強中 (テテンテンテン MM4b-tb1I)
垢版 |
2023/01/08(日) 15:58:12.08ID:rKxkZV8zM
だからソフィスト先生。
この話題では、記述の対格が主格の補語として使われいる文でやり合っても
論破は難しいのです。
I bought a car the size of a killer whale.
Why can’t you be like other girls your age?
のような後置修飾のケースで戦いましょう。

ここは、共闘しませんか?
0254名無しさん@英語勉強中 (テテンテンテン MM4b-tb1I)
垢版 |
2023/01/08(日) 21:46:12.55ID:bk2HHPDaM
>>253
メトニミーがどうたらというアホな説明はきれいさっぱり忘れた方が吉
0255名無しさん@英語勉強中 (テテンテンテン MM4b-tb1I)
垢版 |
2023/01/08(日) 22:01:14.33ID:GVtX5X6jM
>>180
>>177 の質問

「これはseeの場合は目的格補語に(代)名詞をとることはないという意味でしょうか?」
の回答になっていませんでしたね。

その通りです。seeの場合はありません。
0257名無しさん@英語勉強中 (テテンテンテン MM4b-tb1I)
垢版 |
2023/01/08(日) 23:18:06.79ID:pArBiH8qM
>>256
誰が見えも敗者ではあるのだけど
あいつ自身は俺のコメントをアボーンしてるから
勝った気でいやがるんだよなぁw
未だにメトニミー説が正しいと言ってるみたいだし。

まぁいいか。
0258エキセントリックなソフィスト ◆BWpafMA3P8me (ワッチョイ 7996-DWwd)
垢版 |
2023/01/08(日) 23:23:26.03ID:B4OT/7k00
>>246

A This building is of the same size as that one.
B This building is the same size as that one.

BはAと同じものである。補語部は主語の性質を表す働きをしている。この種の文で、メトニミーを持ち出す必要はないと思う。
0259エキセントリックなソフィスト ◆BWpafMA3P8me (ワッチョイ 7996-DWwd)
垢版 |
2023/01/08(日) 23:28:51.94ID:B4OT/7k00
ちょっと追撃してみた (^-^)
0260名無しさん@英語勉強中 (テテンテンテン MM4b-tb1I)
垢版 |
2023/01/08(日) 23:29:37.74ID:pArBiH8qM
えええ、これ以上追撃する必要もないでしょうって言ってたのに
続けてるし
しかもまた主格補語じゃんかぁw

またアホ太郎は
This buildingは、The size of this building のことと解釈すれば意味が通じる
って言うだけで
前の繰り返しになるだけなんだが・・・泣
0261◆tnTl1nZEu77W (ササクッテロ Sp4d-GG+p)
垢版 |
2023/01/08(日) 23:31:26.01ID:TKYmeN4Gp
test
0262名無しさん@英語勉強中 (テテンテンテン MM4b-tb1I)
垢版 |
2023/01/08(日) 23:33:11.62ID:pArBiH8qM
主格補語の場合はメトニミーで強引に解釈されてしまうから
完膚なきまでの論破ができないのだよ。

I bought a car the size of a killer whale.
Why can’t you be like other girls your age?

↑これで戦ったくれよおーーーーーーーーーー
メトニミーでは全く説明できないのだからさぁ
面白いじゃんか。

共闘してもらいないのなら、まぁいいよ。
明日、どっかの喫茶店のWi-fiで書き込むわ。
0263エキセントリックなソフィスト ◆BWpafMA3P8me (ワッチョイ 7996-DWwd)
垢版 |
2023/01/08(日) 23:47:44.34ID:B4OT/7k00
>>260 ここからが本題 (^-^)

>>246
では、3年ちゃん、次の例を説明してみて。

「a building the same size as that one」(あれと同じ大きさの建物)

なんで、of がなくても正しいの?

修飾できるということは、the same 以下が形容詞句として働いているからだよ。

This building is the same size as that one. も、the same size as that one.は、形容詞句として働いているのだ。
だから、メトニミーを持ち出して、「主語は比喩である」とする必要などまったくないのだ。わかったかな?
0265名無しさん@英語勉強中 (テテンテンテン MM4b-tb1I)
垢版 |
2023/01/08(日) 23:59:33.70ID:pArBiH8qM
>>263
おお、サンクス。

形容詞的な働きがあるとするなら
メトニミーなんて持ち出す必要性は皆無なんだよな。

こんな簡単なことが、アホ太郎にはわからないらしい 苦笑
0266エキセントリックなソフィスト ◆BWpafMA3P8me (ワッチョイ 7996-DWwd)
垢版 |
2023/01/09(月) 00:08:11.98ID:dJxLWaeH0
3年ちゃんへ

ジーニアスの「size」の項目に次の例が載っている。
『an area the size of Nagoya 名古屋ほどの大きさの地域 《◆名詞のあとに来たときも同様にofはしばしば省略される》』

この用法では、名詞を後ろから修飾できるという性質があるのだ。これは、ofが省略された言い方も「形容詞句」のように働いていることを意味している。

これは、The area is the same size of Nagoya.の「the same size of Nagoya」も、形容詞句のように働いている証拠である。
よって、主語がメトニミーである必要などまったくない。

以上のことから、3年ちゃんの>>94の解釈(メトニミー説)は、間違いである。
0267名無しさん@英語勉強中 (ワッチョイ 91fe-9O0i)
垢版 |
2023/01/09(月) 00:49:06.41ID:WwL22Vy90
2語以上の分詞句も形容詞句も後置修飾になる場合は「who/which is」が省略された縮約関係詞節(reduced relative clause)と取られるから、となるとこの位置に来る「形容詞句」は実質的には主語の叙述補語(predicative)だな。

いずれにしろ後置修飾(postpositive)は後置修飾で、普通の形容詞の前置修飾=限定用法(attributive)とは別物。
0268🐤三年英太郎🍓 ◆3CZBjOt3.Y (ワッチョイ 9386-K0Bq)
垢版 |
2023/01/09(月) 00:54:07.21ID:/zS8A8Nw0
>>258
わたし最近、この人の頭の悪さにいらつくこと多いのよね・・・
ちょっと口が悪くなったときは、そーゆー時なんでご寛恕ください!

> A This building is of the same size as that one.
> B This building is the same size as that one.

そもそもBの形の方が普通である。

be of the same size as 5件
be the same size as 159件

be of the same color as 3件
be the same color as 100件 (COCA調べ)

むしろ、of 入りの方が論理的正しさを重視した「人工的な」言い方かもしれない。
0269🐤三年英太郎🍓 ◆3CZBjOt3.Y (ワッチョイ 9386-K0Bq)
垢版 |
2023/01/09(月) 00:57:30.33ID:/zS8A8Nw0
A is B だが、正確にはBがAの性質を表していない。じゃあ、この非論理性はどーゆーわけなん?って昔の学者が討論して、一部の人が「ofが省略されたんじゃね?」とか、「なんちゃらの対格だ!」と言い出したわけ。
んで、通時的に言えば、一部は確かに前置詞の脱落もあるし、他に発音の類似とか、いくつ理由があるとのこと。

が、共時的にはその非論理性を許しているのは何か、といえばメトニミーだろう。接点があるものに代替しても通じる。通じるから、そういう言い方が許容される。

おバカさんは、ひたすらBだけ着目してあーだこーだ形容詞だ(笑)と言ってるけど、視野が狭いのよ~

これはAとBの関係の問題なのです。ついてこれないなら、別についてこないでいいです。迷惑だからね
0272🐤三年英太郎🍓 ◆3CZBjOt3.Y (ワッチョイ 9386-K0Bq)
垢版 |
2023/01/09(月) 01:20:19.41ID:/zS8A8Nw0
権威が大好きなソフィスト先生に教えてやると、この非論理性はなにもおれだけが言っていることではない。

> (1)の名詞の用法(※)は、論理的に不都合である。つまり、when I was your age において、I と your age は明らかに同一のものではないからである。
安井稔 編 『コンサイス英文法辞典』 p. 13

(※)様々な「記述の対格」を(1)で挙げている。
0273エキセントリックなソフィスト ◆BWpafMA3P8me (ワッチョイ 7996-DWwd)
垢版 |
2023/01/09(月) 01:31:40.60ID:dJxLWaeH0
>>266
頭が悪いのは、3年ちゃんだよ。人格攻撃じゃなく、論理的に論破してみなさい。論理的な反論もできず、人格攻撃する人間は、前頭葉が未熟なのだ。

『 "is of the same size as" 』 でフレーズ検索すると2,630,000 件ヒットする。ほんとにほとんど誰も使ってないように思ってるとしたら、相当の情弱だよ。

情弱の反論はいいから、はやく、>>266に反論してみてよ。書いてあることは、すごく単純なことだぞ。

・the same sizeは形容詞句になれる。これはyour ageも同様だ。
・「形容詞句」なら、被修飾語(または主語)と同じものである必要はない。
よって、被修飾語はメトニミーである必要はない。

これのどこが完全に間違ってるんだよ? その論拠があるなら、出してみてよ。出せないなら、3年ちゃんのほうが頭が悪いことを認めるべきだ。
0274名無しさん@英語勉強中 (テテンテンテン MM4b-tb1I)
垢版 |
2023/01/09(月) 01:38:22.69ID:9likDtSdM
主格補語の例を出すと、こうやって逃げるんだよ、この馬鹿は。

後置修飾の例をメトニミーで説明できないことはスルー。
0275エキセントリックなソフィスト ◆BWpafMA3P8me (ワッチョイ 7996-DWwd)
垢版 |
2023/01/09(月) 01:39:15.17ID:dJxLWaeH0
>>28
『「私」が「あなたの年齢」というプロパティを持っていたとき』で解釈できるので、問題ない。

>>272
それは、「an area the size of Nagoya」も論理的に不都合で、同じこと。そんなことは誰でもわかること。
その不都合をどう考えるかが重要。
0276名無しさん@英語勉強中 (テテンテンテン MM4b-tb1I)
垢版 |
2023/01/09(月) 01:40:04.97ID:9likDtSdM
>>271
「私」が「あなたと同じ歳」という属性があるってことだろ
アホ。
0277名無しさん@英語勉強中 (テテンテンテン MM4b-tb1I)
垢版 |
2023/01/09(月) 01:41:10.27ID:9likDtSdM
あ、かぶったけど
ソフィスト氏はアンカー間違えてね?
0278エキセントリックなソフィスト ◆BWpafMA3P8me (ワッチョイ 7996-DWwd)
垢版 |
2023/01/09(月) 01:44:41.32ID:dJxLWaeH0
>>277 あ、ほんとだ (^-^;

>>271
『「私」が「あなたの年齢」というプロパティを持っていたとき』で解釈できるので、問題ない。
0279名無しさん@英語勉強中 (テテンテンテン MM4b-tb1I)
垢版 |
2023/01/09(月) 01:44:55.48ID:9likDtSdM
>>267
その話、ここで出す必要性あんの?
勿論attributiveではなくpostpositiveだけど、で?
0280😉三年英太郎🌈 ◆3CZBjOt3.Y (ワッチョイW 9386-iXfb)
垢版 |
2023/01/09(月) 01:46:54.16ID:/zS8A8Nw0
>『「私」が「あなたの年齢」というプロパティを持っていたとき』で解釈できる

それがメトニミーだと言ってるのにね

もう二十歳から30~40年たつでしょう?そろそろ新しいこと学んだら🤣
0282名無しさん@英語勉強中 (テテンテンテン MM4b-tb1I)
垢版 |
2023/01/09(月) 01:49:10.55ID:9likDtSdM
>>272
そのまま、つまり
your ageの前にofを補って考えたり
どっかのアホのようにメトニミーを持ち出したり
しなければ
非論理的であることなんて
誰も否定してねーだろ。
そんなのは前提で話してんだろ。

頭わいてんのか?ジジイ
0283😉三年英太郎🌈 ◆3CZBjOt3.Y (ワッチョイW 9386-iXfb)
垢版 |
2023/01/09(月) 01:51:41.93ID:/zS8A8Nw0
>・the same sizeは形容詞句になれる。

これも数ヶ月前、副詞句の話で散々言ったじゃん。主要部に形容詞がないので形容詞句ではございません。何にも新しいものを吸収する気がないのね🤷🏻‍♂
0284S&G (ワッチョイ d33f-3BFc)
垢版 |
2023/01/09(月) 01:53:09.28ID:E807MOes0
あんまりイジメちゃかわいそうだよ
ふつうに考えればもうだいたいわかったからいいじゃん
0286名無しさん@英語勉強中 (テテンテンテン MM4b-tb1I)
垢版 |
2023/01/09(月) 01:54:34.54ID:9likDtSdM
I bought a car the size of a killer whale.
Why can’t you be like other girls your age?
結局、これらの例をメトニミーで説明できない時点で
アホ太郎のメトニミー説は破綻している!

↑これを徹底的に突いていくのが、勝ち筋なんだよなぁ。
頼むよ、ソフィスト氏。
0287名無しさん@英語勉強中 (テテンテンテン MM4b-tb1I)
垢版 |
2023/01/09(月) 01:57:39.06ID:9likDtSdM
>>283
まさに揚げ足とりで逃走。

学校英語では「形容詞句」だと教えられたりもするだろ。
要は後置修飾しているということを言ってんだろーが。
0288名無しさん@英語勉強中 (テテンテンテン MM4b-tb1I)
垢版 |
2023/01/09(月) 01:58:30.49ID:9likDtSdM
>>285
まさにお前自身が嚙み締めるべき言葉だぞ、ホモジジイ。
0289S&G (ワッチョイ d33f-3BFc)
垢版 |
2023/01/09(月) 02:03:24.61ID:E807MOes0
不毛すぎるなあ
0290名無しさん@英語勉強中 (テテンテンテン MM4b-tb1I)
垢版 |
2023/01/09(月) 02:03:40.24ID:9likDtSdM
最終結論

記述の対格という名称がデタラメであろうが、なかろうが

まず確定している事実は
I bought a car the size of a killer whale.
Why can’t you be like other girls your age?
をメトニミーで説明することは不可能

記述の対格という概念は不要、メトニミーで説明できるから
というアホ太郎の主張がデタラメであるという事実は覆せない。
0291名無しさん@英語勉強中 (テテンテンテン MM4b-tb1I)
垢版 |
2023/01/09(月) 02:07:20.91ID:9likDtSdM
>>284
どっちがイジメてるって言ってんの?
ふつに考えてどっちが正しいわけ?
0292名無しさん@英語勉強中 (テテンテンテン MM4b-tb1I)
垢版 |
2023/01/09(月) 02:10:27.41ID:9likDtSdM
アホ太郎、元々アホだけど
なんだか急激に尋常じゃないアホになってね?

痴呆?
0293S&G (ワッチョイ d33f-3BFc)
垢版 |
2023/01/09(月) 02:11:33.47ID:E807MOes0
metonymyとか言ってる時点でダメに決まってるだろが
0294エキセントリックなソフィスト ◆BWpafMA3P8me (ワッチョイ 7996-DWwd)
垢版 |
2023/01/09(月) 02:15:07.70ID:dJxLWaeH0
>>280
メトニミーの意味もわからないの?
>『「私」が「あなたの年齢」というプロパティを持っていたとき』で解釈できるなら、それがメトニミーだ

じゃあ、The building is of the same size.の「of the same size」も「同じサイズというプロパティを持っている」ので、メトニミーなのかよ?言ってることがメチャクチャ。

>>281
"is of the same size as" 』 でフレーズ検索すると2,630,000 件ヒットする。 ←3年ちゃんの「ほとんど誰も使ってない」のウソを暴く結果

>>283
は?日本語の形容詞句は、形容詞が無くても言えるんだよ。中学校からやり直したらいかが(^O^)
英語において、形容詞句は adjectival phrase または adjective phrase と呼ばれるが、前者には「形容詞のような句」が含まれる傾向がある。
ソース:https://ja.wikipedia.org/wiki/%E5%BD%A2%E5%AE%B9%E8%A9%9E%E5%8F%A5
英語では、「adjectival phrase」と呼ばれている。

ほんと、3年ちゃんの理解がここまでひどいとは思わなかったよ
0295エキセントリックなソフィスト ◆BWpafMA3P8me (ワッチョイ 7996-DWwd)
垢版 |
2023/01/09(月) 02:19:04.29ID:dJxLWaeH0
>>285 再放送

頭が悪いのは、3年ちゃんだよ。人格攻撃じゃなく、論理的に論破してみなさい。論理的な反論もできず、人格攻撃する人間は、前頭葉が未熟なのだ。

『 "is of the same size as" 』 でフレーズ検索すると2,630,000 件ヒットする。ほとんど誰も使ってないという3年ちゃんの意見はウソ。

情弱の反論はいいから、はやく、>>266に反論してみてよ。書いてあることは、すごく単純なことだぞ。

・the same sizeは形容詞句になれる。これはyour ageも同様だ。
・「形容詞句」なら、被修飾語(または主語)と同じものである必要はない。
よって、被修飾語はメトニミーである必要はない。

これのどこが完全に間違ってるんだよ? その論拠があるなら、出してみてよ。出せないなら、3年ちゃんのほうが頭が悪いことを認めるべきだ。
0296名無しさん@英語勉強中 (テテンテンテン MM4b-tb1I)
垢版 |
2023/01/09(月) 02:21:17.55ID:a23AlIzDM
>>293
まぁ普通に考えてそうだよね。
でも、イジメちゃかわいそうとか言うからさ、
だって、metonymyとか言ってるアホの方も相手に罵声を浴びせてきてんじゃんw
0297S&G (ワッチョイ d33f-3BFc)
垢版 |
2023/01/09(月) 02:23:02.87ID:E807MOes0
5ちゃんで頭が良いとか悪いとか
バカバカしくないか?
0298名無しさん@英語勉強中 (テテンテンテン MM4b-tb1I)
垢版 |
2023/01/09(月) 02:29:49.84ID:a23AlIzDM
>>297
日頃からアホのくせに威張りくさってるアホ太郎には
このくらいの攻撃は必要なのさ

つか、もっと早い段階でアイツの方から
素直に間違いを認めればよかったんだよ
普通の人ならそうするよ
0299S&G (ワッチョイ d33f-3BFc)
垢版 |
2023/01/09(月) 02:32:07.77ID:E807MOes0
5ちゃんに普通の人がいないのはいまや常識だと思うがな
0300名無しさん@英語勉強中 (テテンテンテン MM4b-tb1I)
垢版 |
2023/01/09(月) 02:34:11.62ID:a23AlIzDM
>>299
じゃあ好きにやらせてくれよw
0301S&G (ワッチョイ d33f-3BFc)
垢版 |
2023/01/09(月) 02:38:57.58ID:E807MOes0
ふふ、そりゃ止めたってみんな好きにやるんだろうさ
ただなんてーのかな、もうちょっと穏やかなにやれんもんかと
ちょっとそんな風に思っただけでね
まあ、5ちゃんだものな
みんなジャンジャンバリバリやってくらはい
寝よっと
0302名無しさん@英語勉強中 (ワイエディ MM6b-tb1I)
垢版 |
2023/01/09(月) 04:55:38.23ID:tym5Q1Y6M
TOEIC公式問題集の text-message chain 中に
Have you confirmed with Luciano's about tonight?
とあります。

confirm は他動詞なのに目的語が省略されている理由を調べたのですが、見つけられませんでした。
口語では目的語の無い他動詞が許されるのでしょうか?
どなたかよろしくお願いいたします。
0303The OED Loves Me Not (ワッチョイ 2b89-K0Bq)
垢版 |
2023/01/09(月) 06:18:30.78ID:vOIvO0Yv0
>>302
>>Have you confirmed with Luciano's about tonight?

通常の辞書に、ちゃんと confirm を目的語なしで使う用例が載っているよ。

[no object]
We have a reservation for you for tomorrow night. Please ★confirm★.

(Merriam-Webster's Advanced Learner's Dictionary)
0305The OED Loves Me Not (ワッチョイ 2b89-K0Bq)
垢版 |
2023/01/09(月) 06:23:22.71ID:vOIvO0Yv0
>>302
confirm が「自動詞」としても使われると書いている辞書もある。

MacMillan English Dicationary for Advanced Learners, Second Edition

confirm
2
[Intransitive/transitive]
用例
You can make an appointment now, and then call nearer the time to ★confirm★.
0306The OED Loves Me Not (ワッチョイ 2b89-K0Bq)
垢版 |
2023/01/09(月) 06:30:05.22ID:vOIvO0Yv0
少しばかり古い辞書を見ると、確かに confirm を他動詞としてしか
紹介していないな。「アンカー木津美香」(2007)とか「おーレックス、第2版」(2013)
は少なくとも、他動詞としてしか紹介していない。

なるべく最新の辞書(つまりここ5年間に刊行されたもの)とか英英辞典も含めて、
何種類も参照しないと、こういうことをきちんと調べることはできないと思う。
0308The OED Loves Me Not (ワッチョイ 2b89-K0Bq)
垢版 |
2023/01/09(月) 06:50:38.82ID:vOIvO0Yv0
>>302
>>Have you confirmed with Luciano's about tonight?

(1) ジーニアスの最新版(第6版、2023)
confirm を自動詞としては紹介していない。

(2) ウィズダム英和、第4版、2019)
自動詞として紹介している。しかし用例はない。
「自動詞。(約束・日時などを)確認する、承認する)」と書いているだけ。

(3) Oxfored Advanced Learner's Dictionary, 10th Edition, 2020)
自動詞としては、紹介していない。
0313The OED Loves Me Not (ワッチョイ 2b89-K0Bq)
垢版 |
2023/01/09(月) 07:16:46.79ID:vOIvO0Yv0
>>302
Cambridge Advanced Learner's Dictionary, Fourth Edition, 2013

confirm
[Intransitive, transitive]
to make an arrangement or meeting certain, often by phone or writing
I've accepted the job over the phone, but I haven't ★confrimed★ in writing yet.

上のように、自動詞として紹介し、用例を載せている。
0314The OED Loves Me Not (ワッチョイ 2b89-K0Bq)
垢版 |
2023/01/09(月) 07:26:40.88ID:vOIvO0Yv0
>>302
結論として、僕の見直にある辞書のうち、英英辞典としては
3冊だけが、自動詞としての confirm の定義文と用例とを
1つずつ載せている。しかし英和辞典では、2022年の末に出た
「ジーニアス6」でさえそれを載せず、ウィズダム英和だけは
confirm を自動詞として載せてはいるが、その定義文だけであって
用例はまったくない。


confirm を自動詞として載せている辞書

(1) Merriam-Webster's Advanced
(2) MacMillan Advanced
(3) Cambridge Advanced
(4) ウィズダム英和、第4版
0315エキセントリックなソフィスト ◆BWpafMA3P8me (ワッチョイ 7996-DWwd)
垢版 |
2023/01/09(月) 12:20:23.15ID:dJxLWaeH0
日本語の「形容詞句」は、英語でいう「Adjectival Phrase」(形容詞を含まないが形容詞句のような働きをする句)の意味を含んでいる。
日本では、形容詞的に働く句も「形容詞句」と呼んでよいのだ。
証拠: https://www.eibunpou.net/01/chapter2/2_2.html
※Bring me the newspaper on the table.のon the table を形容詞句としている。教育現場でも、それが一般的だ。

また、下記のページでは、
https://study.com/skill/learn/how-to-identify-adjectival-phrases-explanation.html

The boy with mud on his shoes dirtied up his mother's clean floor. の「with mud on his shoes」を「Adjectival Phrase」、つまり、「形容詞(を含まないが形容詞のような働きをする)句」と説明している。

ジーニアスの「size」の項目に次の例が載っている。
『an area the size of Nagoya 名古屋ほどの大きさの地域 《◆名詞のあとに来たときも同様にofはしばしば省略される》』
この修飾関係はofの省略によってもたらされたものであるから、前置詞句による修飾から来ていると考えて差支えない。よって、the size of Nagoyaは日本では「形容詞句」と呼んでよいのである。
0316🐤三年英太郎🍓 ◆3CZBjOt3.Y (ワッチョイ 9386-K0Bq)
垢版 |
2023/01/09(月) 13:16:56.40ID:/zS8A8Nw0
>>295
枯木にいくら水をやってもムダなので、他の箇所にはもう付き合わないが

"is of the same size as" でググる
1ページ目 約 2,630,000 件 (0.49 秒)?
10ページ目 約 92 件中 10 ページ目 (0.54 秒)?

>最も的確な検索結果を表示するために、上の 92 件と似たページは除外されています。
>検索結果をすべて表示するには、ここから再検索してください。

で、再検索すると最後のページは
約 249 件中 25 ページ目 (1.35 秒)

googleの1ページ目のヒット数はまったく正確ではない。
最後に表示される数も、おそらくgoogleは取り込んでいるすべてのテキストからの検索ではない。

★googleは、指定された文字列があるページを検索するツールであって★文字列の数を測るツールではないのだよ。

二度とこんな雑なこそしないように。 恥 ず か し い か ら ね!
0317🐤三年英太郎🍓 ◆3CZBjOt3.Y (ワッチョイ 9386-K0Bq)
垢版 |
2023/01/09(月) 13:34:20.90ID:/zS8A8Nw0
be of the same size as 1件
be the same size as 25件

be of the same colour size as 1件
be the same colour size as 14件

BNC調べ

重要なのはヒットの絶対数じゃなくて、比率やねん。of なしの方が普通なのは揺るぎない。
Sweet さんは of の脱落ではないかと考えたそうが、おそらく歴史的事実ではない。

コンサイス英文法辞典に前置詞の脱落が起源だとして出てるのは、次の例である。

What trade art thou (on)?
(相当訳 What job are you?)
0318名無しさん@英語勉強中 (ワッチョイ 2b84-GrLB)
垢版 |
2023/01/09(月) 13:43:52.08ID:hrEAS/I80
>>241
これ(一部理解不能の文章でごめんなさいでした。)を書いた者ですが、どなたか答えて頂けたら。
当初の質問の(誤答の方の)文で、
After the lease expired, customers have the option of purchasing the car or returning it to their local dealer.
afterを前置詞として考えることは絶対に不可能でしょうか?
調べてみるとコンマがあると接続詞と取るのが普通のようですけれど。
そこでexpiredを完了の形容詞として
After expired lease, (以下同じ)
にしたらどうでしょうか?afterは前置詞として解釈できますか?コンマは邪魔?そもそもafterでなく他の前置詞、
例えばwithあたりになってしまいます?
0319名無しさん@英語勉強中 (テテンテンテン MM4b-tb1I)
垢版 |
2023/01/09(月) 14:00:56.34ID:luUgx0Y3M
>>316-317
攻撃のポイントがぶれると、
このように枝葉末節な論点で反論して、あたかも勝ち逃げたような印象操作をするのが
この糞野郎の特徴なのだ。

I bought a car the size of a killer whale.
Why can’t you be like other girls your age?
をメトニミーでは全く説明できないという本質的な欠点を
ひたすら突いていくのが勝ち筋なのに。
0320名無しさん@英語勉強中 (テテンテンテン MM4b-tb1I)
垢版 |
2023/01/09(月) 14:08:50.95ID:luUgx0Y3M
I bought a car the size of a killer whale.
Why can’t you be like other girls your age?
をメトニミーでは全く説明できないという事実を完全スルーして
あたかも勝ち逃げしたかのように言っている、この馬鹿の厚顔無恥さには
本当に恐れ入る。

老害の典型
0321名無しさん@英語勉強中 (テテンテンテン MM4b-tb1I)
垢版 |
2023/01/09(月) 14:43:37.14ID:luUgx0Y3M
>>318
After the expired lease
期間満了したリースの後

after が「〜の時間的後」だという意味の前置詞だとして
the expired lease は時間軸のある一点を表す目的語としては不向きではないですか?

>>234 のOED氏の説明が素晴らしいと思います。
まずはOED氏にお礼を言って、これをよく読み、
次に進んだらどうでしょうか?
0322名無しさん@英語勉強中 (テテンテンテン MM4b-tb1I)
垢版 |
2023/01/09(月) 14:50:40.70ID:luUgx0Y3M
あ、すまん
>>245 でお礼を言ってますね。

とにかく「満期を迎えたリース」って、出来事でもないし
「〜の時間的後」だという意味の前置詞の目的語としてふさわしくないと思います。
0323名無しさん@英語勉強中 (テテンテンテン MM4b-tb1I)
垢版 |
2023/01/09(月) 15:02:12.85ID:luUgx0Y3M
分詞構文にして
With the lease expired, にするなら・・・

独立分詞構文にwithがつくときは
普通「付帯状況」を表すので、やはりおかしいと思う。
(自動詞の過去分詞を使った分詞構文って、どのくらいあるのか知らないけど)
0324名無しさん@英語勉強中 (ワッチョイ 2b84-GrLB)
垢版 |
2023/01/09(月) 15:10:40.42ID:hrEAS/I80
>>321
レスありがとうございます。
そうですか、やはりafterは不向きと感じますか…。
誤答の理由を知りたかったのでこだわっているのですけどね。

>>234のOEDさんの説明が素晴らしいのは私でもわかります。
OEDさんの年齢から考えると50年以上英語と真剣に関わってきたその蓄積を
惜しげもなく私たちに晒してくれているのですから感謝至極です。
ただ次には進みますけど、もう少し誰かの回答を待ちたいですね。
誰も答えてくれないかもですが。w
0325名無しさん@英語勉強中 (テテンテンテン MM4b-tb1I)
垢版 |
2023/01/09(月) 15:11:28.02ID:luUgx0Y3M
特に付帯状況を表すwithがつく場合には、完了を表す自動詞の過去分詞との相性は悪い
ように思う。
0326名無しさん@英語勉強中 (ワッチョイ 2b84-GrLB)
垢版 |
2023/01/09(月) 15:11:48.81ID:hrEAS/I80
>>322
それ、私ではないです。
私はカッコつけて変な回答をしたおじさんです。
0327名無しさん@英語勉強中 (ワッチョイ 2b84-GrLB)
垢版 |
2023/01/09(月) 15:12:47.27ID:hrEAS/I80
>>325
なるほど、相性悪いですか。
例文当ってみます。
0328名無しさん@英語勉強中 (テテンテンテン MM4b-tb1I)
垢版 |
2023/01/09(月) 15:21:42.33ID:luUgx0Y3M
>>327
ごめん。
「リースが満了している状態で」と訳せば
意味が通るような気がしてきた。
他の人の回答を待って下さい。

まぁwithにした場合は、この問題の誤答理由とは直接関係ないとは思いますが。
0329名無しさん@英語勉強中 (ワッチョイ 2b84-GrLB)
垢版 |
2023/01/09(月) 15:26:28.37ID:hrEAS/I80
>>328
ありがとうございます。
0330名無しさん@英語勉強中 (ワイエディ MM6b-tb1I)
垢版 |
2023/01/09(月) 15:31:58.24ID:tym5Q1Y6M
>>303
ありがとうございます。
教えていただいたようなことがありうるのですね。
いくつかの辞書を当たる様にします。
0331エキセントリックなソフィスト ◆BWpafMA3P8me (ワッチョイ 7996-DWwd)
垢版 |
2023/01/09(月) 15:50:47.83ID:dJxLWaeH0
>>316 とうとう「検索方法」しか反論できなくなったのね・・・(^O^)  まあ、付き合ってあげよう。

=============================================================================
3年ちゃんの主張(詐欺的印象操作)
"is of the same size as" でググる
10ページ目 約 92 件中 10 ページ目 (0.54 秒)
「みてみて、of を使う文は、世界で29回しか使われてないでしょ!ね、誰も使ってないでしょ。
===============================================================================

私が「https://netspeak.org/」で調べた結果。
A "is the size of"
B "is of the size of"
 Aは37万件、Bは7900件。

確かに、ofは少数派だけど、ほぼ使われいないレベルではない。
そして、実はこれにはハンディがあるのだ。3年ちゃんは、視野が狭いから、そこに気づけてないけど、次レスでそれを説明しよう。
0332エキセントリックなソフィスト ◆BWpafMA3P8me (ワッチョイ 7996-DWwd)
垢版 |
2023/01/09(月) 15:51:15.26ID:dJxLWaeH0
A "is the size of"
B "is of the size of"
 Aは37万件、Bは7900件。実は、Aのほうは、名詞句と形容詞句の両方を含んだ結果が出てくるのに対し、Bのほうは形容詞句の用法しか出てこないのだ!

もっとわかりやすく説明してあげよう。
Aは、形容詞句の文とともに名詞句の文も含まれる。
This is the size of the sun. (これは太陽の大きさを表しているものです)
https://ejje.weblio.jp/content/and+so+this+is+the+size+of+the+sun.
「This = the size」 が成り立つので、「名詞句」である。これをofを使って言うことはできない。だから、Bが少なく出て当然である。3年ちゃんは視野が狭いので、公平な比較になってないことに気づいていないようだ。

しかしながら、ジーニアス英和辞典が言っているようにofを使わないほうが好まれるのは事実である。以下、引用。
『This building is (of) the same size as that one. <◆一般にofがないほうが好まれる>』
3年ちゃんの「ofなんてほぼ誰も使っていないことをほのめかす印象操作」は、有害なのである。せめて、「ofのないほうが多数派である」にとどめておくべきだった。
0334The OED Loves Me Not (ワッチョイ 2b89-K0Bq)
垢版 |
2023/01/09(月) 18:46:53.02ID:vOIvO0Yv0
>>318
他の言い回しを使ったらどうなるかという問題なんだったら、次のようなものも考えられる。

On the expiration of the lease, customers have the option of...

というのも、次のような用例がネットで見つかるから。

Section 5(5) states that if, ★on the expiration of the lease★, the tenant remains in the dwelling
with the express or tacit consent of the landlord, the parties are deemed, in the absence of
a further written lease, to have entered into a periodic lease, on the same terms and conditions
as the expired lease, except that at least one month’s written notice must be given of
the intention by either party to terminate the lease.
https://www.linkedin.com/pulse/what-does-one-month-mean-lease-agreement-sayed-iqbal-mohamed

さらに、after the lease expiration にして

After the lease expiration, customers have the option of...

と書くという手もある。

参考:
However, if there is no money paid ★after the lease expiration★, RPL §232-c does not set up
a month-to-month tenancy since the statute says that it is the payment of rent that sets up
the month-to-month tenancy, nothing else.
https://www.alblawfirm.com/articles/post-lease/
0335名無しさん@英語勉強中 (SGW 0H65-4NbX)
垢版 |
2023/01/10(火) 12:19:46.06ID:EDKCcA24H
Make him a friend って、
彼を友達にする(彼に自分の友達になってもらう)と、
彼に友達(第三者)を作ってあげる のどっちの意味でしょうか?
0337名無しさん@英語勉強中 (ワッチョイ 2b84-GrLB)
垢版 |
2023/01/10(火) 14:46:10.27ID:o1fQJ37O0
ありがとうございます。
>>333
普通に自動詞の完了の形容詞が使われてますね。afterでも良いですね。
リンク先読んでみました。意外と私(トーイック700点前後)でも読めますね。学術論文でなく、
一般人向けハウツー物なので出来るだけ分かり易い文章で書かれているのでしょうね。
臆病な性質なので今まで単語とか検索しても英語の原文のページは避けていましたが、自分が
読めるレベルの文章を探してみるのも良いですね。

>>334
expireからexpirationへの置き換えですね。expirationって単語帳レベルでは分かるけど、
実際には頭に浮かんで来ません。やっぱり文章で出会ってないからでしょうね。
やっぱり多読しないといかんですね。童話とか児童向けって飽きてきちゃうんですよね。
上のリンク先などのハウツーものなどはアメリカの生活などが頭に浮かんできて楽しそうです。
これからぼちぼち探していきます。

大変ためになりました。ありがとうございました。
0338The OED Loves Me Not (ワッチョイ 2b89-K0Bq)
垢版 |
2023/01/10(火) 14:56:02.08ID:QXj5KsVj0
>>335
So Dykstra’s on-field heroics eventually ★made him a friend★ of the Hollywood icon, Jack Nicholson.
https://www.essentiallysports.com/mlb-baseball-news-jack-nicholson-how-is-old-nails-lenny-dykstra-once-gushed-over-a-three-time-oscar-winner-while-recalling-celebrity-encounters/

この上の文の made him a friend の意味も、文脈を読み取れば誰でもわかる。
もしこの1文だけを読んでわからなければ、リンク先にある記事を冒頭から4行ほど読めば、すぐにわかる。

というわけで、made him a friend が使われている記事を片っ端から読んでいけば、
こんなところで質問なんかしなくても、簡単に自分で解答が導き出せる。
0341名無しさん@英語勉強中 (ワイエディ MM6b-tb1I)
垢版 |
2023/01/10(火) 17:11:18.23ID:y8kZ/Xx/M
DUO 3.0 の中に
My grandma strained her back when she bent down to hug my son.
とあるのですが、

おばあちゃんは曾孫を抱こうとしたという解釈で良いのでしょうか?
それとも話者が、自分の息子の視点で、自分の母を grandma と呼んで、おばあちゃんが孫を抱こうとしたと解釈すべきでしょうか?

なんとなく、曾孫を抱くおばあちゃんを想像できなくて、こんなことを考えてしまいました。
0342The OED Loves Me Not (ワッチョイ 2b89-K0Bq)
垢版 |
2023/01/10(火) 17:30:46.35ID:QXj5KsVj0
>>341
My grandma strained her back when she bent down to hug my son.

>>おばあちゃんは曾孫を抱こうとした

その解釈で間違いなくて、それ以外には考えられないと思う。

>>話者が、自分の息子の視点で、自分の母を grandma と呼んで、おばあちゃんが孫を抱こうとしたと解釈

そういう解釈はできないと思う。もしもそういう視点で英文を書くとしたら、次のように書かないといけないはずだと思う。

(1) When she bent down to hug my son, his grandma strained her back.

(2) Bending down to hug my son, his grandma strained her back.

(3) When my son was about to be hugged by his grandma, she strained her back.

要は、もしも自分の息子の視点でその女性を grandma と呼んでいるんだったら、
my son と書いたあとに、その my son を he (his) にして、その grandma だから
his grandma になってしまうのだ。なお、上の3つの文章のうち、 (3) は無理やりに書いた
ものであって、3つのうちのいずれかを選べと言われたら、やはり短くてわかりやすい
(2) を選びたいところだ。
0343名無しさん@英語勉強中 (ワイエディ MM6b-tb1I)
垢版 |
2023/01/10(火) 17:55:23.92ID:y8kZ/Xx/M
>>342
ありがとうございました。
duo は単語を詰め込んだ例文集なので、状況を想像すると少し違和感のある例文があるのは仕方ないのかもしれません。
早く duo を卒業して生の英文に触れるようにします。
0354名無しさん@英語勉強中 (ワッチョイW 79e3-4rlc)
垢版 |
2023/01/11(水) 06:13:15.69ID:xFvffIf90
英検準1級のための勉強をするにあたって
よく単語は例文と一緒に覚えた方がいいみたいに言われていますが
それだと1つの単語あたりに時間がかかりますよね
 
ある程度の短期間で単語を覚えるいい方法はなんだと思いますか?
人によるとは思いますが

趣味で総合的な英語力を上げていきたいと思っております
0355The OED Loves Me Not (ワッチョイ 2b89-K0Bq)
垢版 |
2023/01/11(水) 06:32:40.01ID:gF0lNB680
>>346
All four orbital launch pads fully loaded with rockets for the first time!
https://mobile.twitter.com/elonmusk/status/1612699840571637760

新聞や雑誌を普段から読んでいて、その文体に慣れていれば、
ここでの Elon Musk の言葉の文体がなぜこのようになっているかがわかる。

それがまだわからないのなら、そもそも Twitter なんかをいきなり英文で
読もうとしないで、まずは新聞雑誌で最小限度の基礎力を身に付けることが肝要。
そのレベルは英語検定で言えば、おそらくは英検2級、TOEIC 800 くらいだろう。
https://twitter.com/5chan_nel (5ch newer account)
0361名無しさん@英語勉強中 (ワッチョイ 2b89-AyIk)
垢版 |
2023/01/11(水) 12:51:09.93ID:7rSTkgpm0
>>360

「自由な時間よりももっと多くの友人を持っている」

「あまりに友人が多いので、ほとんど自分の自由な時間が持てないぐらいだ」
でしょう、たぶん。
ちょっとひねった表現。 (´·ω·`)
0362The OED Loves Me Not (ワッチョイ 2b89-K0Bq)
垢版 |
2023/01/11(水) 12:59:58.12ID:gF0lNB680
>>360
500 ページもあるこんな洋書を次から次へと読んでいくいつもの質問者さんだな。
あなたの力があれば、今回のこんな英文は、実に簡単に理解できるはずだ。

あなたの英語力は、ここにいるほとんどの回答者を大いに抜いている。
あなたが質問するのではなく、他の回答者があなたに質問してあなたに
教えを乞うべきなんだけどね。

あなたには回答者になってほしい。ここには、文法用語さえ振り回せば英語力があると
勘違いしている似非回答者がたくさんいるからね。
本物の回答者に参加してほしいのだ。僕が教わりたいよ。
0363エキセントリックなソフィスト ◆BWpafMA3P8me (ワッチョイ 7996-DWwd)
垢版 |
2023/01/11(水) 18:04:16.72ID:KPQyl+hN0
>>360
それに関する「ジーニアス英和辞典」からの説明を引用しましょう

『more A than B  「BよりむしろA(という語句を使う方が適切)」 同一の人・物についての異なる性質などの比較』とあります。
※この構文の重要なポイントは、AとBには、「名詞(句)」や「形容詞の原級」などが来るということです。他者との比較ではなく、主語の持つ性質などの比較なのです。

例:She is more an acquaintance than a friend. (彼女は友人というよりも知り合いだ)
She is more smart than intelligent. (彼は知的というより利口だ)

従って、has more friends than free timeは、「主語は、暇な時間を持っている人というよりは友人たちを持っている人と描写するのがより適切である」が直訳。
あとは、自然な日本語になるよう適当に意訳してもらえればよいでしょう。
0366The OED Loves Me Not (ワッチョイ 2b89-K0Bq)
垢版 |
2023/01/11(水) 21:31:09.49ID:gF0lNB680
>>360 については、顔文字男さん(kawakami さん)が
この下のリンク先で質問してくれているね。

https://forum.wordreference.com/threads/has-more-friends-than-free-time.3997597/

さらに、>>365 が書いてくれているように、

have more friends than free time は、次のようないろんな言い回しと同じような意味であるらしいということが、
検索すればわかるね。

have more friends than time
(1) to hang out with them.
(2) to connect with them all.
(3) to maintain the friendships.
(4) to see them all.
(5) to keep up with all of them.
(6) to spend with them.
0368361 (ワッチョイ 2b89-AyIk)
垢版 |
2023/01/11(水) 21:56:37.64ID:7rSTkgpm0
>>360
すでに他の人が言及しているけど。

ちょっと不安になったので、wordreference,com で質問してみた。
結果的に自分の回答とちょっとズレる部分がある。

ご自分でご確認をお願いします。 (´·ω·`)
0371エキセントリックなソフィスト ◆BWpafMA3P8me (ワッチョイ 7996-DWwd)
垢版 |
2023/01/11(水) 22:39:00.61ID:KPQyl+hN0
「主語は、暇な時間を持っている人というよりは友人たちを持っている人と描写するのがより適切である」(直訳)
 ↓
「友人たちはいっぱいいるが、暇な時間は多くない」(意訳)
「自由時間と友人、どっちが多いかと言えば、友人のほうである」(意訳)
(=主語 has lots of friends but 主語 doesn't have a lot of free time.) ※上記英語フォーラムより
 (=The amount of friends is greater than the amount of free time.) ※上記英語フォーラムより 

ここまではわかっていたが、「友人全員とつきあうには時間が足りない」までもっていけるのね。これは、一本とられました。(T_T)
0372🐤三年英太郎🍓 ◆3CZBjOt3.Y (ワッチョイ 9386-K0Bq)
垢版 |
2023/01/11(水) 22:40:51.71ID:uNTe95xc0
東に言語学の議論あれば
あんなものはくだらねえと毒づき
西に英語学の議論あれば
文法用語ばっか使ってわかった気になってやがると罵り

南にネイティブ友だちに質問する人あれば
白人の肉便器だとさげすみ
北にTwitterで英語を学ぶ人いれば
てめえにはまだ早いと見下す

そーゆーものに
わたしはなり・・・たくない!
0375名無しさん@英語勉強中 (スップー Sdf3-+ALs)
垢版 |
2023/01/11(水) 23:58:06.58ID:agJAI3GJd
You can ask my team,there is multiple times where somebody is like, "On Sunday morning, I wake up and figure it out."


接続詞無いのが気になるし、whereがどういう使われ方してるのかがわかりません
また訳はどんな感じになりますか?
0376名無しさん@英語勉強中 (ワッチョイ 93f0-Or7w)
垢版 |
2023/01/12(木) 01:26:36.45ID:kzQqT6bT0
私のチームメンバーに聞いてみてください。
何度もあるんですよ、「日曜の朝に起きたら問題の解決法がひらめいた」みたいなことが。
0377The OED Loves Me Not (ワッチョイ 2b89-K0Bq)
垢版 |
2023/01/12(木) 08:28:00.66ID:cYkJGmra0
>>375
この英文は個人の電子メールか、Twitter 上で書き散らしたものか、YouTube 上で投稿された
ビデオに対するコメントなのかどうか知らないけど、ともかく気楽に(悪く言えば軽薄に)
書いただけのものだから、通常の文法とか行儀のよい文体から外れていて当然だ。

接続詞のないこういう文体は、至る所にある。至る所で使われているからと言って、それを
僕らが英語ネイティブの上司に対して書くメールの中だとか、企業内での業務連絡や
職業的に書く顧客向けの文章の中で使っていいというわけにはいかない。

さらに、times where なんてのも、至るところにあるんだろうな。だからこそ
https://oneminuteenglish.org/en/times-where-or-times-when/
この上のリンク先にあるような文章をわざわざ誰かが書かないといけなくなる。
0378ブタ耳 (ワッチョイW 4be3-LyGK)
垢版 |
2023/01/12(木) 09:30:32.93ID:wY2dg0Xx0
ブ耳:
I cannot get the meaning of the text below. Especially, the phrase “somebody is like” is unclear.

You can ask my team,there is multiple times where somebody is like, "On Sunday morning, I wake up and figure it out."

ChatGPT:
The text is referring to a situation where someone (in this case, a member of the speaker's team) is making a statement similar to "On Sunday morning, I wake up and figure it out."
It is saying that this type of statement has been said multiple times by someone. The phrase "somebody is like" is colloquial way of introducing a quote or repeating a statement that someone has previously said. In this case the phrase conveys the idea that the speaker has heard this statement many times before.
0379ブタ耳 (ワッチョイW 4be3-LyGK)
垢版 |
2023/01/12(木) 09:31:27.78ID:wY2dg0Xx0
ブ耳:
I’m not sure what the phrase "On Sunday morning, I wake up and figure it out." means. Can you paraphrase it?

ChatGPT:
Sure, the phrase "On Sunday morning, I wake up and figure it out" is an idiomatic way of saying that the person wakes up on Sunday morning without a plan for the day and then decide what to do as the day goes on. It means that the person does not plan ahead what to do on Sunday morning, but rather wait and decide what to do in the moment.
It could also mean that the person is uncertain what they will do on Sunday morning and they have to wait until that morning to decide their plan.
0380ブタ耳 (ワッチョイW 4be3-LyGK)
垢版 |
2023/01/12(木) 09:32:22.30ID:wY2dg0Xx0
ブ耳:
Maybe, it is like this, "On Sunday morning, I wake up and suddenly understand that it is Sunday and that I don’t have to go to work.”?

ChatGPT:
Yes, that is a similar way of interpreting the phrase. The idea is that the person wakes up on Sunday morning without any specific plans or obligations, and the realization that it's Sunday, meaning they don't have to go to work, is something that they understand once they are awake, they then decide how to spend the day based on that realization.
It is a way to express an idea of not having any fixed schedule or obligation on Sunday morning, and using that freedom to plan or decide what to do based on the feeling of the moment.
0381ブタ耳 (ワッチョイW 4be3-LyGK)
垢版 |
2023/01/12(木) 09:33:12.28ID:wY2dg0Xx0
ブ耳:
Or, like this, “On Sunday morning, I wake up and come up with a solution for the problem on which I have been working all through the past week.”?

ChatGPT;
Yes, that's another possibility of interpreting the phrase. This interpretation suggests that the person has been thinking about a problem or an issue during the weekdays and when they wake up on Sunday morning, they finally come up with a solution or an idea that they have been searching for. It emphasizes that the person's mind is still active on Sunday morning and not only relaxed, and that they are still actively working on something even though it is the weekend.
The phrase "figure it out" can have a broad meaning, depending on the context it is used, in this interpretation means coming up with an idea or solution to something that has been giving the person trouble.
0386名無しさん@英語勉強中 (ワッチョイW 93f0-HWoF)
垢版 |
2023/01/12(木) 13:38:46.84ID:dG2YRWM30
現在、電話番号を調べていますので少々時間をください。

↑英語でどう書いたらいいですか?
0389ブタ耳 (ワッチョイW 4be3-LyGK)
垢版 |
2023/01/12(木) 14:22:14.16ID:wY2dg0Xx0
>>386
ChatGPTに尋ねてみました。

"I am currently looking up a phone number, so please give me a moment."

もっと丁寧なバージョンとしては、
"I apologize for the inconvenience, please allow me a moment to retrieve the phone number for you, thank you for your patience."

詳しいやり取りは>>382のスレに報告しました。
0390名無しさん@英語勉強中 (ワッチョイ c1bc-0nh0)
垢版 |
2023/01/12(木) 15:27:53.86ID:OoXVxwjh0
>>372
と、老害の帝王が申しております 苦笑
0391名無しさん@英語勉強中 (ワッチョイ c1bc-0nh0)
垢版 |
2023/01/12(木) 15:30:11.44ID:OoXVxwjh0
記述の対格という名称がデタラメであろうが、なかろうが

まず確定している事実は
I bought a car the size of a killer whale.
Why can’t you be like other girls your age?
をメトニミーで説明することは不可能

記述の対格という概念は不要、メトニミーで説明できるから
というアホ太郎の主張がデタラメであるという事実は覆せない。
0392名無しさん@英語勉強中 (ワッチョイ c1bc-0nh0)
垢版 |
2023/01/12(木) 15:32:50.68ID:OoXVxwjh0
>>162
偉そうに言ってるけど
いざあの世の江川先生と対面したらボコボコにされるだろうねw

江川「じゃあ君、Why can’t you be like other girls your age?
という文をメトニミーで説明できるのかね?」
ホモ太郎「・・・・・」
0393名無しさん@英語勉強中 (ワッチョイ c1bc-0nh0)
垢版 |
2023/01/12(木) 15:40:05.36ID:OoXVxwjh0
>>190
>>比喩という意味論的解釈に、同格云々の機能論的視点を持ち込んで反論できた気になってるアホですか?🤔

意味解釈そのものを、機能的視点によって反論しているのではない。
そもそも、その文についての解釈自体はたまたま<成り立っている>と言っているのだ、馬鹿め。

Her eyes, a deep blue, were quite impressive.
このケースでは、記述の対格である a deep blue が意味的に非制限的に修飾しているため、
The color of her eyes, a deep blue, were quite impressive.
と、同格の名詞句として、解釈することがたまたまでき得ると言っている。
馬鹿過ぎてわからないのだろうか?

(このような場合でも、述部が例えば、"were getting weaker" だとしたら、勝手に
Her eyesをThe color of her eyesの意味だと強弁すると、明らかにおかしなことになる。)
0394名無しさん@英語勉強中 (ワッチョイ c1bc-0nh0)
垢版 |
2023/01/12(木) 15:46:25.59ID:OoXVxwjh0
最終結論

記述の対格という名称がデタラメであろうが、なかろうが

まず確定している事実は
I bought a car the size of a killer whale.
Why can’t you be like other girls your age?
をメトニミーで説明することは不可能。

「記述の対格」と呼ばれる用法には、形容詞的な意味合いがあるのは明らかである。

Sweetはofが欠落したものだと解した。
Onions(5文型の元になる理論を提唱した人)が「記述の対格」と命名した。

歴史的に、本当にofが欠落して生じた用法かはわからない。
しかし、意味的にはそのように解釈できるとネイティブの学者は主張しているわけ。
実際、主格補語や目的格補語だけでなく、後置修飾として使われている場合も
きれいにそれで理解できる。
そして、そのように理解すればメトニミーで説明する必然性は皆無である。

これを、メトニミーだ!なんて言ってるアホ太郎はやはりオツムガ足りないのだろう。
0395名無しさん@英語勉強中 (ワッチョイ c1bc-0nh0)
垢版 |
2023/01/12(木) 15:48:01.84ID:OoXVxwjh0
反論するのなら
I bought a car the size of a killer whale.
Why can’t you be like other girls your age?
をメトニミーで説明してみろ、馬鹿。

できるわけがないのだから。
色々と言い訳をいって誤魔化して逃走するのだろうな、糞ホモ野郎。
0396名無しさん@英語勉強中 (ワッチョイ c1bc-0nh0)
垢版 |
2023/01/12(木) 15:52:53.93ID:OoXVxwjh0
>>394
訂正
×必然性
〇必要性
0397🐤三年英太郎🍓 ◆3CZBjOt3.Y (ワッチョイ 9386-K0Bq)
垢版 |
2023/01/12(木) 16:28:00.90ID:/5BCQzWZ0
無能な働き者って一番困るのよね・・・
NGする前に教えといてやろう

I was your age
I, your age

other girls your age
other girls were your age

a car the size of a killer whale
The car was the size of a killer whale

her eyes, a deep blue
her eyes were a deep blue

SVCだろうが同格だろうが、そんなこたー全く無関係
ふたつの対象物の関係を意味論的の分析してるのに、統語的問題はいま一切関係ない

※ またその意味論とは無関係で、江川のいう目的格云々は的外れ
江川せんせーは当然おれよりも英語がおできになるけど、そのこととこの問題は別

お前はすべて無関係なものを無理に結び付けてるあほう
0400🐤三年英太郎🍓 ◆3CZBjOt3.Y (ワッチョイ 9386-K0Bq)
垢版 |
2023/01/12(木) 19:52:08.86ID:/5BCQzWZ0
>>397
さらにやさしく

「AはBだ」という文の、AとBの”意味における”関係性をメトニミーで説明したのである

それを「Bは形容詞句だ!」と言ったのがアホソフィスト先生

それを「『A, B』は同格だ。『A is B』とは別物!」と言ったのがアホテテン
0403名無しさん@英語勉強中 (ワッチョイ 0932-Iqi2)
垢版 |
2023/01/12(木) 21:08:09.50ID:YilCjHj30
副詞句表現である with time to spare「時間に余裕をもって」を
with plenty of time to spare と書く必要はありますでしょうか?

あとホテル内の英語表示に朝の朝食フロアは混み合うので早めに来てください
の意味で ...... with plenty of time. と語尾に書かれていましたが
この表現だけだとちょっと不自然な感じもするのですが、どうでしょうか?
0404The OED Loves Me Not (ワッチョイ 2b89-K0Bq)
垢版 |
2023/01/12(木) 21:16:01.83ID:cYkJGmra0
>>403
朝の朝食フロアは混み合うので早めに来てください

Please come to the dining hall for breakfast in good time.
It gets crowded at that hour.

上のように、in good time と言えば十分だし、その方が自然なんだろうなと思う。
0405名無しさん@英語勉強中 (テテンテンテン MM4b-0nh0)
垢版 |
2023/01/12(木) 21:47:22.04ID:x9dXApcVM
本当に往生際の悪い屑だな

>>397
>SVCだろうが同格だろうが、そんなこたー全く無関係
>ふたつの対象物の関係を意味論的の分析してるのに、統語的問題はいま一切関係ない

関係なくねーわ、基地外
ふたつの対象物の関係を、一方が他方のメトニミーと分析することは

記述の対格がSVCのCや、意味的に非制限的に後置修飾しているときは
たまたま整合性がとれる。

I was your age.
→My age was yourn age.

Her eyes, a deep blue, were quite impressive.
→The color of Her eyes, a deep blue, were quite impressive.
0406名無しさん@英語勉強中 (テテンテンテン MM4b-0nh0)
垢版 |
2023/01/12(木) 21:50:57.11ID:x9dXApcVM
ところが
意味的に後置修飾しているときには
このような解釈をすると、文意がおかしなことになる。

↓例えば
Why can’t you be like other girls your age?

このgirlsが、この文の中でthe ages of girls の意味を表すわけねーだろ 苦笑
0407名無しさん@英語勉強中 (テテンテンテン MM4b-0nh0)
垢版 |
2023/01/12(木) 21:51:50.52ID:x9dXApcVM
「記述の対格」と呼ばれる用法には、形容詞的な意味合いがあるのは明らかである。

Sweetはofが欠落したものだと解した。
Onions(5文型の元になる理論を提唱した人)が「記述の対格」と命名した。

歴史的に、本当にofが欠落して生じた用法かはわからない。
しかし、意味的にはそのように解釈できるとネイティブの学者は主張しているわけ。
実際、主格補語や目的格補語だけでなく、後置修飾として使われている場合も
きれいにそれで理解できる。
そして、そのように理解すればメトニミーで説明する必然性は皆無である。

これを、メトニミーだ!なんて言ってるアホ太郎はやはりオツムガ足りないのだろう。
0408名無しさん@英語勉強中 (テテンテンテン MM4b-0nh0)
垢版 |
2023/01/12(木) 21:55:43.68ID:x9dXApcVM
>>400

>「AはBだ」という文の、AとBの”意味における”関係性をメトニミーで説明したのである

なんか言い訳をつぶやきはじめたw
記述の対格に形容詞的に意味があるとさえ、認めれば
実際、主格補語や目的格補語だけでなく、後置修飾として使われている場合も
きれいにそれで理解できる。
そして、そのように理解すればメトニミーで説明する必然性は皆無である。

これを、メトニミーだ!なんて言ってるアホ太郎はやはりオツムガ足りないのだろう。
0409名無しさん@英語勉強中 (テテンテンテン MM4b-0nh0)
垢版 |
2023/01/12(木) 21:57:28.32ID:x9dXApcVM
Why can’t you be like other girls your age?
のyour ageが、形容詞的な意味合いあると解釈するなら
I was your ageのyour ageも同様に解するべき
そして、その場合、メトニミーがどうとかいという話は全く不要

すべての学問にオッカムの剃刀が指針となるとは限らないかもしれんが
英文法については従うべき。
0410名無しさん@英語勉強中 (JP 0H9d-0nh0)
垢版 |
2023/01/12(木) 22:06:27.96ID:9Eq5JgnCH
本当に往生際の悪い屑だな

>>397
>SVCだろうが同格だろうが、そんなこたー全く無関係
>ふたつの対象物の関係を意味論的の分析してるのに、統語的問題はいま一切関係ない

関係なくねーわ、基地外
ふたつの対象物の関係を、一方が他方のメトニミーと分析することは

記述の対格がSVCのCや、意味的に非制限的に後置修飾しているときは
たまたま整合性がとれる。

I was your age.
→My age was yourn age.

Her eyes, a deep blue, were quite impressive.
→The color of Her eyes, a deep blue, were quite impressive.
0411名無しさん@英語勉強中 (JP 0H9d-0nh0)
垢版 |
2023/01/12(木) 22:08:06.84ID:9Eq5JgnCH
ところが
意味的に後置修飾しているときには
このような解釈をすると、文意がおかしなことになる。

↓例えば
Why can’t you be like other girls your age?

このgirlsが、この文の中でthe ages of girls の意味を表すわけねーだろ 苦笑

「文意が成り立つかなんて、関係ない。
ふたつの対象物の関係をメトニミーで説明可能と言ってるだけ」
とでも言い出すつもりなのかね、この馬鹿は 苦笑
0412名無しさん@英語勉強中 (JP 0H9d-0nh0)
垢版 |
2023/01/12(木) 22:08:31.41ID:9Eq5JgnCH
>>400

>「AはBだ」という文の、AとBの”意味における”関係性をメトニミーで説明したのである

なんか言い訳をつぶやきはじめたw
記述の対格に形容詞的に意味があるとさえ、認めれば
実際、主格補語や目的格補語だけでなく、後置修飾として使われている場合も
きれいにそれで理解できる。
そして、そのように理解すればメトニミーで説明する必然性は皆無である。

これを、メトニミーだ!なんて言ってるアホ太郎はやはりオツムガ足りないのだろう。
0413名無しさん@英語勉強中 (JP 0H9d-0nh0)
垢版 |
2023/01/12(木) 22:08:56.88ID:9Eq5JgnCH
Why can’t you be like other girls your age?
のyour ageが、形容詞的な意味合いあると解釈するなら
I was your ageのyour ageも同様に解するべき
そして、その場合、メトニミーがどうとかいという話は全く不要

すべての学問にオッカムの剃刀が指針となるとは限らないかもしれんが
英文法については従うべき。
0414名無しさん@英語勉強中 (JP 0H9d-0nh0)
垢版 |
2023/01/12(木) 22:11:47.68ID:9Eq5JgnCH
つべこべ言い訳をせず、反論するのなら
I bought a car the size of a killer whale.
Why can’t you be like other girls your age?
をメトニミーで説明してみろ、馬鹿。

できるわけがないのだから。
色々と言い訳をいって誤魔化して逃走するのだろうな、糞ホモ野郎。

できないのなら
「記述の対格には、形容詞的な意味合いがあると理解すればそれで足りるのであって
メトニミーがどうとかアホなことを言ってすみませんでした」
と謝罪しろ、基地外
0415名無しさん@英語勉強中 (テテンテンテン MM4b-0nh0)
垢版 |
2023/01/12(木) 22:38:45.06ID:W4XThdhxM
メトニミーで解釈したら、文の意味が通らなくなる。
よって、そのような解釈はおかしいと批判してるのに

>>SVCだろうが同格だろうが、そんなこたー全く無関係
>>ふたつの対象物の関係を意味論的の分析してるのに、統語的問題はいま一切関係ない

と、尋常じゃないアホな再反論をしてくる知能障害wwwwwww
0416名無しさん@英語勉強中 (JP 0H9d-0nh0)
垢版 |
2023/01/12(木) 22:45:02.21ID:9Eq5JgnCH
メトニミーで解釈したら、文の意味が通らなくなる。
よって、そのような解釈はおかしいと批判してるのに

>>SVCだろうが同格だろうが、そんなこたー全く無関係
>>ふたつの対象物の関係を意味論的の分析してるのに、統語的問題はいま一切関係ない

と、尋常じゃないアホな再反論をしてくる哀れな知能障害wwwwwww
0417名無しさん@英語勉強中 (JP 0H9d-0nh0)
垢版 |
2023/01/12(木) 22:47:59.47ID:9Eq5JgnCH
アホ太郎
ふたつの対象物の関係を意味論的の分析してるだけと逃走の準備を始めたが

>>94
って、記述の対格なんて概念は必要なくて
メトニミーで説明できちゃうよってことを主張してると解釈するよね、普通w
0418名無しさん@英語勉強中 (JP 0H9d-0nh0)
垢版 |
2023/01/12(木) 22:48:49.94ID:9Eq5JgnCH
つべこべ言い訳をせず、反論するのなら
I bought a car the size of a killer whale.
Why can’t you be like other girls your age?
をメトニミーで説明してみろ、馬鹿。
文意が通るようにだぞ!!!!!!!!

できるわけがないのだから。
色々と言い訳をいって誤魔化して逃走するのだろうな、糞ホモ野郎。

できないのなら
「記述の対格には、形容詞的な意味合いがあると理解すればそれで足りるのであって
メトニミーがどうとかアホなことを言ってすみませんでした」
0419エキセントリックなソフィスト ◆BWpafMA3P8me (ワッチョイ 7996-DWwd)
垢版 |
2023/01/12(木) 23:17:49.92ID:Cg+UnOfr0
>>400
3年ちゃん、メトニミー説が間違いであることがまだ理解できないの?あんなにわかりやすく説明してあげたのに。ほんと理解力がないんだね。もう一度説明してあげる。

ジーニアスの「size」の項目に次の例が載っている。
『an area the size of Nagoya 名古屋ほどの大きさの地域 《◆名詞のあとに来たときも同様にofはしばしば省略される》』

この用法は、名詞を後ろから修飾できるという性質があるのだ。これは、ofが省略された言い方も「形容詞(的)句」のように働いていることを意味している。

これは、The area is the same size of Nagoya.の「the same size of Nagoya」も、形容詞(的)句のように働いている証拠である。
よって、主語がメトニミーである必要などまったくない。

以上のことから、3年ちゃんの>>94の解釈(メトニミー説)は、間違いである。
0420エキセントリックなソフィスト ◆BWpafMA3P8me (ワッチョイ 7996-DWwd)
垢版 |
2023/01/12(木) 23:24:10.14ID:Cg+UnOfr0
>>400
私が間違っている論拠もロクに示せない頭の悪そうな反論はいいから、はやく、>>419に反論してみてよ。書いてあることは、すごく単純なことだよ。

・the same sizeは形容詞(的)句になれる。これはyour ageも同様だ。
・「形容詞(的)句」なら、the sizeもyour ageも、被修飾語(または主語)とイコールである必要はない。
よって、被修飾語はメトニミーである必要はない。

これのどこが完全に間違ってるの? 論拠があるなら出してみてよ。出せないなら、自分がアホ論を言ってることを認めましょう。(^O^)
0422エキセントリックなソフィスト ◆BWpafMA3P8me (ワッチョイ 7996-DWwd)
垢版 |
2023/01/12(木) 23:41:48.81ID:Cg+UnOfr0
>>421

・the same sizeは形容詞(的)句になれる。これはyour ageも同様だ。
・「形容詞(的)句」なら、the sizeもyour ageも、被修飾語(または主語)とイコールである必要はない。
よって、被修飾語はメトニミーである必要はない。


3年ちゃんは、この考えがなぜダメなのかをまだ説明していないよ。ほんとに、大ウソつきなんだから。それとも、健忘症?(^-^)
0423名無しさん@英語勉強中 (ワッチョイ 7996-Ci/c)
垢版 |
2023/01/13(金) 09:46:15.86ID:b8lzz30z0
お願いします

It will be a long time before Mars is this close to Earth again.(火星がこれほど地球に再び近付くのはずっと先だろう)

この文のthisが不自然な気がします。this closeで「これほどの接近」と解釈すべきなんでしょうか?
むしろ、↓こういう文の方がわかりやすい気がしますが、ダメですか?
It will be a long time before Mars is close to Earth like this again.
0424The OED Loves Me Not (ワッチョイ 2b89-K0Bq)
垢版 |
2023/01/13(金) 10:21:59.91ID:CwbohBKc0
>>423
(1) It will be a long time before Mars is this close to Earth again.

「この文のthisが不自然」って、なぜそう思うのかな?理由も根拠も証拠もなしに
そんなことを言わないでほしい。このような this や that は巷に溢れかえり、
会話でも書き言葉でも大いに使われ、きわめて自然で英語らしい。

>>this closeで「これほどの接近」と解釈すべきなんでしょうか?

当然のことだ。

(2) It will be a long time before Mars is close to Earth like this again.

(1) よりも (2) の方がわかりやすいと思うのは、あなたが読んできた英文の量が
あまりに少ないからだ。(1) の方がはるかに英語らしく、英語ネイティブにとっては
その方がはるかにわかりやすいからこそ、このようにふだん大いに使われるのだ。
(2) なんていう英文を、僕はほとんど(あるいは「まったく」)見聞きしたことがない。
0425The OED Loves Me Not (ワッチョイ 2b89-K0Bq)
垢版 |
2023/01/13(金) 10:28:39.12ID:CwbohBKc0
>>423
"is this close" がごく一般的で "is close like this" のような形をほとんど(あるいはまったく)
僕が見聞きしたことがないと言ってしまったが、検索してみると後者の用例はあるにはある。
しかし少ないな。


(1) is close like this
https://www.google.com/search?q=%22is+close+like+this%22&newwindow=1&rlz=1C1FQRR_enJP949JP949&sxsrf=AJOqlzWBLuH4eBvQzvOY6WQ5QCgZFgfL1g:1673573051124&source=lnms&tbm=nws&sa=X&ved=2ahUKEwic4d65scP8AhWKsVYBHTvaByQQ_AUoA3oECAEQBQ&biw=1536&bih=746&dpr=1.25

(2) is this close
https://www.google.com/search?q=%22is+this+close%22&newwindow=1&rlz=1C1FQRR_enJP949JP949&sxsrf=AJOqlzX0-P11y4HMEBB_PUbbcphtiIsWEg:1673573083409&source=lnms&tbm=nws&sa=X&ved=2ahUKEwi1q5HJscP8AhUcplYBHXSwBXgQ_AUoAnoECAEQBA&biw=1536&bih=746&dpr=1.25
0426The OED Loves Me Not (ワッチョイ 2b89-K0Bq)
垢版 |
2023/01/13(金) 10:32:55.15ID:CwbohBKc0
>>423
>>425 にて示した "is close like this" の用例は、通常の Google 検索結果のみならず、
上に示したようなニュースサイトに限った画面上でも一応は出てくる。

しかし "is this close" が圧倒的に英語らしく英語圏で普通に使われていて、
"is close like this" がそれに比べるときわめて少ないことは確かのようだ。
僕が思うには、おそらくは "is close like this" は崩れた英語なのか、あるいは
移民たちが流行らせてしまった言い回しなのかもしれないと思っている。
現にこの下に示す Ngram.viewer の調査によると、書籍データ検索データベース内での
統計によると、"is this close" はもちろん大いに使われているのに対して、
"is close like this" の出現頻度はゼロ。

is close like this と is this close との、書籍データ検索データベース内での出現頻度グラフ
https://books.google.com/ngrams/graph?content=is+close+like+this%2Cis+this+close&year_start=1800&year_end=2019&corpus=en-2019&smoothing=3
0427名無しさん@英語勉強中 (ワッチョイW c1fb-S0bH)
垢版 |
2023/01/13(金) 11:42:16.54ID:+YWA76fa0
oedの書き込みばかりになったらつまらなくなるので
適当に無視しないといけないな。役に立つ書き込みは
読みながら。
oedは自分語りをするけど、自分に対して肯定しか
しないんだよな。全ては環境が悪い、というふうに
やる人で。
0428名無しさん@英語勉強中 (ワッチョイW c1fb-S0bH)
垢版 |
2023/01/13(金) 11:44:55.38ID:+YWA76fa0
>>423
オーレックスにはこう書いてあった

━━副
これほど,こんなに
I know this much.
このくらいまでは知っている
The fish was about this big.
魚はこれくらいの大きさだった(◆ 身振りなどで実際に大きさを示しながら言う.→ so 副)
0429ブタ耳 (ワッチョイW 4be3-LyGK)
垢版 |
2023/01/13(金) 11:54:59.20ID:QwPTgzdZ0
>>423
this を辞書でひいてください。
「副詞」としての用例が出てるはずです。

以下、ランダムハウス英和から引用。
(特に最後の▶︎以下の説明が良い。このthatもsoと同じような副詞です)

((話)) 量・程度を示す形容詞・副詞と共に,通例否定的な構文でこれだけ,これほど,こんなに,この程度まで:
this far この距離まで,こんなに遠く
this softly こんなにそっと
Add this much water to the ingredients. 材料にこれだけの水を加えなさい(cf. adj. THIS much)
I didn't expect to have to wait this long. これほど長時間待たねばならないとは思わなかった
I didn't expect it to be this full. それがこんなに一杯になっているとは思いませんでした
▶︎this は現在の状態に言及する.that は過去の状態に言及し,so であれば中立的な表現となる.
0430The OED Loves Me Not (ワッチョイ 2b89-K0Bq)
垢版 |
2023/01/13(金) 12:02:51.59ID:CwbohBKc0
>>423 はもしかして、this close の this が so という意味、
つまり副詞だってことに気づかなかったのか?
that close だって、so close という意味(「あんなに close」という意味)
だから、this と that は両方とも、副詞として頻繁に、会話でも書き言葉でも
頻繁に出てくる。

(学術文献などのいかめしい文章に出てこないかもしれないが、評論文
くらいになら大いに出てくる。)
0431The OED Loves Me Not (ワッチョイ 2b89-K0Bq)
垢版 |
2023/01/13(金) 12:12:20.30ID:CwbohBKc0
>>423 が提示している this close に関連して、
"not that + 形容詞" の用例として "not that bad" を
使った面白い用例はないかと思っていたら、
次のような笑いを誘う用例を見つけた。

He said he picked him because he’s seen him naked before,
and because he’s not married and not attractive (at all, eww, a
lthough my husband says he’s ★not THAT bad★) and therefore not a threat to steal me away.
https://slate.com/human-interest/2023/01/dear-prudence-photoshopping-erotic-images-wife.html

リンク先を見ると、この話の前後の文脈は長い。けっこう面白そうなので、斜め読みしたい人はしてみてほしい。
実にくだらないといえば、確かに下らない。でも英語表現を追いかけながら笑いたいと思うなら、読んでみてほしい。
0434The OED Loves Me Not (ワッチョイ 2b89-K0Bq)
垢版 |
2023/01/13(金) 14:23:37.31ID:CwbohBKc0
>>423 に関して
因数分解を解いている人に向かって「足し算引き算が大切だよ」なんてことを
平気で言える人がいるので、仕方なく、次のような超初心者でもわかる
ことを付け加えないといけなくなる。

>>424 にて僕が

*******

>>this closeで「これほどの接近」と解釈すべきなんでしょうか?

当然のことだ。

********

上のように答えたとき、まさか僕が this を指示代名詞で、close を「接近」という
意味の名詞として理解しているなんていう、脳の腐りきったゾンビみたいな
解釈をしていしまう奴はいないだろうな。

質問者が「これほどの接近」と解釈すべきか、と言ったとき、
僕は当然のことながら質問者が
「これほどまで接近している」(これほどまで近い)という
意味でそういったのだと解釈した。

だからこそ僕は「当然のことだ」と言った。

まさか this close を「この接近」という意味として解釈するなんて
いう馬鹿な解釈をする人なんていないと思い込んでいた。

以上のような、ごく当たり前の、文字通りの馬鹿でもわかるような
ことをいちいち書かねばならないとは、日本人は終戦後ずっと
脳死しているのだろうか?
0435The OED Loves Me Not (ワッチョイ 2b89-K0Bq)
垢版 |
2023/01/13(金) 14:29:44.37ID:CwbohBKc0
第一、closing ならば「接近」という意味での名詞として解釈できるが、
close で「接近」という意味の名詞になるはずがないことは、
馬鹿でもすぐに気づく。というか、最初からそんなことを言う必要もない。

this close と言えば、so close という意味に決まっている。
そういう前提で、僕は最初から話をしていた。

本当にレベルが低くて、まるで幼稚園児に対するように
話をしないといけないらしいので、疲れ切る。
0436名無しさん@英語勉強中 (ワッチョイW c1fb-S0bH)
垢版 |
2023/01/13(金) 14:42:48.09ID:+YWA76fa0
>>434
わからないから質問してるのに、

脳の腐りきったゾンビ
馬鹿な解釈をする人
脳死している
幼稚園児に対するようにはなしをしないといけないらしいので、疲れる

とか書かれると>>423が気の毒すぎる。ひでーと思った。
ここ質問スレなのに。
0437The OED Loves Me Not (ワッチョイ 2b89-K0Bq)
垢版 |
2023/01/13(金) 14:51:32.57ID:CwbohBKc0
僕は、>>423 の悪口を言っているのではない。
僕の言っていることを、変に誤解している奴がいるらしいので、
そいつのことを言っていた。

>>423 は謙虚に質問している。質問者は超初心者なので仕方がない。
問題は、回答者のふりをしていながら、何の努力もしないで
自分で物を考えることを幼少時代からやめてしまっているがゆえに、
脳が腐ってしまっている連中だ。
0438The OED Loves Me Not (ワッチョイ 2b89-K0Bq)
垢版 |
2023/01/13(金) 14:58:51.39ID:CwbohBKc0
>>424 の僕のコメントを読んで、
僕が this close の this を指示代名詞である「あの」という意味で、
close が「接近」という意味の名詞として理解しているだなどという
馬鹿げたことを考えてしまう人がいるとは、本当にびっくりした。

しかし >>424 の僕の発言だけを読んでいて、しかもv僕の実力が
どれくらいかをまったく知らない人が見たら、そう思っても仕方がないだろうとは思う。

しかし僕に言わせれば、その程度の英語力って英検で言えば2級とか TOEIC 700 レベルだろう。
僕がそんな変な誤解をしてるわけないというのは、文字通りの馬鹿でも気づくことだと思っていた。

だから今回のことには、本当にびっくり仰天した。
想像力もなければ、ほんの少しだけ自分で物を考えて客観的に冷静に物を見ようという
態度がまったくない人がいるもんだな、と思った。

それなら、英語のみならず日本語の力も、時代と共に低下していって当たり前だなと思う。
0439名無しさん@英語勉強中 (テテンテンテン MM4b-8Mzb)
垢版 |
2023/01/13(金) 15:19:20.77ID:IGVBMml/M
例えば中学生レベルの知識しかない人なら
thisが副詞で使われることを知らない可能性は十分予想できることだし、
それを教えるのに疲れ切ることもないだろうし、
戦後の日本人は・・・とまで話を広げる必要はないと思うけどなあ。
0440名無しさん@英語勉強中 (テテンテンテン MM4b-8Mzb)
垢版 |
2023/01/13(金) 15:25:37.14ID:IGVBMml/M
>僕が this close の this を指示代名詞である「あの」という意味で、
>close が「接近」という意味の名詞として理解しているだなどという
>馬鹿げたことを考えてしまう人がいるとは

そんな人はいないと思いますけど?
どのレスのこと、言ってます?
0441The OED Loves Me Not (ワッチョイ 2b89-K0Bq)
垢版 |
2023/01/13(金) 16:01:07.67ID:CwbohBKc0
>>440
>>433 のことを言ってるんだ。

中学生レベルの人の質問には、最初から僕はなるべく答えないようにしているので、
疲れ切ったりはしないんだよね。

僕が疲れるのは、回答者のふりしていて、実は中学2年レベルの英語力しかない人。
そして、なにかと他人にかみつく。僕はそいつらをまったく無視しているのに、何かと
僕に絡もうとする。

それから、偏差値に関係なく、精神的に幼稚園児レベルの人には、本気で疲れるので、
そういう人のことは片っ端から透明あぼーんして、最初から完全に無視している。

テテンさんは、そういう人のことを無視しないで不良少年補導員みたいに
社会福祉活動をしておられるので、僕はあなたを偉いと思っている。いくらか相手をしてあげないと、
そいつらはもっと悪いことをするだろうけど、あなたがその相手をしてあげてるから、
かろうじてそいつらは最小限度の悪事しか働かないで済んでいる。
0444The OED Loves Me Not (ワッチョイ 2b89-K0Bq)
垢版 |
2023/01/13(金) 16:25:09.05ID:CwbohBKc0
うんざりしているのは、こっちの方だ。
周囲の連中が俺に喧嘩を売って来なければ、
俺は静かに淡々と英語についてのみコメントを続ける。
0445名無しさん@英語勉強中 (テテンテンテン MM4b-0nh0)
垢版 |
2023/01/13(金) 17:36:47.40ID:sO3kDu05M
>>441
本当だ。失礼。
>>433 はアホですな。
0446名無しさん@英語勉強中 (テテンテンテン MM4b-0nh0)
垢版 |
2023/01/13(金) 17:41:14.69ID:sO3kDu05M
>>442
君がアホな煽りをしたことが
原因になってんだろ 苦笑

アホな煽りじゃないというなら
>>433 の真意を説明してみたまえ
0447名無しさん@英語勉強中 (テテンテンテン MM4b-0nh0)
垢版 |
2023/01/13(金) 18:00:53.99ID:sO3kDu05M
確かに「これほどの接近」という訳は名詞句のものであるのだが
OED氏の実力を知っている人ならば、副詞の「これほど」という用法を知らない筈がないとわかるので
ちょっとした書き間違い程度のことと、特に意識にも上らずにスルーするのだが

(ワッチョイW c1fb-S0bH)くんは、きっと今回はじめて英和辞典をみて
この用法を知ったのだろうw
で、OED氏も知らないようなマイナーな用法かもとも思ってしまったのだと推測する。
OED氏をやりこめるとでも思ったのだろうなぁ。
(違ったらスマンw)
0450😉三年英太郎🌈 ◆3CZBjOt3.Y (ワッチョイW 9386-iXfb)
垢版 |
2023/01/13(金) 18:26:29.09ID:VnUdtQWd0
ASDの主な特性としては、次の2つが挙げられます。

👉対人関係における相互的反応の障害

「相手の心情を表情や言葉のニュアンスから察することが苦手」「場の雰囲気を読むことが難しい」などが多いです。

👉同一性へのこだわり

「特定の対象に対して強い興味を示す」「反復的で機械的な動作が見られる」「こだわりが強く、状況に応じた柔軟な対応が難しい」などが多いです。
0451😉三年英太郎🌈 ◆3CZBjOt3.Y (ワッチョイW 9386-iXfb)
垢版 |
2023/01/13(金) 18:27:40.03ID:VnUdtQWd0
⭐ASDの治療方法

ASDを完全に治療する方法は、今のところはないといわれています。自閉症スペクトラムの特性があることで日常生活や仕事で困りごとなどが発生している場合、自分にあった対処法や環境調整、スキルを身につけることで、困りごとなどを軽減させていきます。
https://works.litalico.jp/column/developmental_disorder/008/
0452423 (ワッチョイ 7996-Ci/c)
垢版 |
2023/01/13(金) 18:28:56.64ID:b8lzz30z0
>>423です。
ご回答いただいた皆様、ありがとうございました。

>>429
thisが副詞の用法があるということ、まったく知りませんでした。ありがとうございました。



私の質問がどういうわけだか気にいらなかった方、また、その方の私への回答の仕方が気にいらなかった方等
いらっしゃるようですが、私としては、今後質問する際の参考にさせていただくばかりです。
英語が苦手な質問者にも感情があることをわかっていただいている方には感謝申し上げます。
0454エキセントリックなソフィスト ◆BWpafMA3P8me (ワッチョイ 7996-DWwd)
垢版 |
2023/01/13(金) 19:17:03.60ID:Hie8Pfsu0
□□□三年英太郎ちゃんの意見が間違っていることのわかりやすい説明(まとめ)□□□

三年英太郎ちゃんの説明: 「When I was your age」は、I と your ageが概念的に異なるものなので、「I」は「my age」のメトニミー(比喩)なのだ。

私の主張: このyour ageは、The area is the size of Nagoya.のthe sizeと同じものである。この文は、The area is of the size of Nagoya.と言っても文法的に正しい。同様に、「When I was your age」は、「When I was of your age」と言っても、文法的に正しい。

ジーニアスの「size」の項目に次の例が載っている。
『an area the size of Nagoya 名古屋ほどの大きさの地域 《◆名詞のあとに来たときも同様にofはしばしば省略される》』

これは修飾関係にあるので、「(of) the size」は、「(of) your age」と同様、形容詞のように働く句であることがわかる。(英語では「Adjectival Phrase」(形容詞的な句)と呼ばれ、日本の教育現場では「形容詞句(の一種)」として扱われている。) ジーニアスの例文の被修飾語「area」が「エリアの大きさ」という意味の比喩でないのだから、「When I was (of) your age」の「I」も何かの比喩(メトニミー)であるわけがない。「(of) your age」は、主語の持ってるプロパティ(属性・性質)を示す語、つまり主語を形容する語なのだから、同じ概念である必要はない。よって、3年ちゃんのメトニミー説は大間違いである。
0456名無しさん@英語勉強中 (テテンテンテン MM4b-0nh0)
垢版 |
2023/01/13(金) 19:34:49.63ID:sNd7vLegM
>>450
素直にメトニミー説の間違いを認めたらどうですか?患者さん。
0459The OED Loves Me Not (ワッチョイ 2b89-K0Bq)
垢版 |
2023/01/13(金) 22:32:29.10ID:CwbohBKc0
>>435
>>第一、closing ならば「接近」という意味での名詞として解釈できるが、

ボケたことを言ってしまっていた。これは大間違いだった。
closing なら、「閉鎖」という意味の名詞となる。決してこれは
「接近」という意味の名詞にはならない。
coming close なら「接近」という意味の名詞句となる。

いやはや、ボケがひどい。「こんなにまでの接近」と言われたとき、
それを「こんなに接近している」(coming this close OR coming so close)
という意味だと解釈してしまっていた時も、
やはりボケてしまっていたのだろう。
0460名無しさん@英語勉強中 (テテンテンテン MM4b-0nh0)
垢版 |
2023/01/13(金) 22:35:40.48ID:sNd7vLegM
>>458
君には聞きたいことがあったんだ。

>>253
>このスレで質問して良かった。
>記述の対格についてよくわかった。

と、あるけど、何が良かったわけ?
ofを補って解釈すればいいだけの話であって
それは普通に英文法書にも書いてあると思うのだが・・・

メトニミーの話はどう思ったの?
0461名無しさん@英語勉強中 (アークセーT Sx6d-wtyD)
垢版 |
2023/01/14(土) 03:14:26.31ID:PT+39W0Jx
「My having been attached to any particular person does not make me feel the same attachment to the n
ext person that I may chance to meet; but, if I have once associated strong feelings of delight with the objects of natural scenery, the tie becomes indissoluble, and I shall ever after feel the same attachment to other objects of the same sort. I remember when I was abroad, the trees, and grass, and wet leaves, rustling in the walks of the Thuilleries, seemed to be as much English, to be as much the same trees and grass, that I had always been used to, as the sun shining over my head was the same sun which I saw in England; the faces only were foreign to me. Whence comes this difference? It arises from our always imperceptibly connecting the idea of the individual with man, and only the idea of the class with natural objects. In the one case, the external appearance or physical structure is the least thing to be attended to; in the other, it is everything. 」
の最後の文のIn the one caseとin the otherについて、私はIn the one caseとは人間のことでin the otherは自然のことと解釈したのですが、他の方からは逆にIn the one caseとは自然のことでin the otherは人間のことではないかと言われました。
どちららが正しいでしょうか?
0464The OED Loves Me Not (ワッチョイ 8689-hn8B)
垢版 |
2023/01/14(土) 05:58:04.17ID:1OvUoQN20
>>461
>>It arises from our always imperceptibly connecting the idea of the individual with man, (人間について)
>>and only the idea of the class with natural objects. (自然について)
>>★In the one case★, the external appearance or physical structure is the least thing to be attended to;
>>●in the other●, it is everything.
=============

最後のこの数行だけを読むと、

in the one case = the idea of the individual with man のこと
in the other = the idea of the class with natural objects のこと

というふうに解釈したくなる。

そのことはその前、つまり今回の長い一節の最初の方である次の一節を読んでも同じ。

My having been attached to any particular person does not make me feel
the same attachment to the next person that I may chance to meet; (人間について)

but, if I have once associated strong feelings of delight with the objects of natural scenery,
the tie becomes indissoluble, and I shall ever after feel the same attachment to other objects of the same sort. (自然について)

(続く)
0465The OED Loves Me Not (ワッチョイ 8689-hn8B)
垢版 |
2023/01/14(土) 06:00:16.53ID:1OvUoQN20
>>461 への回答の続き

つまり、この長い一節の前半でも後半でも一貫して、人間のことを述べたあと、自然について述べている。そして
in the one case は「前者においては」という意味で、in the other は「後者においては」という意味なので、
当然のことながら
in the one case = 人間のこと
in the other = 自然のこと
だということになるので、質問者さんが正しい。

他の人はもしかして
in the one case を in one case つまり「この二つのうちの一方において」という意味だと勘違いして、
自分が勝手にこれは自然のことだろうと思ってそう解釈してしまったのかもしれない。

====

この質問は、とてもよかった。こういうまともな質問が、5年くらい前まではたくさん
出され、それに対する回答も充実していた。最近の質問と回答は、日本人の中学2年生レベルのものが圧倒的。
0466The OED Loves Me Not (ワッチョイ 8689-hn8B)
垢版 |
2023/01/14(土) 06:04:01.18ID:1OvUoQN20
>>461
さらには、この下の文言を読んでいても、

>>>>★In the one case★, the external appearance or physical structure is the least thing to be attended to;
>>●in the other●, it is everything.

「前者では外見や形はほとんど問題にならないが、後者では外見がすべて」と書いている。
これを見ただけでも、前者が人間で後者が自然だと理解できる。
0467The OED Loves Me Not (ワッチョイ 8689-hn8B)
垢版 |
2023/01/14(土) 06:59:02.58ID:1OvUoQN20
>>461
念のために、この長い一節をいくつもの部分に分けて、その概略を説明しておこう。
真面目に読みさえすれば、誰でも質問者が正しいのだと納得できるはず。
この下に示す日本語での解説は、逐語訳ではなくてあくまで概略を日本語で示しただけ。

My having been attached to any particular person
私は、誰か一人に愛着を感じても

does not make me feel the same attachment to the next person that I may chance to meet;
次にたまたま出会った人に同じような愛着を示すわけではない。

but, if I have once associated strong feelings of delight with the objects of natural scenery,
でも、いったん私が自然の景観に強い歓びを感じると

the tie becomes indissoluble,
そのつながりはいつまでも続き、

and I shall ever after feel the same attachment to other objects of the same sort.
そのあとはずっと、同じ種類のものに対しては、同じような愛着を感じるのだ。

I remember when I was abroad,
外国に行ったとき、

(続く)
0468The OED Loves Me Not (ワッチョイ 8689-hn8B)
垢版 |
2023/01/14(土) 06:59:44.07ID:1OvUoQN20
>>461 続き

the trees, and grass, and wet leaves, rustling in the walks of the Thuilleries,
樹木や草や濡れた木の葉がチュイルリー公園(テュイルリー宮殿)の散歩道でカサカサと音を立てると、

seemed to be as much English, to be as much the same trees and grass, that I had always been used to,
そのイングランド的な雰囲気と、私がかつて常に慣れ親しんでいたのと同じ樹木や草と同じもののように感じられたのだが、

as the sun shining over my head was the same sun which I saw in England;
それは、頭上で輝く太陽がイングランドで私が見たのと同じものだったのと同じくらいイングランド的であるように思われた。

the faces only were foreign to me.
人の顔だけは、私には見慣れぬものだった。

Whence comes this difference?
この違いは、どこから来るのか?

(続く)
0469The OED Loves Me Not (ワッチョイ 8689-hn8B)
垢版 |
2023/01/14(土) 07:00:16.42ID:1OvUoQN20
>>461 続き

It arises from our always imperceptibly connecting the idea of the individual with man,
それは、私たちが常に気づかないうちに個々のものを人類と結び付けて考え、

and only the idea of the class with natural objects.
類型的なものだけは自然の事物と結び付けることから由来するのだ。

In the one case, the external appearance or physical structure
前者では、外見や物理的な構造は

is the least thing to be attended to;
あまり気に留まらないが、

in the other, it is everything.
後者の場合は、そういう外見がすべてなのだ。

(終わり)
0472The OED Loves Me Not (ワッチョイ 8689-hn8B)
垢版 |
2023/01/14(土) 07:20:29.43ID:1OvUoQN20
>>461 さん、よかったら教えてほしいんだが、

(1) この英文は誰が書いたどういう本からの引用なのかな?
(2) そしてこの本を、質問者さんともう一人の人と2人で一緒に読んでいたのかな?
(3) あるいは大学か何かの講義で取り上げられた一節なのかな?
(4) もし大学か何かで取り上げられたのなら、どういうタイトルのテキストに載っていたのかな?

ネットで検索しても、この一節の出典がどうしても僕には見つけ出せなかったから、知りたいのだ。
0476名無しさん@英語勉強中 (ワッチョイ cae2-KxRG)
垢版 |
2023/01/14(土) 21:16:19.63ID:V9VnDgBN0
>>475
YOUNEEDTOBUYADRINKFIRSTがwifiパスワード。
0481The OED Loves Me Not (ワッチョイ 8689-hn8B)
垢版 |
2023/01/15(日) 04:13:12.32ID:9YmDPQ910
>>479
たとえば
You should have been more careful.
という英文を読むとき、
「~した方がよかったのに」
というふうな意味だと思えばそれでいいんだろうけど、
それ以上に難しく考える必要があるんだろうか?
0482名無しさん@英語勉強中 (ワッチョイ ed9d-TTdX)
垢版 |
2023/01/15(日) 08:15:02.89ID:BwalZcJF0
should have 過去分詞というのは話し手の個人的見解を言っているな。
~しなかったけどすべきだった、という見解。
I should have beenと自分のことをいうときは後悔の念が感じられる。
0483😉三年英太郎🌈 ◆3CZBjOt3.Y (ワッチョイW ca86-S8YK)
垢版 |
2023/01/15(日) 11:37:39.84ID:5JxgksG+0
元々は
shall - should
のように should 自体が過去形だったけど、should が過去ではない意味を持ち始めた時に、じゃあその過去も必要だといって should have -en が出来たんじゃない🤔
その他の法助動詞も同じ
0484😉三年英太郎🌈 ◆3CZBjOt3.Y (ワッチョイW ca86-S8YK)
垢版 |
2023/01/15(日) 11:39:41.72ID:5JxgksG+0
いつも思うのだが、言語学をやってるというある人が、「それ以上の考える必要がない」とか言うのだが、「それ以上を考える」のが学問じゃなくって?🤔

この人は文法用語が苦手で嫌いなんだそうだけど、「文法用語が嫌いな人のための言語学」というものはなくない?🤔

現象をカテゴライズしてラベルを貼るのが学問の第一歩でしょ?🤔
0485名無しさん@英語勉強中 (JP 0Hae-Txcj)
垢版 |
2023/01/15(日) 11:49:59.04ID:k5RQjjwUH
初心者用テキストの英文です。

母親におつかいを頼まれた子どもが「おつりをもらってもいい?」と聞くと、母は

As long as you don't spend it on chocolate. と答えました。

as long as というのは、「~の限り」と習った覚えがありますが、この文はどういう用法なのでしょうか?
これ、おつりは好きにしていい、という意味も含んでますか?
0487エキセントリックなソフィスト ◆BWpafMA3P8me (ワッチョイ 5996-w88e)
垢版 |
2023/01/15(日) 14:05:32.32ID:b293oufL0
>>485
「~する限り」とは、「~しさえすれば」と言い換えられる場合がある。
よって、As long as you don't spend it on chocolate.は、「お釣りでチョコーレートを買いさえしなければ(、お釣りを好きに使ってよいよ)」の意。

余談:シュークリームならいいのか? もっと脂質と糖分が多いぞ (^O^)
0488😉三年英太郎🌈 ◆3CZBjOt3.Y (オッペケ Sr6d-S8YK)
垢版 |
2023/01/15(日) 14:25:09.76ID:0Xf9aAyRr
言葉は実用が全てというスタンスなら私は批判しないが、読めもしないなんちゃら語を並べたてたり、屁理屈の極みである哲学とやらはありがたかったりする人には特大ブーメランとして帰ってくるだろう
0489The OED Loves Me Not (ワッチョイ 8689-hn8B)
垢版 |
2023/01/15(日) 17:28:50.45ID:9YmDPQ910
>>485
CHILD: Csn I keep the change?

MOTHER:
(1) Yes, you may -- as long as you don't spend it on chocolate.

または

(2) Yes, you may. As long as you don't spend it on chocolate, that is.

上の母親の台詞のうち、Yes, you may が省略されただけ。
なお、母親の台詞として you can でもいいけど、子供になるべく正しい英語を
覚えさせようとする親は昔からこういうときに can ではなく may を
使えと教えると聞いているから、そのように書いた。
0491名無しさん@英語勉強中 (スップ Sdca-jsA2)
垢版 |
2023/01/15(日) 20:26:22.73ID:oi7XzmZLd
スクリプトがShe really likes to date different men.という文のリスニング音声を聞くと、シビリライクス~と聞こえます。ヒビの音はどこから来ているのでしょうか?
0493The OED Loves Me Not (ワッチョイ 8689-hn8B)
垢版 |
2023/01/16(月) 07:07:29.43ID:uWmYK+kf0
質問
Who invented television? It is not an easy question to answer. In the early years of the 20th century,
there was something called a mechanical television system, but it was not a success.
Inventors were also competing to develop an electronic television system, which later became
the basis of what we have today. In the US, there was a battle over the patent for the electronic television system,
which attracted people’s attention because it was between a young man and a giant corporation.
This patent ★would★ give the inventor the official right to be the only person to develop, use, or sell the system.
0494The OED Loves Me Not (ワッチョイ 8689-hn8B)
垢版 |
2023/01/16(月) 07:16:19.01ID:uWmYK+kf0
>>493 に示したのは、みんながこよなく愛する大学受験問題のごく一部だそうだ。
最後の文に僕が★の印をつけた would の意味と用法について解説してほしい。
答えるときには、その根拠を示すために証拠らしきものも見せてほしい。

なお、どうせこれを検索したら出てくるので、あらかじめ種明かしをすると、
この would の意味と用法については、別の人が別のサイトにて
質問し、それに対して僕と別の回答者との2人がすでに回答している。
https://okwave.jp/qa/q10092179.html
0496名無しさん@英語勉強中 (ワッチョイW 1a8a-4D7Z)
垢版 |
2023/01/16(月) 08:32:33.39ID:KadnNrSu0
>>493
People thought が本来ある的な?
根拠の証拠と言われると困るけど、それ以外に思いつく意味だとなんか意味が変だから
0497The OED Loves Me Not (ワッチョイ 8689-hn8B)
垢版 |
2023/01/16(月) 09:49:34.37ID:uWmYK+kf0
>>493
うっかりしていたが、試験に出たこの一節は、もっと長い文章の一部だ。
残りの文章も含めた全文は、
https://mmsankosho.com/post-17389/5/
このリンク先で読めることに、たった今、気づいた。しかし僕のもともと
持っていたこの would の意味合いについての考えは、全文を読んだあとも
変わらない。やはり最初のほんの数行を読んだだけで、全体が予測できるというものだ。

最初に取り上げた5行ほどの一節のあとで、この発明家の名前が Farnsworth という
名前であって、いろいろ苦心したあと、ついに特許を獲得した、というふうに
次のように書いている。

The US government gave him a patent for this system in 1930.
0498The OED Loves Me Not (ワッチョイ 8689-hn8B)
垢版 |
2023/01/16(月) 10:02:02.96ID:uWmYK+kf0
というわけで、課題文である

(1) This patent ★would★ give the inventor the official right to be the only person to develop, use, or sell the system.

という一節のずっとあとで、

(2) The US government ●gave him a patent● for this system in 1930.

と書いている。ということは、(1) の would に関連して、
People thought that this patent ●would● give the inventor the officialk right....
というふうには解釈できない。通常、英語では
People (または we, you, I など) that [something or someone] would [do something].
と言った場合には、その人(people など)が「そうなるだろうな」と思ったけど、
そのあとに実はそうじゃなかったということがわかる、という文脈で使われる。
0499The OED Loves Me Not (ワッチョイ 8689-hn8B)
垢版 |
2023/01/16(月) 10:02:13.77ID:uWmYK+kf0
じゃあ、
(1-a) This patent ★was to★ give the inventor the official right....
と書いたらどうなるか。それならば、(1) と (1--a) が同じような意味になるだろうと思う。
というわけで、やはり最初に僕が OK Wave にて解説した通り、この would は
(文法学者がどういう文法用語を使って解説しているかは知らないが)
言ってみれば、筆者(および読者など、一般の人々が)過去の時点
(つまりその発明家に特許が下る前)に戻って、その過去の時点から見て未来
(つまり特許が下った時点)を見通しているからこそ、いわば「過去から見た未来」という
わけで would になっているのだ」ということになる。

間違っているだろうか?僕から見れば、これはごく当たり前のことにしか見えない。
もちろんこれを時制の一致だとかいうふうにも説明できるだろう。でもこれを
時制の一致と言って見ても、結局は同じことを言っているに過ぎない。

それはともかく、この would をたとえば
The door would not open. (どうしても開かなかった)
の would と同じだとか、あるいは「おそらく~だろう」というふうな推量だとかいう
いい加減な回答をしておしまいにし、質問者たちはいつもこういうときに、簡単に納得してしまっている。
0500The OED Loves Me Not (ワッチョイ 8689-hn8B)
垢版 |
2023/01/16(月) 10:08:28.18ID:uWmYK+kf0
たいていの文法事項は、一つの短い文だけを読み取りさえすれば
解決できる。だからこそ、大多数の日本人は、そして文法の大好きな
日本人たちは、文法解説にいそしむ。文法書を読み、辞書を引き、
日本語で書かれた参考書さえ読めば、あとはときどき新聞を
拾い読みしさえすれば英語が身に付くと勘違いする。

しかし今回の would は、文脈、しかも2行か3行の文脈ではなく、
10行も20行も読み続けなければその would の意味合いが読み取れない。

僕が OK Wave にて寄せた回答に出てくる Barack Obaha による
広島での演説の中に出て来た would の使い方も同じだ。
この演説の1文だけを読んでも、その would の本当の意味合いは
つかみにくい。しかし30行くらい読み続ければ、全体の文脈の中での
would の意味合いがはっきりと浮き出てくる。

文法用語だけ振り回して文脈を読まない連中を僕が揶揄するのは、
そういうことを踏まえた上で言っている。

極端に言えば、一つの単語の使われ方、その文法的な意味合いや
語法は、その言葉の出てくる本の全体、つまり300ページくらいをすべて
読み切った上でないとわからない。
0503The OED Loves Me Not (ワッチョイ 8689-hn8B)
垢版 |
2023/01/16(月) 10:26:11.30ID:uWmYK+kf0
>>498 にて、試験に出た原文は
★People thought that★ this patent would give the inventor...
という意味ではないと書いた。それでは、
★People were certain that★ this patent would...
ならどうか、と疑問に思う人もいるかもしれない。それも間違い。なぜなら

At the time people were certain that this patent would give the inventor the official right to....
But later on, it became known that this was not the case.

なんていう文脈もざらにあるからだ。

しかし今回の文脈における課題文である
This patent ★would★ give the inventor....
での would は、あとになって必ずこれが実現する、ということを示している。だからこそ、あとで
The US government gave him a patent for this system in 1930.
とはっきり書いているのだ。
0504エキセントリックなソフィスト ◆BWpafMA3P8me (ワッチョイ 5996-w88e)
垢版 |
2023/01/16(月) 10:46:02.89ID:uSz1ytBu0
>>503
>あとになって必ずこれが実現する
  ↑
これは言い過ぎ。
★Dropping the bomb would kill approximately 200,000 Japanese and bring forth massive destruction to the city.★
広島・長崎の原爆での死者数は、16万人程度。「約20万」は実現していない。あくまでも、「過去における潜在的未来」の意味。
0505エキセントリックなソフィスト ◆BWpafMA3P8me (ワッチョイ 5996-w88e)
垢版 |
2023/01/16(月) 10:54:55.48ID:uSz1ytBu0
訂正

(×) 広島・長崎の原爆での死者数は、16万人程度。
(〇) 広島・長崎の原爆での死者数は、それぞれ16万人程度。

いずれにしても、「約20万人」ではなく、約「15万人」が正確な表現。
このように、「約20万人」は「必ずこれが実現する」という意味で使われてはいない。あくまでも、当時の潜在的未来であって、現在実現していなくても使える。
0507The OED Loves Me Not (ワッチョイ 8689-hn8B)
垢版 |
2023/01/16(月) 12:15:02.70ID:uWmYK+kf0
>>506
> formulate 〜を策定する、formalize 〜を正式なものとする の二つがゴッチャになるのですが、ゴロでもなんでもよいので両者の意味の見分け方を教えてください

formulate は、formula (公式、製法、方法、解決策) から派生した単語だとしたら、
「解決策を設定する」のだから「策定する」。あるいは、例のレーシングカーである
Formula One (F1) をどのように改造・運用して次の試合で勝つかという
方策を「策定する」という意味だというふうに、こじつけてもいい。

formalize は、formal (フォーマルな、正式な) なものにする (-izw) わけだから「正式なものとする」。
「フォーマルな」服を着てこい、と言われたら、正式な礼服を着ていくだろ?
カジュアルな服じゃ、ダメってことだ。
0508名無しさん@英語勉強中 (アウアウウー Sa91-llgr)
垢版 |
2023/01/16(月) 12:42:19.63ID:N3cxDCHOa
>>491
低倍速で聴いたら解決しませんか?
想像ですがsheの強い擦過音に引きずられてrが消えているように聞こえていると思います。
0509ブタ耳 (ワントンキン MM45-HT4l)
垢版 |
2023/01/16(月) 13:57:45.50ID:glOb94YkM
>>497
リンク先のテキスト読んでみました。
wouldの解釈はまったくその通りだと思いますが、そこで1930年の特許を論拠に持ち出すのはちょっと変です。

裁判の流れを整理すると、
① 1930: Fransworth、特許獲得
② RCA、1923年に特許を得ていた
③ 1931: RCA、Fransworthの特許を買い取ろうとして失敗
④ 裁判開始
⑤ 1934 Fransworth勝利、特許獲得確定

件の“would” は、④の裁判が開催中(there was a battle over the patent)で、誰が特許を獲得するか分からない時点を基準にしています。なのでここで①を持ち出すのは不適切です。
0511名無しさん@英語勉強中 (ワッチョイ 9589-wtyD)
垢版 |
2023/01/16(月) 14:42:10.98ID:qqoMnELp0
6問以外 かるく飛ばし読みで読んでみたけど
実用的な英語学習を促進する目的でこんな問題つくってるんだろうか

古文のような解釈を問う問題もないし 量が多いし 
あまり知られていない語句の注釈ないし 
精読よりも だいたいの読みでたくさん読ませようとするわけだから受験生の読解力は低下してそう

設問も第六問の 
48の答え 3の選択肢にある they can also thrive のcanを 
可能性として解釈した場合でも 正解にならないんだろか
プレゼンの主張ってクマムシの生存能力が高いってことだろうし

49は解答が4になるにしても 
選択肢なく この資料だけから そんな推測できそうには思えない
0512名無しさん@英語勉強中 (ワッチョイ 9589-wtyD)
垢版 |
2023/01/16(月) 15:08:43.05ID:qqoMnELp0
竹岡の説明
https://www.youtube.com/watch?v=WHqMGH6raLE

いいすぎでだめっていってるけど
それは they can also thrive のcanを 能力 として解釈するからだめなわけで
Perhaps even more amazing than their ability to survive on earth is their ability to survive in space.
って書いてるし
7割弱が 10日間 地表より1000倍強いX線や紫外線に耐えられるって書いてるんだから
月で生息できる可能性は 否定されてないように思える

4が正答としても
急に人類との比較を プレゼンの結論にもってくるのは 唐突に感じる
0513ブタ耳 (ワントンキン MM45-HT4l)
垢版 |
2023/01/16(月) 15:08:54.72ID:glOb94YkM
リンク先の問題読んでみた。クマムシかわいいよクマムシ。

48は、プレゼンの最後のスライドとしてふさわしいものを選ぶ問題。
④には発表者自身の推測が含まれているが、最後のスライドで発表者が自分の感想や今後の展望を述べることはよくある。
③の選択肢は、2枚目のスライドの内容と重なるので冗長な上に、月の上で生存可能かどうかは未確認なので最後の”they can also thrive on the moon”が不適切。
0514名無しさん@英語勉強中 (ワッチョイ 9589-wtyD)
垢版 |
2023/01/16(月) 15:14:55.62ID:qqoMnELp0
主張を最初に述べて 結論でも繰り返すのが ひとつのパターン
というか 典型パターン

>月の上で生存可能かどうかは未確認なので最後の”they can also thrive on the moon”が不適切。

不適切な理由がない
can を 可能性と解釈したら なんでだめなん?生存可能性は否定されてない
0515名無しさん@英語勉強中 (ワッチョイ 21f0-CsaQ)
垢版 |
2023/01/16(月) 15:16:55.63ID:I1rbDeGz0
>>507 ありがとうございます。
てっきり同じ単語のグループだと思ったのですが、
formula と formal で
違う単語のグループだったのですね。
勉強になりました。
0516名無しさん@英語勉強中 (ワッチョイ 9589-wtyD)
垢版 |
2023/01/16(月) 15:21:12.04ID:qqoMnELp0
they can also thrive on the moon
さらに クマムシは 月でも生育する可能性がある
って解釈して でなにがだめなんだ
0517ブタ耳 (ワントンキン MM45-HT4l)
垢版 |
2023/01/16(月) 15:22:31.28ID:glOb94YkM
>>514
プレゼンの最後スライドに③見せられて、
they can also thrive on the moon
と書いてあったら、
「ああクマムシは月でも生きられるんだ、スゲーなー」と普通考えますよ。
0518名無しさん@英語勉強中 (ワッチョイ 9589-wtyD)
垢版 |
2023/01/16(月) 15:26:38.73ID:qqoMnELp0
プレゼンとしても 主張のくり返しが自然

プレゼンの締めでよく使われる表現をご紹介する。
自分の一番伝えたかったことを繰り返すことで聞き手に強く印象付けよう。終了を明確に示すことも忘れずに。
https://english-club.jp/blog/business-english-presentation/#4

スピーチは以下の構成で行うと良いです。

導入(トピックの主題「○○について話します」)
本論(主題をサポートする根拠「○○の理由は3つあります….1,..2,..3…」)
結論(本論の内容を軽くまとめて、主題をもう一度言い換えて伝える)
https://www.fourskills.jp/prepare-for-ielts-speaking#23

「結論(Conclusion)」とは、ライティングにおけるまとめの部分で、これまでに述べてきた主張の再提示、重要なアイデアの要約などを行います。
NGな結論 新しい情報を述べる
https://solo-ielts-toefl.com/ibt-writing-conclusion-paragraph/
0519名無しさん@英語勉強中 (ワッチョイ 9589-wtyD)
垢版 |
2023/01/16(月) 15:28:56.08ID:qqoMnELp0
>>517
解釈が 客観的に1つに限定されるなら それでいいけど
この場合 自分には 解釈が限定されてるようには思えないな
0520ブタ耳 (ワントンキン MM45-HT4l)
垢版 |
2023/01/16(月) 15:38:15.21ID:glOb94YkM
>>518
月はさておき、まとめだとしても、③は中途半端に詳し過ぎるんですよ。
山頂だとか海底だとか温泉だとか、何で今更そんなこと言うの? 話が振出しに戻ってません?
って感じませんか?

で、月に戻るけど、「山頂でも海底でも温泉でも生存できる」の後に“they can also thrive on the moon” と来たら、「ああ月でも生存できるのね」と普通は考えます。
0521名無しさん@英語勉強中 (ワッチョイ 9589-wtyD)
垢版 |
2023/01/16(月) 15:46:56.08ID:qqoMnELp0
IELTSのスピーチでも 結論で 繰り返すのは典型だし(>>518
急に人類との比較を主張してくるのは 唐突に思える

試験問題なら 客観的に 一定の英文解釈能力ある人なら全員が
正答を選べる問題にすべきだとおもう
0522名無しさん@英語勉強中 (ワッチョイ 9589-wtyD)
垢版 |
2023/01/16(月) 15:52:18.19ID:qqoMnELp0
しかも 2のスライドに
in space (possibly) って書いてるやん
0523名無しさん@英語勉強中 (ワッチョイ 9589-wtyD)
垢版 |
2023/01/16(月) 16:00:45.03ID:qqoMnELp0
3 のスライドにも
They have an ability to withstand extreme levels of radiation.
が書かれている と考えられるし
結論で まとめ的に クマムシがultimate survivorsって主張を繰り返すのは
別に変な感じしないけどなあ

予備校の解答は 4だけど・・
0524ブタ耳 (ワントンキン MM45-HT4l)
垢版 |
2023/01/16(月) 16:01:59.75ID:glOb94YkM
>>521
> 急に人類との比較を主張してくるのは 唐突に思える

この問題に問題があるとすればそこですね。
特に、発表者が当該の記事を作成した科学者だったとすればかなり問題があります。記事の中でも書いとけよ、という話です。

しかし、今回の場合、「学生の科学プレゼンコンテスト」という初期設定を考慮すべきでしょう。学生たちが第3者の立場から、独自の感想を最後に「付け足して」述べても構わないのでは?
0525名無しさん@英語勉強中 (ワッチョイ 9589-wtyD)
垢版 |
2023/01/16(月) 16:04:26.32ID:qqoMnELp0
まあ たしかに実際のプレゼンなら全然ありうると思う
0526名無しさん@英語勉強中 (ワッチョイ 09e2-rGbT)
垢版 |
2023/01/16(月) 16:15:51.69ID:pdXfcSPs0
宇宙では10日間で68%生き延びた(=32%は死滅)との言及の後に、月面に残されたクマムシのその後はわからんと書いてある流れで、「can also thrive on the moon」はないでしょ。
0527名無しさん@英語勉強中 (ワッチョイ 9589-wtyD)
垢版 |
2023/01/16(月) 16:24:23.39ID:qqoMnELp0
でも 2のスライドに
in space (possibly) って書いてるし
thrive on the moon の可能性って否定されてないやん
そりゃ 生存できるって主張ならだめだけど
0530名無しさん@英語勉強中 (ワッチョイ 4af0-f6s+)
垢版 |
2023/01/16(月) 16:31:32.45ID:knHy0/FE0
月と宇宙が同じと思う人いるね
0532名無しさん@英語勉強中 (ワッチョイ 9589-wtyD)
垢版 |
2023/01/16(月) 16:34:33.73ID:qqoMnELp0
>>528
資料にはこう書いてる
This means that for 10 days
most ←
were able to survive X-rays and ultraviolet radiation 1,000 times more intense than here on earth.
0533名無しさん@英語勉強中 (アウアウウーT Sa91-JXbu)
垢版 |
2023/01/16(月) 16:39:29.61ID:gscXE3k/a
thriveなんてシス単にもターゲットにも載っている
0534名無しさん@英語勉強中 (ワッチョイ 9589-wtyD)
垢版 |
2023/01/16(月) 16:42:35.57ID:qqoMnELp0
プレゼンする人が without a doubt って言ったらだめとか聞いたことない

クマムシの生存能力がすごいって主張のプレゼンなのに
人類よりもoutlive する可能性があるって主張を 結論にもってくるのは奇妙すぎる

それなら それまでに クマムシの生存能力と人間の生存能力の比較すべき
0536名無しさん@英語勉強中 (ワッチョイ 9589-wtyD)
垢版 |
2023/01/16(月) 16:55:50.57ID:qqoMnELp0
トピック クマムシ VS 人類 (クマムシって人類より生存能力高くね?)
理由1 クマムシは高山に生存できる  人間は生存できない
理由2 クマムシは深い海に生存できる 人間はできない
理由3 クマムシは放射線に耐えられる 人間は耐えられない
結論 クマムシは人類より長く生息する可能性がある
とかならわかる
0537名無しさん@英語勉強中 (ワッチョイ 9589-wtyD)
垢版 |
2023/01/16(月) 16:56:38.44ID:qqoMnELp0
>>535
4の「正答」のほうが非論理的に見える
0538名無しさん@英語勉強中 (ワッチョイ 9589-wtyD)
垢版 |
2023/01/16(月) 16:59:39.00ID:qqoMnELp0
私見とか主観的判断いれていいなら
without a doiubyっていってもええやろ
0539名無しさん@英語勉強中 (ワッチョイ 9589-wtyD)
垢版 |
2023/01/16(月) 17:00:32.14ID:qqoMnELp0
>>538
without a doubt
0540エキセントリックなソフィスト ◆BWpafMA3P8me (ワッチョイ 5996-w88e)
垢版 |
2023/01/16(月) 17:06:49.19ID:uSz1ytBu0
根拠のない主観的推測なら、「~かもしれない」を付けないと信頼は得られないよ。「かもしれない」を付けないと「こいつのプレゼンは信用ならない」という評判が立つ。

また、地球の最後は、太陽が膨張し「赤色巨星」になって、その熱で生物が死に絶える可能性が高い。となれば、高温でも生きていけるクマムシは人間より長生きすることは確かだ。
0542名無しさん@英語勉強中 (ワッチョイ 9589-wtyD)
垢版 |
2023/01/16(月) 19:07:53.63ID:qqoMnELp0
>>540
理由示すときに 人間の生存能力との比較せずに
いきなり クマムシが人間より長生きできるって主張するのは変に感じる
トピックのメインアイディアからはずれてる印象を受ける
メインアイディアはあくまでもクマムシの生存能力の高さ ふつうは結論でこの主張を繰りかえすもんだとおもうけど

この予備校の先生も 未確認だからだめっていってて
能力を示すcanつかってるなら わかるんだけど 可能性のcan で解釈してだめな理由がよくわからない
https://www.youtube.com/watch?v=gZ0Mig12cAg
0543名無しさん@英語勉強中 (ワッチョイ 9589-wtyD)
垢版 |
2023/01/16(月) 19:22:53.65ID:qqoMnELp0
もりてつも同じ
https://www.youtube.com/watch?v=QA1bAbF_QKY

can って論理的可能性らしいけど
(「それ自身がもつ根拠(実績や要素など)に基づいた可能性」、論理的可能性を表現する時)
She can pass the examination. 
「彼女は(成績も良いので)試験に合格できるだろう」と理由を基にした理論上の可能性を表します。
Do you have some coins?「コインありますか」
I may[×can] have some.「たぶんあります」
理論上の可能性は問題にしていないのでここでは can は使いません。
https://kou.benesse.co.jp/nigate/english/a13e0703.html

極地に強い+放射線に強い(根拠) 

宇宙外でも生存可能かも 月でも生存可能かも(論理的可能性)
なんであかんねん
0544名無しさん@英語勉強中 (ワッチョイ 9589-wtyD)
垢版 |
2023/01/16(月) 19:32:14.24ID:qqoMnELp0
>>543
宇宙外って変だな 地球外だったわ
0546名無しさん@英語勉強中 (ワッチョイ 9589-wtyD)
垢版 |
2023/01/16(月) 19:40:09.14ID:qqoMnELp0
こういうのガチでさらしてくれるの助けるわ

【共通テスト英語2023】英検1級満点が最速で全問正解しようとした結果...
https://www.youtube.com/watch?v=XwvZw-6Bsr4
0547名無しさん@英語勉強中 (ワッチョイ 9589-wtyD)
垢版 |
2023/01/16(月) 19:53:36.39ID:qqoMnELp0
>>545
主張してはいけないといはいってない
ふつうは 主張を繰り返す それが典型的パターン(>>518
英検のスピーチでも  受験対策したことある人ならしってるスタイル

英語のプレゼンテーション〈スキルアップ術〉の目次
第13章 プレゼンテーションを終わらせる メッセージを再度強調する
0548名無しさん@英語勉強中 (ワッチョイ 9589-wtyD)
垢版 |
2023/01/16(月) 20:09:59.34ID:qqoMnELp0
この人も can を能力として解釈してるけど
能力で解釈しないといけない必然性ってあるんだろか
あるなら まあ これが不正解の選択肢なんだろな

【英語のトリセツ】
https://www.youtube.com/watch?v=8X1o2rhtuYQ
0549名無しさん@英語勉強中 (ワッチョイW 1a8a-4D7Z)
垢版 |
2023/01/16(月) 21:08:26.71ID:KadnNrSu0
>>510
興味持ったのでやってみた
これは4-4でしょ
0550名無しさん@英語勉強中 (ワイエディ MM72-sAzJ)
垢版 |
2023/01/16(月) 22:50:29.34ID:2EXg/ZB0M
TOEICの問題集のなかに

That, your meals, and your helicopter transportation between the site and our project offices will all be provided courtesy of Portland Projects.
それ(前に出てくる部屋のこと)やお食事、それに現場と当社のプロジェクトオフィスの往復でご利用いただくヘリコプターは、全てPortland Projects社の取り計らいでご用意させていただきます。

というセンテンスがあり、provided と courtesy の間に by か何か前置詞が必要な気がします。
文法的に理解できず、頭に入れることができません。

どなたか分かる方、よろしくお願いいたします。
0551名無しさん@英語勉強中 (ワッチョイW 1a8a-4D7Z)
垢版 |
2023/01/16(月) 23:04:44.88ID:KadnNrSu0
>>550
そのbyは省略可で、無くても良いみたいです
0552名無しさん@英語勉強中 (ワイエディ MM72-sAzJ)
垢版 |
2023/01/16(月) 23:16:49.72ID:2EXg/ZB0M
provided は受動態では無く、形容詞なのでしょうか?

provided (courtesy of Portland Projects)
供給された(Portland Projects社の好意)
0553名無しさん@英語勉強中 (ワッチョイW 1a8a-4D7Z)
垢版 |
2023/01/16(月) 23:19:35.33ID:KadnNrSu0
いや普通に受動態です
まぁ一つの慣用的な決まった言い方みたいなもんだと思いますよ
0556The OED Loves Me Not (ワッチョイ 8689-hn8B)
垢版 |
2023/01/17(火) 04:41:31.39ID:obMvYxpD0
>>554
> Adobe Premiere Pro

「とまういう物」って、typo なんだろうけど、意味がわからん。
それはともかく、Adobe 社がそれを Premiere と命名した
理由なんて、大した意味はないと思う。文字通りの意味でしかない。

premiere
the first performance of a musical or theatrical work or the first showing of a film.

premiere がもともとこういう意味だし、さらには YouTube では自分が作ったそれぞれの
動画を初めて公表するときに Premiere っていう option を選んで、その動画を
初めて公表する日時を指定して、視聴者に対して前もって

「新しい動画を作ったよ。その公開は、12時間後の夜 8:00 からだよ」

などと告示し、公開開始と同時にたくさんの subscribers たちが押し寄せてくるのを
楽しみたいと思う YouTubers たちのために便宜を図っているんだろうよ。
いわば、YouTubers たちに対して、有名人になったかのような自己満足を与えるための
投稿編集 option の一つだろうと思う。

そういう YouTube 上の option 名を、そのままこの動画ソフトの title として
Adobe Premiere と命名して発売し、それと同時に(あるいはそのあとで)
Adovbe Premiere Pro もついでに発売したんだろうと思う。

いずれにしても、Premiere なんていういかにもフランス語っぽい響きを持つ言葉を使って、
余計に YouTubers たちを自己満足させたいんだろう。英米人は何百年も前から
ラテン語と共にフランス語が大好きだからな。日本人の英語コンプと同じ。
0557The OED Loves Me Not (ワッチョイ 8689-hn8B)
垢版 |
2023/01/17(火) 04:56:25.29ID:obMvYxpD0
>>550 >>552
他の回答者さんの言う通りだけど、

(1) Courtesy of [社名].
(2) By courtesy of [社名].

という言い回しは、写真のすぐ下によくつけられている注意書きだな。さらにそれを
普通の文の中に入れて、次のように書くこともある。

(2) Photo is courtesy of [社名].
(3) The photo is courtesy of [社名].
(4) The photo is by courtesy of [社名].
(5) The photos are courtesy of [社名].
(6) All the photos are courtesy of [社名].

上のあらゆる場合に、courtesy of... の前に by をつけてもつけなくてもいい。

(7) The photos, courtesy of Manchester Libraries, show many of the traditional family run business still operating, ....
0558The OED Loves Me Not (ワッチョイ 8689-hn8B)
垢版 |
2023/01/17(火) 05:02:48.87ID:obMvYxpD0
>>550 >>552

(1) courtesy of [社名]
(2) by courtesy of [社名]

この上の二つが photos や pictures の注意書きとして
頻繁に使われているところを見ると、"courtesy of [社名]"
が元々は "by courtesy of [社名]" という形だったんだとわかる。

さらに、"is (are) provided courtesy of [社名]" というところで
provided が courtesy にかかる形容詞でなくてあくまでも受動態なんだ
ということは、

(3) is (are) provided by courtesy of [社名]
という言い回しがあちこちで出てくることから伺える。

参考:
In this photo ★provided by courtesy of★ MacArthur Justice Center,
Michael Politte smiles after being released on parole on Friday, April 22, 2022,
at the Jefferson City Correctional Center in Jefferson City, Mo.
https://www.ky3.com/2022/04/22/missouri-man-imprisoned-two-decades-mothers-death-out-parole/
0559The OED Loves Me Not (ワッチョイ 8689-hn8B)
垢版 |
2023/01/17(火) 05:10:29.10ID:obMvYxpD0
>>550 >>552
この下の英文を見た方が、わかりやすい。

(1) The map ★was provided by courtesy of★ the Earth Science and Remote Sensing Unit, NASA Johnson Space Center
https://www.nature.com/articles/s41598-022-10940-3

上のような、本来の
[The photo, map, picture など] was (または is?) provided by courtesy of [社名].
というような英文がもともと盛んに使われてきたんだろうと思う。それがいつしか
写真や絵のすぐしたに

(2) Provided by courtesy of [社名].

という簡略化した注意書き (caption) がつけられるようになり、それがさらに簡略化されて

(3) By courtesy of [社名].

となり、さらに

(4) Courtesy of [社名].

となっていったんだろうと想像すればいいんだ。
0560名無しさん@英語勉強中 (ワッチョイ 86b5-PB+X)
垢版 |
2023/01/17(火) 09:36:34.67ID:xihgZGR80
生来の話者からすると、
リズトラス(Liz Truss)

レタス(Lettuce)

語呂が似ているふうに感じるものなん?
Can Liz Truss outlast this lettuce?
というのが。英語的には、三つ似た語呂が並んでるふうなそうな。
0561名無しさん@英語勉強中 (ワイエディ MM72-sAzJ)
垢版 |
2023/01/17(火) 09:45:22.57ID:YOYmJGDjM
>>559
ありがとうございました!
教えていただいたことが辞書でも確認できました。
0562名無しさん@英語勉強中 (アウアウウー Sa91-Xlz1)
垢版 |
2023/01/17(火) 10:27:12.06ID:ixm1Zn4ta
>>556
ありがとうございます
とまいうもの は
どういうもの のタイプミスです
スマホで濁点打つつもりがミスって1個上のまをタップしただけですね

ちなみにYouTubeとAdobe Premiere Proは
Adobe Premiere Proの方が数年早くリリースはされていますが
まあ意義的にはまさに他の動画サイトとかに対してそんな意味があったんだろうなと納得しました
ありがとうございました!
0563The OED Loves Me Not (ワッチョイ 8689-hn8B)
垢版 |
2023/01/17(火) 10:31:18.30ID:obMvYxpD0
>>560
Can Liz Truss outlast this lettuce?

確かにそういわれてみれば、三つ並んでいると感じることもできるな。
この下に発音記号を書くけど、文字化けするかもしれん。

(1) Liz Truss - 'l?z 'tr?s -- ?l?ztr?s - 'l?str?s だから 'let?s に似てくる。

(2) -last this -- 'last 'd?s -- 'last d?s -- 'last?s -- だから 'let?s に似てくる。

(3) lettuce
0564The OED Loves Me Not (ワッチョイ 8689-hn8B)
垢版 |
2023/01/17(火) 10:37:00.39ID:obMvYxpD0
>>560
やっぱり文字化けしたな。仕方ないから、カタカナで。

Liz Truss -- Liztras - listras - listas -- lettuce に似てくる。
z はそのあとの t に影響されて s になりやすい。tra の r は消えて ta になりやすい。

-last this -- last dis -- lastis - lettuce に似てくる。
this の th は d になりやすい。そしsて last dis が lasdis になり、d は s に影響されて t になりやすい。
0567名無しさん@英語勉強中 (ワッチョイ b596-7DGa)
垢版 |
2023/01/17(火) 22:11:47.80ID:HtsqvnfI0
Deterioration is proceeding to serious situation.
「劣化は深刻な状況になってきている。」

と表現したいのですが、上記の英文は通じるでしょうか?
また、より自然な表現があれば、ご教示いただけると幸いです。
(※長年使っている設備の老朽化がひどくなってきた事を伝えたいです。)

よろしくお願いいたします。
0568名無しさん@英語勉強中 (ワッチョイ 4af0-f6s+)
垢版 |
2023/01/17(火) 23:10:43.84ID:+vwi/aeR0
The condition of the equipment has been severely deteriorating
0570ブタ耳 (ワッチョイW d6e3-HT4l)
垢版 |
2023/01/18(水) 00:20:29.75ID:x1nyjfnM0
The situation is getting worse due to the serious deterioration of the equipment.

というのを私は考えました。

この他にどんな言い方があるかChatGPTに尋ねた結果は別スレの方に発表します。
0571ブタ耳 (ワッチョイW d6e3-HT4l)
垢版 |
2023/01/18(水) 00:40:26.83ID:x1nyjfnM0
>>567のレスを一語一句そのままの形で質問するとどうなるかも試してみました。

こちらになります。

ChatGPTに質問してみた
https://lavender.5ch.net/test/read.cgi/english/1673484651/

このスレ、他の方もChatGPTを色々試して結果を書き込んでくれると嬉しいです。
ただしChatGPTが嫌いな人は金輪際来ないでください。
よろしくお願いします。
0572The OED Loves Me Not (ワッチョイ 8689-hn8B)
垢版 |
2023/01/18(水) 10:06:15.65ID:gd1aF/Fj0
>>567
質問者の英文原案:
>>Deterioration is proceeding to serious situation.
>>「劣化は深刻な状況になってきている。」
>>(長年使っている設備の老朽化がひどくなってきた)

>>AI robot である ChatGPT による回答の一部
>>"The deterioration is becoming increasingly severe." or "The situation is becoming increasingly dire as the deterioration progresses."

=====

上のように、AI robot はきちんと deterioration という無冠詞の名詞に定冠詞をつけて、
the deterioration としているな。まあ、英語ネイティブを初めとして、冠詞を持つ言語の
話者にとっては常識的で簡単なことなんだろうけど、冠詞を持たない日本人にとっては、
もともとの質問者が無冠詞の deterioration を使っていたように、やっぱり難しいことなんだろうな。

「劣化は~という状態になってきている」と言ったとき、ついつい日本人は主語の「劣化」を
deterioration とだけ言いたくなってくる。「スレッドを立てるまでもないスレッド」の回答者たちの
ほとんどはそれを "the deterioration of the equipment" などというふうに処理したので、of the equipment に
よって限定された derterioration には the が付くはずだと考えて the をつけたんだろうけど、
of the equipment をつけなくても、今回の場合にはやはり the deterioration となるわけだな。
0573The OED Loves Me Not (ワッチョイ 8689-hn8B)
垢版 |
2023/01/18(水) 10:06:49.66ID:gd1aF/Fj0
>>567 続き

だからたとえば
The deterioration is becoming increasingly severe.
という英文がいきなり出て来たとしても、これを単に
「劣化はますますひどくなってきている」
とだけ訳してすべて理解したつもりにならずに、きちんとこの deterioration の前についている
the がついている意味を考えて、
「(われわれが所有している設備・施設・システムなどの)劣化」という意味で the deterioration という
ふうに the がついているんだな、と理解しないといけないというわけだ。

もしも主語を The deterioration ではなくて Deterioration という無冠詞の名詞にした場合には、
「世の中で一般的に見られる劣化と言うものは、これこれこういうものである」という一般論を述べていることになる。
0574The OED Loves Me Not (ワッチョイ 8689-hn8B)
垢版 |
2023/01/18(水) 10:17:00.54ID:gd1aF/Fj0
>>567
たとえば Love is blind. (愛は盲目) という言葉があるけど、その一方で

The love is felt, & extremely real. No matter race or religion everybody coming together in prayer! ????
? ?????????? ???????????? (@HamlinIsland) January 8, 2023

https://www.cbsnews.com/news/damar-hamlin-first-public-comments-cardiac-arrest-buffalo-bills/

上のような言葉もネット上のニュースで飛び交っている。ここでもしも
Love is felt.
というふうに the なしで書いたら、それは
「愛と言うものは、(一般的に言って)感じられるものだ」
という意味になってしまう。しかしここでスポーツ選手が言っているのは
the をつけた love であって、断じて冠詞抜きの love ではない。だから、
「ファンの皆さんやそれ以外の関係者たちからの僕に対する love(つまり「温かいご支援」)
という意味で定冠詞付きの the love となっていることがわかる。
0575名無しさん@英語勉強中 (ワッチョイ cdd1-+ite)
垢版 |
2023/01/18(水) 12:34:11.80ID:TlB/t6V80
しょーもない質問でしょうがお願いします

not valid
invalid

の違い、使い分けとかありますか?
シチュエーションとしては、この項目/書類は使えないよ、
くらいの簡単な内容なんだけど
0576The OED Loves Me Not (ワッチョイ 8689-hn8B)
垢版 |
2023/01/18(水) 13:08:20.37ID:gd1aF/Fj0
>>575
(1) invalid とは
An invalid document, ticket, law, etc is not legally or officially acceptable.
https://dictionary.cambridge.org/dictionary/learner-english/invalid

(2) valid とは
A valid document is legally acceptable, usually because it has been done according to official rules:
You must have a valid driver’s license to drive a car.
https://dictionary.cambridge.org/dictionary/english/valid

上の (1), (2) を見ると、おそらくは valid と invalid とは反対語であり、ゆえに invalid とは not valid のことであり、
この二つは同じような意味だと考えていいだろうな、という気がしてくる。その上で、次のような用例を見てみる。

(続く)
0577The OED Loves Me Not (ワッチョイ 8689-hn8B)
垢版 |
2023/01/18(水) 13:10:20.23ID:gd1aF/Fj0
>>575 続き

How to ★Authenticate★ an Electronic Transcript
The electronic version of the official transcript of the University of Chicago contains a digital signature that can be instantly ●validated●.
The document will display a blue ribbon symbol as assurance that the digital signature is ◆valid, authentic,◆ and the contents of the document have not been altered.

When the document is viewed through Adobe Reader versions 6.0 or higher, or through Adobe Acrobat versions 6.04 or higher,
it will reveal that a digital signature has been applied to the document. A pop-up screen will indicate
whether the document is ★valid or invalid★, or whether the signer of the document is unknown and other measures must be taken to authenticate it.
Ihttps://registrar.uchicago.edu/records/transcripts/etranscript-authentication/

★や●などの印をつけた言葉に注意しながら上のような文章を読んでいると、どうやら
not valid と invalid とを同じような意味として「(書類などが)使える、使えない」という文脈で
使えそうだという気がしてこないだろうか?
0578名無しさん@英語勉強中 (ワッチョイ cdd1-+ite)
垢版 |
2023/01/18(水) 14:54:42.16ID:TlB/t6V80
>>576 577
ありがとうございます
どちらも同じ意味で使える上で、

「Document is not valid」
「Document is invalid」

どう使い分けるのか、
英語話者の使い分け、あるいは受け取り方に違いあるのかな、と
0580名無しさん@英語勉強中 (ワッチョイ cdd1-+ite)
垢版 |
2023/01/18(水) 16:02:56.86ID:TlB/t6V80
>>579
リンク先、123以降も含めとても参考になりました
ありがとうございます
0581名無しさん@英語勉強中 (ワッチョイ b596-7DGa)
垢版 |
2023/01/18(水) 20:34:35.87ID:EvOvqy6O0
>>568-574
多くのご回答をありがとうございます。
たった一つの文に、多くの訳文例をいただき、とても参考になります。
また、ChatGPTというものも今回初めて知りました。
自分の書いた英文の添削や確認に使えるとなると、大変便利ですね。

定冠詞のご指摘はまったくおっしゃる通りです。
「我々の」設備が老朽化しているわけですから、theが付く道理ですね。
冠詞についても、もっと意識していきたいと思います。

ありがとうございました。
0582名無しさん@英語勉強中 (ワッチョイ 4132-60iq)
垢版 |
2023/01/19(木) 00:59:12.47ID:jMmTMAz30
He stopped to smoke.
この「to不定詞の副詞的用法」を「目的」ではなく「結果」として
解釈してもかまわなでしょうか?
0583The OED Loves Me Not (ワッチョイ 8689-hn8B)
垢版 |
2023/01/19(木) 08:07:55.67ID:3O8X3TRY0
>>582
文脈による。しかしたいていは「結果」として解釈していいんだ。

https://www.italki.com/en/post/question-180432

上のリンク先を読めばわかるけど、
Where as, "I stopped to smoke" means that you were going somewhere and stopped, so that you could smoke. For example, "I stopped to smoke on my way back home."

というふうに解釈するのが普通だ。しかし、

I stopped to smoke but realized I had stopped smoking. :P

こんなふうに冗談半分で書いている人もいるが、こういう文ももありうるかもしれない。
しかしやはり

"I stopped to smoke" is when you are doing something, then you stop doing that and pull out a cigarette and light it and start smoking it.

こんなふうに解釈するのが普通だ、と別の人がまた説明しなおしている。
0584The OED Loves Me Not (ワッチョイ 8689-hn8B)
垢版 |
2023/01/19(木) 08:13:01.68ID:3O8X3TRY0
>>582
>>So, it would be theoretically possible to say “I stopped to smoke, but I didn’t have any matches so I continued on my way”.
https://scottthornbury.wordpress.com/2010/06/06/g-is-for-gerund/

上のように書いている人もいる。ともかく、stop to verb ははもともと「目的」を表す言葉なんだから、
そのように解釈しておけばいいんだ。そして前後の文脈で結果を表すんだということがわかったら、
結果を表しているように訳したらいいだけだ。
0585The OED Loves Me Not (ワッチョイ 8689-hn8B)
垢版 |
2023/01/19(木) 08:16:38.20ID:3O8X3TRY0
>>582
ともかく、文脈なしでその一文だけを見て論議するのは無意味だということ。
しかし常識的に言って、He stopped to smoke. と言っている人がいたら、
実際にその he は煙草を吸い始めたと解釈するのが普通だと
英語ネイティブたちが言っている。それは、僕がすでに示したリンク先の
文章を読めばわかる。
0586エキセントリックなソフィスト ◆BWpafMA3P8me (ワッチョイ 5996-w88e)
垢版 |
2023/01/19(木) 10:27:04.89ID:ZiRNq0cM0
>>528
結果用法の不定詞は、She grew up to be a famous singer.「大きくなって有名な歌手になった」のように、「~するために」と訳すと不自然であるという特徴がある。
また、「~という結果になった」「~という結果に終った」と訳せる文脈で使う。

He stopped to smoke.は、「~するために」と訳せるし、「~という結果になった」と訳せないので、文法学上目的用法である。

しかし、「立ち止まってタバコを吸った」という結果用法に近い訳し方をしてまったく問題はない。実質的にそういう意味だからだ。
※試験では、バカ正直に「タバコを吸うために立ち止まった」と書きましょう (^-^)
0587エキセントリックなソフィスト ◆BWpafMA3P8me (ワッチョイ 5996-w88e)
垢版 |
2023/01/19(木) 13:50:40.87ID:ZiRNq0cM0
He stopped to smoke.が、文法学上、結果用法に絶対に分類されない決定的な理由は、以下のとおり。

"He stopped to smoke. However, he left his cigarettes at home." (タバコを吸うために立ち止まったが、タバコを家に置いてきて吸えなかった)
 ↑
この文が成立するのである。

つまり、この不定詞は、不定詞部が実現した場合と実現しなかった場合があるのだ。
0589エキセントリックなソフィスト ◆BWpafMA3P8me (ワッチョイ 5996-w88e)
垢版 |
2023/01/19(木) 14:28:35.14ID:ZiRNq0cM0
目的用法の不定詞は、それが実現した場合としなかった場合の2つの可能性を持っている。

He went to New York to learn English..
基本訳:英語を学ぶためにニューヨークへ行った。
意訳として可能:ニューヨークに行って、英語を学んだ。

意訳が成立するからと言って、つまり、結果用法に似ているからと言って、これを文法的に「結果用法だ」と言うことはできない。
理由:結果用法とは、「~するために」と訳すと不自然な英文について言うので。

また、この英文は、「英語を学ぶためにニューヨークへ行ったが、まったく英語を学習しなかった」という文脈でも使える。
0595エキセントリックなソフィスト ◆BWpafMA3P8me (ワッチョイ 5996-w88e)
垢版 |
2023/01/19(木) 23:13:25.06ID:ZiRNq0cM0
>>594
わかりやすく説明すると

He went to New York to learn English. は、「~するために」と訳せるので、副詞的用法である。
  ↓
これは、実際にlearn Englishが行われた場合は、「~へ言って、英語を学んだ」と意訳することが許される。
※試験では許されない。
 ↓
副詞的用法の不定詞が、たとえ「~へ言って、英語を学んだ」と結果用法的に意訳されても、それを結果用法と分類してよいと言ってる文法書はない。
0599エキセントリックなソフィスト ◆BWpafMA3P8me (ワッチョイ 5996-w88e)
垢版 |
2023/01/19(木) 23:46:29.70ID:ZiRNq0cM0
>>594 結果用法の不定詞の例を文法書から抜粋しておこう。これらは、「~するために」と訳せないという特徴がある。「~するために」と訳せる余地がある時点で、それらは文法学上「結果用法」ではないのである。
※ 目的用法
● I ran all the way to find that he had gone.
●Ms. R went abroad never to return.
● That night there was a storm and I woke to hear the rain lashing the window panes.
● He lived to be ninety.
● The miserable old days have gone never to return.
● She grew up to be a pretty young lady.
● She woke early (only) to find it was raining.
● She hurried back only to find her mother dying in a hospital.
● He got to the airport only to be told the airplane had taken off.
● I ran all the way to find that she had gone.
● He went home to find his friend Ms. R waiting for him.
● He went home (only) to find his iPhone stolen.
● The drunken man awoke to find himself in a ditch.
0603エキセントリックなソフィスト ◆BWpafMA3P8me (ワッチョイ 5996-w88e)
垢版 |
2023/01/20(金) 00:01:14.05ID:AmrZl4L70
□□□□□□ まとめ □□□□□□
I stopped to smoke.の不定詞は、結果用法には分類されない。必ず目的用法である。
理由 結果用法の不定詞は、文法書を見れば分かるとおり、「~するために」と訳せないという特徴がある。以下、文法書から結果用法の例を抜粋。

● I ran all the way to find that he had gone.
●Ms. R went abroad never to return.
● He lived to be ninety.
● She grew up to be a pretty young lady.
● She woke early (only) to find it was raining.
● She hurried back only to find her mother dying in a hospital.
● He got to the airport only to be told the airplane had taken off.
● I ran all the way to find that she had gone.
● He went home (only) to find his iPhone stolen.

※つまり、共通して言えるのは、「~した。その結果~した」という意味になる。「私は立ち止まった。そして、その結果タバコを吸った」は変。だから、結果用法に分類してはいけない。
0604エキセントリックなソフィスト ◆BWpafMA3P8me (ワッチョイ 5996-w88e)
垢版 |
2023/01/20(金) 00:14:35.11ID:AmrZl4L70
□□□□□□ まとめ2 □□□□□□

I stopped to smoke.は、目的用法であるが、「立ち止まってタバコを吸った」というふうに結果用法に似た意味を持つ場合がある。
しかし、これは、あくまでも結果用法に似ているだけで、文法学上、「結果用法」には分類されない。「~した。その結果~した」と訳せない。
文法学的に、「~するために」と訳せる不定詞は、結果用法に分類されないのだ。

「目的の不定詞が、文脈次第では、結果用法になりえる」なんて述べている文法書は皆無である。(日本でも海外でも)
0605名無しさん@英語勉強中 (ワッチョイW ea0b-pSvJ)
垢版 |
2023/01/20(金) 04:47:47.56ID:/Q38yXXc0
教えてくれてありがとうございます。文法本での定義の問題なんですね。
目的で訳すのができない時に結果という。
結果で訳せても、目的で訳せる時には目的の方に分類するということですね。

I went to NewYork to study English.
については英語を勉強するためにNYに行く人もいるのでその場合は目的
ということでいいんですね?
0609エキセントリックなソフィスト ◆BWpafMA3P8me (ワッチョイ 5996-w88e)
垢版 |
2023/01/20(金) 11:25:08.27ID:AmrZl4L70
過去文における「to 不定詞」の意味が必ず実現したと言えない場合は、文法学上、結果用法にはならない。(たとえ、意味が結果のようであっても)

例文A: She went to New York to see John. 
 ↓
She went to New York to see John, but was unable to meet him. がありえるので、see Johnが必ず実現したとは言えない。だから、例文Aの不定詞は、目的用法である。
0610名無しさん@英語勉強中 (ワッチョイ 6a08-TW7m)
垢版 |
2023/01/20(金) 13:24:11.30ID:Ao464P0n0
Get your butts out there and go do it.

Get off your butt.

この2つの違いってなんでしょうか?

似た言い回しがあれば教えてください!
0611名無しさん@英語勉強中 (ワッチョイ 6a08-TW7m)
垢版 |
2023/01/20(金) 13:54:28.80ID:Ao464P0n0
そして

It’s pushing what’s on the inside of you.

この文で on theがなかったら 全然違う意味になってしまいますか?

inside of you なのに、そのまえに on theが来るのがなんとなく不思議なんです
0612The OED Loves Me Not (ワッチョイ 8689-hn8B)
垢版 |
2023/01/20(金) 14:13:29.34ID:HjvAfEl10
>>610
(1) Get your butts out there and go do it.

"your butts" と複数になっていたら、たぶん yourselves という意味であって、
yourself という意味ではない。

out there は out of there という意味だと思う。アメリカ英語では of をよく省略するから。

従って、上の英文を少し丁寧に言うと
Get yourselves out of there and go do it.
という意味になるから、
「そこから出て行って、(それ)をやれよ」
というような意味になるはず。

ネット上のあちこちの英文での解説を読んだらわかる通り、
Get your butt(s) out of there.
Get your butt here.
Get your butt over here.
などの言い回しは、
「お前、こっちへ来い、あっちへ行け」
などと、相手に移動するように言うための rude な言い回し。


(2) Get off your butt.
これは、直訳すると
「お前の尻をそこから引き上げろ」
という感じだと思う。つまりダラダラと怠けてばかりいる奴に対して、
「そんなところで座ってばかりいて座ってばかりいねえで、
ちゃんと立って行動を起こしやがれ」
というのが本来の意味だろう。これは相手が
本当に座っていなくても使えるはず。
そういうことはすべてネット上に英文で解説してあるから、それを読めばわかる。
0614The OED Loves Me Not (ワッチョイ 8689-hn8B)
垢版 |
2023/01/20(金) 14:26:37.41ID:HjvAfEl10
>>611
(1) It’s pushing what’s on the inside of you.
(2) It's pushing what's inside of you.
(3) It's pushing what's inside you.

一応、この3つはすべて正しくて、おそらく
同じような意味。ともかく (1) みたいな言い回しが
あっちこっちで使われているんだから、すぐに慣れる。

ところで、この言葉は、今どういうわけかネット上のあちこちで
引用されているけど、もともとは次のような言葉から由来してるんだろうな。

When God gives you a dream,
The dream will always be tested.
The pressure in your life right now
Has a purpose.
Its not crushing you,
★It's pushing what's on the inside of you, out.★
It shows you, who you are.
https://twitter.com/Divii_0156/status/1454479257204236294
https://twitter.com/5chan_nel (5ch newer account)
0616The OED Loves Me Not (ワッチョイ 8689-hn8B)
垢版 |
2023/01/20(金) 15:39:03.79ID:HjvAfEl10
>>611
もともと inside には、名詞、形容詞、前置詞、副詞として
の意味がちゃんとあったんだろうな。そして
(1) inside the box
なんていう言い回しも大いに使われていたんだろう。そして
アメリカではそれ以外に
(2) inside of the box
なんていう言い回しが流布した。それから、名詞としての inside を使った
(3) on the inside of [something]
つまり「~の内側に」という言い回しがもともとあったから、
3種類の言い回しが並んでいるに過ぎないんだと考えたら
単純だと思うんだけど。
0618名無しさん@英語勉強中 (ワッチョイ cfb5-BvhM)
垢版 |
2023/01/21(土) 10:12:37.50ID:XUWyk1p70
>>563
ぶっちゃけ、理屈は訊きたい部分ではないのよ。
英語喋るガイジンは語呂良い感じなのか、そうでもないのか、というところ。
理屈で良い感じに韻を踏んでいても、声に出したら、ガイジン的にイマイチとか
理屈で分かりにくいけれど感じ良く語呂や語調が良いとか、体感的なものあるじゃん。
そっちのが訊きたかったのよ
0619名無しさん@英語勉強中 (JP 0H07-BvhM)
垢版 |
2023/01/22(日) 07:41:22.39ID:b/4ZyXQ1H
なるほどな
from the get goみたいな語呂か否かということか
確かにTrussとlettuceを強めに読んだらリズムに乗ってる感じはある
0620名無しさん@英語勉強中 (JP 0H07-BvhM)
垢版 |
2023/01/22(日) 07:41:22.90ID:b/4ZyXQ1H
なるほどな
from the get goみたいな語呂か否かということか
確かにTrussとlettuceを強めに読んだらリズムに乗ってる感じはある
0622名無しさん@英語勉強中 (ワッチョイ 5332-n6IN)
垢版 |
2023/01/23(月) 22:56:21.31ID:/Su1AceN0
関係代名詞と先行詞が離れて位置する「外置」って
that, which, whom 以外の関係代名詞でもありますか?
0623The OED Loves Me Not (ワッチョイ cf89-SUdz)
垢版 |
2023/01/24(火) 07:04:00.47ID:Cp/Qt+M/0
>>622
that, which, whom 以外ということは、たとえば
whose, who あたりかな?それならいくらでも
いろんな文献を自分で読んでたら見つかるだろう。
普通の人はそういう用例を集めるために本を読んでいるのでは
なくて、あくまでその内容を読み取るためなので、気にしてないけどね。

ネットで即席に検索しても、すぐに見つかる。

(1) A directory filename has been found whose size file-size is less than the minimum directory size.

(2) A pressure dependence has been found whose origin is obscure.

これ以外にも、
"has been found whose"
を検索すれば、膨大に見つかるだろうと思う。
"has been found who"
を検索しても、たぶん見つかるだろう。それ以外の単語列を自分で考えて
検索すれば、、すぐに見つかる。
0624The OED Loves Me Not (ワッチョイ cf89-SUdz)
垢版 |
2023/01/24(火) 07:08:28.25ID:Cp/Qt+M/0
>>622
"is found who"
という単語列(string of words)を検索してみても、いろいろ見つかるだろうな。
例えばこの下に示す用例は、relartive pronoun の extraposition の用例なんだろ?

Suppose that a person is found who is deficient in an enzyme required for IMP synthesis.
0625The OED Loves Me Not (ワッチョイ cf89-SUdz)
垢版 |
2023/01/24(火) 07:10:30.49ID:Cp/Qt+M/0
>>622
"is detected whose"
という string of words を検索しても、やっぱりいろいろ見つかるだろうな。
例えば、次のようなもの。

A process that fills packages is stopped whenever a package is detected whose weight falls outside the specification.
0626The OED Loves Me Not (ワッチョイ cf89-SUdz)
垢版 |
2023/01/24(火) 07:27:13.56ID:Cp/Qt+M/0
>>622
"is announced whose"
という string of words を検索しても、それらしき用例が当然のことながらたくさん見つかるだろうな。
例えば、次のようなもの。

All ticket numbers for the draw are collected together, and the winner is announced whose ticket numbers match withdrawn.

こういうときには、どういう単語とどういう単語を組み合わせたら自分の求める用例が出てきそうなのかを予測しないといけない。

たとえば

"has been rejected whose"
"will be rejected who"

"is discarded whose"

"is manufactured whose"

こういう string of words を検索画面に放り込んだら、
必要な用例がわんさか出てくるだろうよ。
0627名無しさん@英語勉強中 (ワッチョイ cf4b-XYFd)
垢版 |
2023/01/24(火) 17:23:54.37ID:jSEH9Tv00
The 200m freestyle is four laps of the pool.
lap(s)の意味がいまいちわからないのですが、このプールの大きさは
・4往復で計200m、200/(4*2)=25m
・都合4回行ったり来たりで計200m、200/4=50m
のどっちなのでしょうか
0629The OED Loves Me Not (ワッチョイ cf89-SUdz)
垢版 |
2023/01/24(火) 17:30:47.02ID:Cp/Qt+M/0
>>627
>>628 に出てくる情報に基づけば

The 200m freestyle is four laps of the pool.

この英文の意味は
「200メートル自由形は、ポールを片道4回分だ」
と言っているのだから、結果的に
「200メートル自由形は、2往復だけ泳ぐものだ」
という意味になるわな。
0630名無しさん@英語勉強中 (ワッチョイ c396-XYFd)
垢版 |
2023/01/24(火) 17:37:08.62ID:Zh9kSQLq0
test
0631名無しさん@英語勉強中 (ワッチョイ cf4b-XYFd)
垢版 |
2023/01/24(火) 17:39:47.84ID:jSEH9Tv00
ありがとうございます
競技で25mプールってのも何かしょぼいのでおかしいと思ったら、下の意味でしたか
辞書にのってた「プールの一往復」ってのは紛らわしいというかヘンですね
0632The OED Loves Me Not (ワッチョイ cf89-SUdz)
垢版 |
2023/01/24(火) 17:42:36.57ID:Cp/Qt+M/0
>>631
ごめん。改めて調べ直すと、この lap の意味は、
人によって違うんだそうだ。

https://www.letsrun.com/forum/flat_read.php?thread=2940859

この上のリンク先を読むと、one lap という言葉を
(1) 片道だけ(つまり、こっちから向こう側にたどり着くまでの距離)
(2) 往復
この二通りの意味で使われていて、swimmer や coach によって
まちまちの意味で使っているんだそうだ。

ただ、オリンピックの場合には one lap と言えば 50 meters なんだそうだ。
0633名無しさん@英語勉強中 (ワッチョイ c396-XYFd)
垢版 |
2023/01/24(火) 17:45:46.04ID:Zh9kSQLq0
There were times when という表現があるのは知っていたが
There were times when his talk about obscure points of philosophy had us completely bemused.
むずかしい哲学の話をしては我々をけむに巻くことがあった. - 研究社 新和英中辞典

There were times that という表現があるのは知らなかった
There were times that I found the plot a little boring but it then gets better.
"http://www.goodreads.com/book/show/613932.How_to_Take_the_Ex_Out_of_Ex_Boyfriend";

このthatは同格のthatでtimesを説明してるのか?
0634The OED Loves Me Not (ワッチョイ cf89-SUdz)
垢版 |
2023/01/24(火) 17:45:48.08ID:Cp/Qt+M/0
>>631
Olympics のプールが片道50メートルなので、
次のような計算になるはずだ、とリンク先の人たちが言っている。

The Olympics are held in a Long Course meter pool which is 50 meters long.

One lap is 50 meters. So 400 meter freestyle has 8 laps, the 800 meter is 16 laps etc.

https://www.letsrun.com/forum/flat_read.php?thread=2940859

まあともかく、こういうことに興味のない僕が片手間に斜め読みしただけの
情報だから、あんまり信用しないで、自分でリンク先の長い英文をきちんと読み取ってほしい。
0635The OED Loves Me Not (ワッチョイ cf89-SUdz)
垢版 |
2023/01/24(火) 17:50:17.06ID:Cp/Qt+M/0
>>633
ここでの
There were times that S V.
における that は、when という関係副詞の代わりに使われているそうだ。
それについては、5ちゃんねる英語版でも何度か取り上げられたことがあるけど、
「ジーニアス英和、第6版」の「代名詞としての that」の項目にある (2) の
「関係副詞的に」と書いてある項目に説明がある。
0636The OED Loves Me Not (ワッチョイ cf89-SUdz)
垢版 |
2023/01/24(火) 17:55:51.19ID:Cp/Qt+M/0
>>631
OED Online の lap の項目は、まだ古い version のままだけど、
ここでもやはり、swimming pool における one lap が
片道を表すときと往復を表すときがあると書いている。

lap
Draft additions January 2005

Swimming.
One defined stage of a course, typically one or two lengths of a swimming pool.
0637名無しさん@英語勉強中 (ワッチョイ 5332-n6IN)
垢版 |
2023/01/24(火) 18:27:51.07ID:YkiUPD950
A waitress came up to us with a menu in her hand. に関してですが
A menu was in her hand なんて言い方をしない以上、
このwithは付帯状況を表すものではなく単にhaving を意味する前置詞と解釈すべきですよね?
0638名無しさん@英語勉強中 (ワッチョイ 5332-n6IN)
垢版 |
2023/01/24(火) 18:53:43.06ID:YkiUPD950
追記すると、学校ではこれをwith+名詞+前置詞句 型の付帯状況構文と習ったのですが
正直、こういうのはネイティブからすると不自然な英語ではないかと思い、訊いてみました。
0639名無しさん@英語勉強中 (ワッチョイW bf8a-mTvl)
垢版 |
2023/01/24(火) 18:59:26.39ID:dAk5IYgU0
別に不自然でも何でもないでしょ
普通の英文

>A menu was in her hand なんて言い方をしない以上、

これも別に言おうと思えば普通に言う
0640The OED Loves Me Not (ワッチョイ cf89-SUdz)
垢版 |
2023/01/24(火) 19:02:57.19ID:Cp/Qt+M/0
>>637
A menu was in her hand.
に似た言い回しは、小説にも出てくる。

When Jessica returned, ★a small ivory menu was in her hand.★
“It smelled amazing. And yes, they deliver to campus.” “Nice,” Kelsey said, tucking her napkin ...

この下のリンク先の下から5行目あたりに出ている。

https://books.google.co.jp/books?id=ByyKDwAAQBAJ&pg=PT94&lpg=PT94&dq=%22menu+was+in+her+hand%22&source=bl&ots=t2EeutNCbc&sig=ACfU3U1axjFl_5L-DxjowhI92SPajo5AJQ&hl=en&sa=X&ved=2ahUKEwiWuoas-d_8AhXZZd4KHQYbCqgQ6AF6BAgJEAM#v=onepage&q=%22menu%20was%20in%20her%20hand%22&f=false
0641The OED Loves Me Not (ワッチョイ cf89-SUdz)
垢版 |
2023/01/24(火) 19:07:47.41ID:Cp/Qt+M/0
>>637
His hat was in his hand.
という言い回しも、あちこちに出ている。たとえば小説らしきものの
中に、次のような一節がある。わざとリンクは貼らないでおく。

★His hat was in his hand★, and his face was white. “Excuse me, ladies,” he said. ...
I was putting my book on the floor beside my cot when I saw him.
0642The OED Loves Me Not (ワッチョイ cf89-SUdz)
垢版 |
2023/01/24(火) 19:10:43.17ID:Cp/Qt+M/0
>>637
A book was in his hand.
His book was in his hand.
などの言い回しも、あちこちに見つかる。
まったく不自然でも何でもない。

with a menu in her hand
にしても、ごくごく普通の言い回し。あっちこっちの
小説などで出てくる。試しに検索してみれば、
それに似たものはいくらでも見つかる。
0643名無しさん@英語勉強中 (ワッチョイ 5332-n6IN)
垢版 |
2023/01/24(火) 19:13:20.20ID:YkiUPD950
ありがとうございます
ただ、これを堅苦しく付帯状況構文として扱うよりも
普通に with を having と解釈して副詞句として理解したほうが
自然な気がいつもしてまして
0645名無しさん@英語勉強中 (ワッチョイ 5332-n6IN)
垢版 |
2023/01/24(火) 19:24:41.83ID:YkiUPD950
なので、A waitress came up to us with a menu in her hand.
なんかよりも A waitress came up to us having a menu in her hand.
の方がナチュラルな英語表現なのじゃないと思っておりまして
0646The OED Loves Me Not (ワッチョイ cf89-SUdz)
垢版 |
2023/01/24(火) 19:35:13.58ID:Cp/Qt+M/0
>>644
> 私は彼にあげたメダルを彼から取り返した
> I took back the medal from him which I gave him
> って間違ってます?
> whichはmedalの後にこないとダメですか?
====

関係代名詞を先行詞の直後に簡単に持って来られるときには、
そうした方がいいに決まってるわな。

A man, about 60 years old, who was waiting for the woman in a black Buick Enclave, then walked up
and pushed the manager and ★took back from him the woman’s purse containing the stolen items★.
https://www.cleveland.com/community/2022/03/theft-suspect-woman-about-65-uses-cane-to-strike-store-manager-and-make-her-getaway-university-heights-police-blotter.html

上のような例文がちゃんとあちこちに見つかると思うので、
質問者の抱えている英文は、次のように書いた方がいいわな。

I took back from him the medal I'd given him.
こう書けば、まずは which なんていう堅い関係代名詞を
使わなくても that でもよくなるし、さらには関係代名詞を
省略することさえできて、身軽な英文になる。
ところで、
I gave him.
じゃなくて
I'd given him
と書かないといけない。
0647名無しさん@英語勉強中 (ワッチョイ c396-XYFd)
垢版 |
2023/01/24(火) 21:29:45.85ID:Zh9kSQLq0
>>635
そんな用法あったんだなぁ
元はと言えば↓この意味わからん英文見たのが始まりだが 英語は奥が深い
関係副詞のwhenとthatがtimesを二重限定修飾してたんだな

There were times when he sat watching the boy sleep that he would begin to sob uncontrollably but it wasnt about death. "The Road"
0648名無しさん@英語勉強中 (スップー Sd1f-wtBt)
垢版 |
2023/01/24(火) 22:47:09.59ID:VBgOWe7gd
女性側通りに面してるのとは反対側のドアからパン屋に入った

a woman,by,entered,to,opening,the door, that,the bakery,the street, onto, opposite,

並べ替えなのですが全くわかりません!助けてください
0649名無しさん@英語勉強中 (ワッチョイW bf8a-mTvl)
垢版 |
2023/01/24(火) 23:18:40.21ID:dAk5IYgU0
A woman entered the bakery by the door opposite to that opening onto the street.
0650名無しさん@英語勉強中 (ワッチョイW bf8a-mTvl)
垢版 |
2023/01/24(火) 23:23:05.08ID:dAk5IYgU0
>>645
そもそもべつにwith構文は堅苦しくも不自然でもないよ
参考書とかで聞いたことあるのかもしれないけど特に気にしなくていいです
その文だとむしろhaving の方が変というかあまり言わないと思う
意味的にメニューを持ちながら〜となるからhave を使いたくなる気持ちはわかる
0651名無しさん@英語勉強中 (スップー Sd1f-wtBt)
垢版 |
2023/01/24(火) 23:42:38.82ID:sh6ioZ0Kd
>>649
by the doorとはどういう意味でしょう

またthatはどういう用法ですか
0654名無しさん@英語勉強中 (スップー Sd1f-wtBt)
垢版 |
2023/01/24(火) 23:47:20.41ID:sh6ioZ0Kd
全部使う問題です
0655名無しさん@英語勉強中 (ワッチョイW bf8a-mTvl)
垢版 |
2023/01/24(火) 23:53:48.33ID:dAk5IYgU0
>>651
by the door はそのドアを通って、という意味で、
thatはdoorの意味の代名詞です
0656名無しさん@英語勉強中 (スップー Sd1f-wtBt)
垢版 |
2023/01/25(水) 00:04:54.01ID:j52K1aVUd
>>655
thatだと私が入った「the door」をそのまま受けることになりませんか?
oneとかじゃなくていいんですかね
0657名無しさん@英語勉強中 (スップー Sd1f-wtBt)
垢版 |
2023/01/25(水) 00:09:02.10ID:j52K1aVUd
あとopening onto the streetは道路に向かって開かれていて、かつ道路に面してる的な意味ですか?
0678名無しさん@英語勉強中 (ワイエディ MMdf-qYbV)
垢版 |
2023/01/25(水) 23:36:34.43ID:FRaDul0YM
こんばんは
toeic公式問題集7に

We're always glad to get skilled writers such as yourself on our team.
あなたのような熟練記者を私たちのチームの一員に迎えるのは、常にうれしいものです。

とあるのですが、この再帰代名詞 yourself は主語と一致しておらず、どういったニュアンスを表す表現なのでしょうか?
分る方よろしくお願いいたします。
0680名無しさん@英語勉強中 (ワイエディ MMdf-qYbV)
垢版 |
2023/01/26(木) 00:04:13.92ID:t8pp0UL4M
>>679 さん
ありがとうございます。
0682名無しさん@英語勉強中 (ワッチョイ c396-DmZS)
垢版 |
2023/01/26(木) 18:03:39.05ID:/yPe67z70
In November 2010, Penrose and V. G. Gurzadyan announced they had found evidence of such circular patterns, in data from the Wilkinson Microwave Anisotropy Probe (WMAP) corroborated by data from the BOOMERanG experiment.

announced they had found evidence
had foundは現在完了の時制の一致なのか大過去なのか
どっちに解釈しても意味はほとんど変わらないと思うけどあえて言うならどっちなんだろうか?
ネイティブはどっちの解釈で読んでるんだろう
0683The OED Loves Me Not (ワッチョイ cf89-SUdz)
垢版 |
2023/01/26(木) 18:38:11.02ID:krQMcJ7G0
>>682
>> In November 2010, Penrose and V. G. Gurzadyan announced they had found evidence of ....

この下に引用した記事を読んでいると、この二人がこの evidence を発見したのがつい最近であるか、
あるいは発見したのがずっと前だったとしても、それを発見したことを announce するのは今回(November 2010)が
初めてであるように読み取れる。もしそうなら、announced they had found のところは、もともとは
they have found というふうに現在完了形だったのだと解釈できる。つまり、言ってみれば
In November 2010, Penrose and V.G. Gurzadyan said, "Hey, look, we have found ewvidence of..."
のようにみんなに呼びかけたというわけだ。

"Cosmic rebirth," Circular patterns in the universe's pervasive background radiation suggest
the Big Bang was only the latest of many. Science News (November 26, 2010). ?About Gurzadyan-Penrose paper.

ただそれは、イギリス英語の場合ならそういうことが言えるということだ。アメリカ英語は曲者で、本来なら
現在完了形で書くべきところを、何でもかんでも過去形で言ってしまう(口語では特にその傾向が強くなる)
ので、ここでも、気楽なアメリカ英語の口語ならば、もとから過去形だったものが過去完了になったのだと解釈することもできる。
0684名無しさん@英語勉強中 (ワッチョイ c396-DmZS)
垢版 |
2023/01/26(木) 18:54:35.29ID:/yPe67z70
>>683
ありがとう これは現在完了よりか
それにしてもなぜ英語は大過去形を捨てたのか ややこしくて仕方ない
0686名無しさん@英語勉強中 (ワッチョイ c396-DmZS)
垢版 |
2023/01/26(木) 19:22:34.58ID:/yPe67z70
>>685
いやラテン語には過去形から独立した
大過去形があったんだよ
今でも確かフランス語にもあるだろう
それが英語ではなぜhad pp形で過去完了形に吸収されたのか
0687The OED Loves Me Not (ワッチョイ cf89-SUdz)
垢版 |
2023/01/26(木) 19:40:20.45ID:krQMcJ7G0
>>686
フランス語の大過去なら、
https://www.francaisfacile.com/exercices/exercice-francais-2/exercice-francais-31313.php
ここに表があるけど、ancien francais つまり古いフランス語についてはよく知らないけど、
少なくとも現代フランス語では have p.p. と同じような形で plus que parfait (pluperfect つまり大過去)
を表現しているけど。
0689The OED Loves Me Not (ワッチョイ cf89-SUdz)
垢版 |
2023/01/26(木) 19:48:56.06ID:krQMcJ7G0
>>686
ラテン語のことはよくわからんけど、僕がうっすらと覚えている限りでは、
http://latindictionary.wikidot.com/verb:confirmare
上のような活用をするよね。

ラテン語の過去から大過去(pliuperfect)は確かに派生していると
言えるだろうけど、その派生の仕方は英語の過去から大過去(過去完了)
が派生するやり方とそんなに違わないように僕には見えるんだけど、
違うかね?
0690名無しさん@英語勉強中 (ワッチョイ 5332-n6IN)
垢版 |
2023/01/27(金) 02:01:40.89ID:fuRf5giW0
擬似関係代名詞 as に関する質問です
When modernization is achieved, the country, as we know, will have disappeared.
この文を接続詞を用いて書くなら、
When modernization is achieved, the country will have disappeared, and we know this.
でよろしいでしょうか?
0691名無しさん@英語勉強中 (ワッチョイ 5332-n6IN)
垢版 |
2023/01/27(金) 02:16:52.50ID:fuRf5giW0
加えて、擬似関係代名詞 as の「先行詞」は
まるまる When modernization is achieved, the country will have disappeared.
ということでよろしいでしょうか?
0692名無しさん@英語勉強中 (ワッチョイ 4389-FlOQ)
垢版 |
2023/01/27(金) 02:51:26.93ID:dp6m23FT0
ネットにある
When modernization is achieved, the country as we know it will have disappeared.
って文なら
as は直前の名詞を限定する接続詞 意味は・・するような
it = the country 
だとおもう
0693名無しさん@英語勉強中 (ワッチョイ 4389-FlOQ)
垢版 |
2023/01/27(金) 02:58:20.42ID:dp6m23FT0
When modernization is achieved, the country, as we know, will have disappeared.
みたいな文あんまりみたことないけど
as(関係代名詞・目的格) の先行詞は the country って解釈になるんじゃないの
私たちが知るその国は存在しなくなるでしょう みたいな
0694名無しさん@英語勉強中 (ワッチョイ 4389-FlOQ)
垢版 |
2023/01/27(金) 03:09:35.81ID:dp6m23FT0
辞書だと
文を先行詞とするの語法はあるけど、この使い方は載ってない気がする
関係代名詞継続用法の as として解釈しようとしても 変な感じがするし
現代英語の表現として自然なんだろか
0695名無しさん@英語勉強中 (ワッチョイ 4389-FlOQ)
垢版 |
2023/01/27(金) 03:22:45.61ID:dp6m23FT0
この文作ったネイティブ(?)がこんな解釈してるみたいだけど
People know that the country will have ceased to exist as a political entity.
よくわからんな
全文を as の先行詞にするなんてあるんだろか
0696名無しさん@英語勉強中 (ワッチョイ c396-DmZS)
垢版 |
2023/01/27(金) 06:25:25.32ID:0Nq/wVZo0
>>690
英辞郎
as we knowとは
as we knowの検索結果
30件 検索結果一覧を見る
as we know の意味・使い方・読み方
as we know
皆さんご存じのとおり、知ってのとおり
0697名無しさん@英語勉強中 (テテンテンテン MM7f-kFiv)
垢版 |
2023/01/27(金) 07:56:34.10ID:lbsdG47EM
What made you decide to apply for this job?

WhatがS、madeがV、youがO、decideがC、to apply以下が副詞的用法でdecideを修飾の第五文型に見えてしまいます
実際はdicideがVでto applyがOのようです
なぜmadeがVにならないのでしょうか?

https://i.imgur.com/qlZ7KZA.png
0698The OED Loves Me Not (ワッチョイ cf89-SUdz)
垢版 |
2023/01/27(金) 08:27:40.05ID:CkBCI1Ub0
>>690
(1) 課題文: When modernization is achieved, the country, as we know, will have disappeared.

結論から言うと、この英文は間違いで、これを書いた人は実は
(2) When modernixzastion is achieved, the country as we know ★it★ will have disappeared.
と言いたかったのだ。

つまり、堅い日本語訳で申し訳ないが

「近代化が達成されたら、我々が今認識しているその国は消滅するだろう。」

つまり、その国は完全にあらゆる意味で滅びて、地図から消えてなくなるとかいうわけではなく、
一応は(例えば日本だったら)日本という国の名前が残って地図上に残るだろうけど、
日本という国はこんな国だと今はみんなが思っているけど、そういう形態での日本という国は、
その頃にはもはやなくなっている、という意味だ。たとえば傀儡政権になってしまっているとか、
外国人がその国の半分を占めているだとか、男性の比率が全体の2割になってしまっているとか、
子供が一人もいないとか、ほとんど誰も働いていないとかいう、およそ「これが日本だ」なんていうふうには
言えないような「国」の残骸みたいなものになり果てているだろう、という意味のことを
この筆者は本当は言いたかったのだ。

このあとに、その根拠を述べる。
0700The OED Loves Me Not (ワッチョイ cf89-SUdz)
垢版 |
2023/01/27(金) 08:38:03.26ID:CkBCI1Ub0
>>690
"as you know" だけで it がなければ、本来は >>696 が言うように
「みなさんも知っているように」という意味だ。しかしそういう意味だと
解釈すると、この原文は変だと感じられる。

(1) 課題文: When modernization is achieved, the country, as we know, will have disappeared.

「皆さんも知っているように」と訳すと、
「近代化が達成された暁には、みなさんも知っているように、この国は消滅してしまっているだろう」
となる。なお、英語では
We know that S V.
は、「~だと確信している」
というような意味になる。たとえば I believe in God. どころか、「俺は God が存在するって確信してるんだ」
と言いたければ I know that God exists. って言う。それと同じように、この課題文の as we know を
「みなさんも知っているように」とか「我々がすでに知っているように」と解釈すると
「我々が確信しているように(知っているように)この国は消滅してしまtっているだろう」
なんてことになる。それは予知能力を持った人間だけが言えることだ。

普通の人間はそんなことは言わない。
仮に1割くらいの人間がそれを確信しているとしても、いくらかの人間は
We predict that it will have disappeared.
くらいに留めておいたり、あるいはもっとぼやけた言い方をするはずだ。

続く
0701The OED Loves Me Not (ワッチョイ cf89-SUdz)
垢版 |
2023/01/27(金) 08:42:21.73ID:CkBCI1Ub0
>>690
>>695 によれば、この課題文を書いた英語ネイティブが
>>People know that the country will have ceased to exist as a political entity.

このように言いたかったのだ、と言っている。この解説の意味は、
「この国は ★政治的な組織★ としては、そのころはもはや小説してしまっているだろう、とみんなが確信している」
という意味だ。

したがって、このネイティブは実は
the country, as we know, will
と言いたかったのではなくて、僕がもともと言っていたように
the country as we know ★it★ will have...
と言いたかったのだ。

この二つの言い回しを、このネイティブはごっちゃにしているだけのことだ。
0704The OED Loves Me Not (ワッチョイ cf89-SUdz)
垢版 |
2023/01/27(金) 08:52:29.12ID:CkBCI1Ub0
>>697
> What made you decide to apply for this job?
>
> なぜmadeがVにならないのでしょうか?

あなたの言う通り、made は確かに V だけど、
それの付属みたいなものとして decide が別の V になっている。

make [人] 動詞
で「人に~させる」という意味になるってことは、辞書を見ればわかる。

そしてそういうことをすべて踏まえた上で、画像内にある解説をした
人は解説をしている。
0706名無しさん@英語勉強中 (テテンテンテン MM7f-kFiv)
垢版 |
2023/01/27(金) 12:50:44.69ID:Ha/rc+GpM
>>704
ありがとうございます!
第五文型という認識であっていて、decideはCであるけど、働きとしてdecide(Vt) to apply(O) でひとまとまりの句だよという意味で著者はVt,Oという文型を書いたということなんですね
0707The OED Loves Me Not (ワッチョイ cf89-SUdz)
垢版 |
2023/01/27(金) 19:14:53.88ID:CkBCI1Ub0
>>690
念のために付け加えると、
"the [名詞] as we know it"
という言い回しは、頻繁に使われる。意味はもちろん、今回の
英文にも出て来たように
「われわれが知っている [名詞] というもの」
みたいな意味になる。ネット上のニュース記事からこのような
フレーズの用例を2つだけ挙げておこう。

(1) Be prepared for the end of ★Israel as we know it★
https://www.thejc.com/lets-talk/all/be-prepared-for-the-end-of-israel-as-we-know-it-5arAUURNp5TKj1KX5jvLET

(2) Blue Monday, and the end of ★the world as we know it★
https://observerxtra.com/2023/01/19/blue-monday-and-the-end-of-the-world-as-we-know-it/

このような
"the * as we know it"
というフレーズは、腐るほど一瞬にして見つかるから、ジャンジャン検索して、
その前後を読み取ってみたらいい。
0708The OED Loves Me Not (ワッチョイ cf89-SUdz)
垢版 |
2023/01/27(金) 19:43:09.72ID:CkBCI1Ub0
>>690
よく考えたら、"as we (you) know it" という表現も、
最初から辞書を引けばよかったんだな。僕はこの
表現を洋書を読む中で自然に覚えたので、
辞書をいちいち引こうと考えたことがなかった。

(1) ジーニアス英和、第6版

as you [we] know it
[名詞の後で] 現在あるような、今のような、慣れ親しんだ
With this invention, ★life as we know it★ will be completely changed.
この発明で、今のような生活はすっかり変わるだろう。

(2) Merriam-Webster's Advanced Learner's Engish Dictionary

★The building as we know it {= as it exists now]★ is quite different
from how it looked when it was first built.

This is the end of ★the organization as we know it★.
(= The organization has changed and will never again be the same as it was.]
0709名無しさん@英語勉強中 (ワッチョイ d35a-DmZS)
垢版 |
2023/01/27(金) 23:17:30.16ID:yFwZz+Vm0
古語のwitのように不定形と語尾変化が異なる動詞って、ほかになにがありますか??
0712名無しさん@英語勉強中 (ワッチョイW 23bd-f93Z)
垢版 |
2023/01/28(土) 12:37:42.42ID:fsJqBU600
https://i.imgur.com/hpj0052.jpg

デートに誘ったらこうしたリプライがきました。
これってどういうニュアンスですか?
彼氏がいるという意味なのか、友達と会う約束があるという意味なのか、、
悲しい
0713名無しさん@英語勉強中 (ワッチョイW 8a8a-FK61)
垢版 |
2023/01/28(土) 12:43:41.18ID:fuuDZdMF0
いや完全に他の人とdating言うとるやろアキラ
0715名無しさん@英語勉強中 (オッペケ Sr3b-fo5h)
垢版 |
2023/01/28(土) 15:07:08.35ID:OTTV2YN0r
アキラメロン
0719名無しさん@英語勉強中 (ワッチョイW 2345-Pxgu)
垢版 |
2023/01/29(日) 03:06:48.65ID:/oFK9qWQ0
いわゆる「魔法の杖」ですが
RPGにロッド、スタッフ、ワンド、ステッキといったアイテムが出てきます。
それぞれに使い分けはあるんでしょうか?
0721名無しさん@英語勉強中 (ニククエW 0ee3-S+s3)
垢版 |
2023/01/29(日) 22:43:46.62ID:4Jrb1n5Z0NIKU
何であれやる事を誰も知らない者よりも、なんて良い共謀者だ
分かりやすい日本語だと
…誰にも知られない者より勝る願ってもない共謀者だ
0722名無しさん@英語勉強中 (ニククエW 0ee3-S+s3)
垢版 |
2023/01/29(日) 23:10:51.53ID:4Jrb1n5Z0NIKU
>>721
訂正
…誰にも知られない者以外願ってもない共謀者
「より」より以外の方がいいね
0724ブタ耳 (ニククエ 3ee3-OEO/)
垢版 |
2023/01/29(日) 23:42:55.99ID:oiH+sTTs0NIKU
そいつとの関わりを他の誰にも知られていないとしたら、そいつ以上に共犯者として好都合な人間が他にいるだろうか?
0727名無しさん@英語勉強中 (ワッチョイW 8a8a-FK61)
垢版 |
2023/01/30(月) 00:45:59.25ID:2kNhpogj0
出来たのは、普通に3秒で意味がわかった俺とchatGPTに聞いてわかった豚耳だけか
0729名無しさん@英語勉強中 (ワッチョイ 5f32-I6UX)
垢版 |
2023/01/30(月) 01:03:38.33ID:3xGwARxd0
He is about my age. 「彼は私とほぼ同い年です」
これは He is a man of about my age. から a man of が省略されもの
と考えていいでしょうか?
0732😉三年英太郎🌈 ◆3CZBjOt3.Y (ワッチョイW 9a86-os8v)
垢版 |
2023/01/30(月) 03:09:18.90ID:R3qqsEni0
メノトミー😎

「私とあなたは同い年」とか「私があなたの歳の時は」という日本語が何かの省略しているわけではない(と日本語ネイティブは感じる)のと同様、その英文も、現代英語では省略ではない

ということいくら言ってもわからないおバカさんたちがいたので、省略で納得できるなら省略でいいんじゃない
0733エキセントリックなソフィスト ◆BWpafMA3P8me (ワッチョイ a796-fO7+)
垢版 |
2023/01/30(月) 10:16:41.35ID:GCbgd7y00
>>732  3年ちゃんのほうがおバカなのは、証明済みなんだが (^O^)
□□□三年英太郎ちゃんの意見が間違っていることのわかりやすい説明(まとめ)□□□

三年英太郎ちゃんの説明: 「When I was your age」は、I と your ageが概念的に異なるものなので、「I」は「my age」のメトニミー(比喩)なのだ。

私の主張: このyour ageは、The area is the size of Nagoya.のthe sizeと同じものである。この文は、The area is of the size of Nagoya.と言っても文法的に正しい。同様に、「When I was your age」は、「When I was of your age」と言っても、文法的に正しい。

ジーニアスの「size」の項目に次の例が載っている。
『an area the size of Nagoya 名古屋ほどの大きさの地域 《◆名詞のあとに来たときも同様にofはしばしば省略される》』

これは修飾関係にあるので、「(of) the size」は、「(of) your age」と同様、形容詞のように働く句であることがわかる。(英語では「Adjectival Phrase」(形容詞的な句)と呼ばれ、日本の教育現場では「形容詞句(の一種)」として扱われている。) ジーニアスの例文の被修飾語「area」が「エリアの大きさ」という意味の比喩でないのだから、「When I was (of) your age」の「I」も何かの比喩(メトニミー)であるわけがない。「(of) your age」は、主語の持ってるプロパティ(属性・性質)を示す語、つまり主語を形容する語なのだから、同じ概念である必要はない。よって、3年ちゃんのメトニミー説は大間違いである。
0735エキセントリックなソフィスト ◆BWpafMA3P8me (ワッチョイ a796-fO7+)
垢版 |
2023/01/30(月) 11:00:19.31ID:GCbgd7y00
>>732
なにが、「メノトミー」だ? メトニミーだろ。俺まで間違ったじゃねーか (^O^) ほんと3年ちゃんは、ボケが来てるなあ。

>>729
He is about my age.は、「He is of about my age. から of が省略されもの」と考えるのがベスト。

次のような英文が、少ないながらも存在する。「This book is of about ☆ pages.」(この本は、ほぼ☆ページある)
※この英文は、今ではふつうofを省略する。「This book is about 100 pages.」がふつう。

では、このofは何かというと、>>733と同様であり、 メトニミーの関与する余地はないのだ。
0737ブタ耳 (ワッチョイW 3ee3-FGvc)
垢版 |
2023/01/30(月) 11:16:45.05ID:csgsJG+t0
次のコテ (^O^)も「ソフィスト氏から後は任せたと言われている」という設定でいくんでしょうか?
0739エキセントリックなソフィスト ◆BWpafMA3P8me (ワッチョイ a796-fO7+)
垢版 |
2023/01/30(月) 11:38:11.39ID:GCbgd7y00
最後に。

「When I was your age」は、「When I was of your age」の「of」が省略されたものである。論拠>>733のジーニアスの引用

この「I was (of) your age」は、「I was in a hurry」と同様、主語の様態を示す表現で、「I」と「hurry/age」が等価である必要はない。(I was a person in a hurryと考える必要がない)

だから、等価にするためのの理論「メトニミー(比喩)」を持ち出す必要などまったくない。

「He is (of) about my age」も、「I was (of) your age」と同様で、主語の様態を表す句であり、等価にするためのの理論「メトニミー(比喩)」を持ち出す必要などまったくない。
0740名無しさん@英語勉強中 (ガックシ 06b6-DSsr)
垢版 |
2023/01/30(月) 12:32:59.72ID:PQOZ+dN16
http://user.keio.ac.jp/~rhotta/hellog/cat_etymology-17.html
これには、First year by OEDとして、「C18?」の例が書かれていますが、
C18は18世紀のことでしょうか。OEDは何の略でしょうか?
0742名無しさん@英語勉強中 (スップ Sdba-9qR+)
垢版 |
2023/01/30(月) 12:47:25.24ID:zLiklSENd
106 名無しさん@英語勉強中 2023/01/28(土) 21:37:00
コーデ
冬の一週間コーデとかこのパンツを使っていくつかコーデを組んでみましたとかいつの間にか定着してんね

107 名無しさん@英語勉強中 2023/01/29(日) 18:46:29
それ和製英語じゃなくて略語

108 名無しさん@英語勉強中 2023/01/29(日) 20:48:01
いや和製英語だろ
服装示すのにcoordinateなんて使わん

109 名無しさん@英語勉強中 2023/01/30(月) 07:29:40
コーディネートが「服装」という意味で使われてる様に見えるお前の読解力がヤバい
日本のカタカナ語でもそんな意味で使ってない

110 名無しさん@英語勉強中 2023/01/30(月) 07:58:16
ファッション業界でcoordinateっつたら「おすすめの組み合わせ」みたいなニュアンスだからな?
みーんな意味が分かっててそこらへんのメスガキとかもそういう意味で使ってると思うぞw

動詞2〈…を〉整合する,調整する,調和させる
名詞2[複数形で] コーディネート 《色調・生地など組み合わせの効果をねらった婦人服》
0743名無しさん@英語勉強中 (ワッチョイ 5f32-I6UX)
垢版 |
2023/01/30(月) 13:14:26.00ID:3xGwARxd0
He is about my age. の分析と解釈とてもよくわかりました 感謝です
ですが 単語の重複や長ったらしくない文である He is of about my age. を
He is about my age. と それでもあえてof を省略するわけはなんでしょうか?
He is of about my age. のままならとても分りやすいのですが
of があると 英語の言葉のリズム感を阻害するからでしょうか?
0747名無しさん@英語勉強中 (ブーイモ MM26-9PUi)
垢版 |
2023/01/30(月) 17:04:11.17ID:0GIoFrTEM
>>744
ものすごく柔らかく考えると文脈によって
「着こなし(た結果)=服装」みたいなニュアンスで使う事はあるけど、
じゃあ「着こなし」って「服装」という意味ですか?って言われると「うーん、それはちょっと」という感じ
0748エキセントリックなソフィスト ◆BWpafMA3P8me (ワッチョイ a796-fO7+)
垢版 |
2023/01/30(月) 17:53:12.91ID:GCbgd7y00
>>743
古典的表現の「When I was of your age」は、今では「When I was your age」と「of」を省略して言うのが普通です。

ジーニアス英和辞典のofの?「記述」用法の「b)]にこのofの用法が載っています。その中の例のいくつかに、「ofを省略するほうがふつう」と書いてありますが、その理由までは書いていません。

私の推測では、単に習慣でそうなったのであり、特に理由はないのではないかと思われます。
0751名無しさん@英語勉強中 (ワッチョイ a796-DSsr)
垢版 |
2023/01/30(月) 19:23:08.38ID:BLwHDsgB0
>>741
shut up 😃
0753名無しさん@英語勉強中 (ワイーワ2W FF12-3iQZ)
垢版 |
2023/01/30(月) 21:59:42.20ID:PIw9W2owF
次のspotの意味がよく分かりません。
〜, because there are only two spots left.
もう2席分しか席がありませんので。
辞書には席や余りなどの意味が載っていなかったです
0777名無しさん@英語勉強中 (ワッチョイ 3b96-DSsr)
垢版 |
2023/01/31(火) 04:09:35.71ID:yklk9Z4u0
今人気の曲 overdose
https://youtu.be/H08YWE4CIFQ
歌詞
Overdose 君とふたり 甘いハッタリ
Don't stop it music,darling

don't stop it music とは? どういう用法だろう? それ音楽を止めるな みたいな?
あと多分 overdose も overdosed の過去分詞形のほうが完了的な意味があっていいと思うんだが……
0779名無しさん@英語勉強中 (スップ Sd5a-D4We)
垢版 |
2023/01/31(火) 07:28:46.25ID:g4uELNiLd
テンポを付けるためにそういう言い方してるだけで
don't stop it / music
it = music
かと
0783The OED Loves Me Not (ワッチョイ 4e89-2biX)
垢版 |
2023/01/31(火) 09:19:09.12ID:/hCkEIhV0
>>777
>>779 と、だいたい同じことを考えている。普通なら
Don't stop the music, darling!
って言うところ。リズム的にはここはit でも the でも、
音楽のメロディには一応は適合する。

でもおそらく、作詞作曲した人はたぶん、the よりもどうしても
it と発音した方が全体的にきれいに聞こえると感じたんだろうと思う。

以上は、あくまでこの Don't stop it music... という文言を本当にこの
singer-songwriter が書いたんだということを前提として言った。

もしもこの singer-songwriter が実際に書いた歌詞が
一般には公開されておらず、ネット上に溢れかえっている
この歌詞らしきものを有志の(ボランティアの)人たちが
自分で音声を聴いて書きとったに過ぎないのであれば、
それを書き込んだ人たちがみんな間違って聴き取っているに
過ぎないという可能性もある。

ひょっとしたら singer は Don't stop the music と
発音しているのかもしれない。the をみんなが it と
聴き取って、そのヴァージョンが巷に溢れているだけかもしれない。
0791名無しさん@英語勉強中 (スププ Sdba-Y2Y+)
垢版 |
2023/02/01(水) 23:41:18.78ID:TdpWwzhFd
これの(4)なんですが
stereotypedとthoughtlessだとどちらが答えになりますかね。
showing以降は分詞構文だからtheyが主語って考えなのか、前のcharactersにかかる分詞なのかで解答分かれる気が
0794名無しさん@英語勉強中 (アタマイタイー MMff-ZHn6)
垢版 |
2023/02/02(木) 07:07:19.85ID:q41IkPBpM0202
>>744 >>747
こんな事言って勝ち誇ってるみたいよ

113 名無しさん@英語勉強中 (ワッチョイW 3b96-ehKl) 2023/01/30(月) 13:14:31.41 ID:fzbpGu+x0
使ってるわw
「洋服の組み合わせ」なんだから「服装」で問題ないだろ
「服装」でない、ということはコーディネートの英訳がoutfitではおかしいということだよな?
0795The OED Loves Me Not (アタマイタイー 4e89-2biX)
垢版 |
2023/02/02(木) 07:11:58.13ID:dAV6G09V00202
>>791-792
> stereotypedとthoughtless 
> showing以降は分詞構文だから
>> theyが主語って考えなのか、前のcharactersにかかる分詞なのかで解答分かれる気が
======

そういう問題ではなく、別の理由により stereotyped が正解。その理由を、このあとで説明する。
0796The OED Loves Me Not (アタマイタイー 4e89-2biX)
垢版 |
2023/02/02(木) 07:30:45.21ID:dAV6G09V00202
>>791-792
(1) 原文(文脈):
Critics complained that cartoons had no educational value
and pointed out that they often included violence
and (4 番 -- stereotyped? thoughtless?) characters,
showing a social or ethnic group in a prejudiced way.

(2) まずは、cartoons や娯楽小説や一般の雑誌の記事などを
批判する文脈で
... cartoons have no educational value...
cartoons often include violence and (XYZ) characters.

というくだりがあったら、そこには必ずと言っていいくらいに
stereotyped という言葉が出てくる。これはもう、英語ネイティブの
間ではほとんど自動的と言ってもいい。

さらに、そのあとを読むと、ご丁寧にもそれを言い換えて
showing a social or ethnic group in a prejudiced way
と書いている。この1行は、stereotyped を別の言葉に
paraphrase しているに過ぎない。

以上の理由で、この問題の解答としては、
thoughtless なんていう中身の薄い、インパクトの弱い
言葉ではなくて、sterreotyped しかあり得ない。
0797The OED Loves Me Not (アタマイタイー 4e89-2biX)
垢版 |
2023/02/02(木) 07:37:19.71ID:dAV6G09V00202
>>791-792
ところで、当然のことながら、構文などは次のようになっている。

(1) Critics --- これが全体の主語

(2) complained that cartoons had no educational value --- Critics を主語とする動詞の1つ目

and

(3) pointed out that they (= cartoons) often included violence and [stereotyped / thoughtless] characters, --- Critics を主語とする動詞の2つ目

(4) showing a social or ethnic group in a prejudiced way.
   --- cartoons を主語とする分詞構文。だから、showing の代わりに thereby showing と書いても、ここでは同じような意味となる。
0798名無しさん@英語勉強中 (アタマイタイー Sd92-aLix)
垢版 |
2023/02/02(木) 07:47:41.84ID:tgowv4N5d0202
>>794
 > 「洋服の組み合わせ」なんだから「服装」で問題ないだろ

日本語に置き換えると完全に「『着こなし』は『服装』という意味!そういう意味で使われている!そういう意味になった!言葉は変わるもの!ヘンカ! ヘンカァァァァ! イギギギギ」みたいな状態。
無意味な屁理屈。
0799名無しさん@英語勉強中 (アタマイタイー Sdba-TtUP)
垢版 |
2023/02/02(木) 07:58:15.75ID:8L4tSt96d0202
似た言葉で言うと「コーディネートは服装という意味」って「デザインは服装という意味」って言ってるようなものだよね
確かにかっこいい服装を決めるのもデザインの一種という考え方は出来るかもしれないけどさ…
0800名無しさん@英語勉強中 (アタマイタイー Sdba-Y2Y+)
垢版 |
2023/02/02(木) 08:44:39.96ID:IieTxH5bd0202
>>795

>>796
ありがとうございます
showing以降がcharactersにかかっていないのであれば、空所に入るのはviolenceと並列関係になる単語ってことでthoughtlessって考えはダメですかね?

ネイティブがよく使うフレーズっていうのは一旦置いておいたとして笑
0801The OED Loves Me Not (アタマイタイー 4e89-2biX)
垢版 |
2023/02/02(木) 08:52:24.04ID:dAV6G09V00202
>>800
すでに僕がくどくどと書いたように、

>>さらに、そのあとを読むと、ご丁寧にもそれを言い換えて
showing a social or ethnic group in a prejudiced way
と書いている。この1行は、stereotyped を別の言葉に
paraphrase しているに過ぎない。

というわけだ。それ以外に僕の回答はない。
日本人は除外して、少しばかり英語のできる人に
いろいろ尋ねてみたらいい。この前後関係をきちんと
読み取っている人で、thoughtless を選ぶ人なんていない。

ただし、日本人に尋ねたらダメ。日本人は文法書と
辞書しか読まない人でも上級者を気取ってるから。
0802The OED Loves Me Not (アタマイタイー 4e89-2biX)
垢版 |
2023/02/02(木) 08:53:31.22ID:dAV6G09V00202
>>800
>>ネイティブがよく使うフレーズっていうのは一旦置いておいたとして笑

なんでいちいち「笑」をつけるんだよ?いちいち人を馬鹿にしないと気が済まないのか?
0805名無しさん@英語勉強中 (アタマイタイー Sdba-Y2Y+)
垢版 |
2023/02/02(木) 11:21:31.18ID:IieTxH5bd0202
>>801
僕が言いたいのはそのshowing〜のところは〇〇charactersに掛かってるわけではなく主語が別にあるのだから、そこがstereotypedの言い換えだという主張がイマイチしっくりこないということです
ネイティブがよく使う表現だというのは理解しました。
0806The OED Loves Me Not (アタマイタイー 4e89-2biX)
垢版 |
2023/02/02(木) 11:58:43.68ID:dAV6G09V00202
>>805
(1) they (= cartoons)
(2) often included violence and [stereotyped / thoughtless] characters,
[thereby] showing a social or ethnic group in a prejudiced way.

もしも chaeracters そのものを a social or ethnic group in a prejudiced way を
表しているんだったら、showing じゃなくて
representing とか being とか that is とかいう言葉をつけると思う。

しかしここでは、単に showing としか言っていない。ということは
showing は they (= cartoons) を主語としているということが
わかる。(もしわからないとしたら、それはまだ英文を読んできた経験が浅いからだ。)

と言うわけで、こういうことは理屈とか文法でゴチャゴチャ言う人が多いが、
英文を読んだ経験がある程度以上にあれば、この単語が来ているからこれは
これにかかっていて、したがってこういう意味を筆者は言い表してるんだな、ということがわかる。

文法とか辞書だけで、あとは理屈をこねまわしさえすれば英文の真意がわかるなどというのは、迷信だ。
だからこそ僕は、文法と辞書だけで英文解釈しようとしてる連中をいつも揶揄している。
文法なんてものは、できて当たり前で、そのあとに読書経験を豊富に積み重ねないと英文は読めない。
0807The OED Loves Me Not (アタマイタイー 4e89-2biX)
垢版 |
2023/02/02(木) 12:02:07.55ID:dAV6G09V00202
>>806
>>(2) often included violence and [stereotyped / thoughtless] characters,
[thereby] showing a social or ethnic group in a prejudiced way.

>>もしも chaeracters そのものを a social or ethnic group in a prejudiced way を
表しているんだったら、showing じゃなくて
representing とか being とか that is とかいう言葉をつけると思う。
=======

上のように書いたが、言葉足らずだったかもしれん。次のように言い直す。


もしも characters そのものが a social or ethnic group を表しているんだったら

characters, who represent a social or ethnic gbroup in a prejudiced way
とでも書くだろう。
0809名無しさん@英語勉強中 (アタマイタイー b7e2-qUWM)
垢版 |
2023/02/02(木) 13:50:55.99ID:4UdNwtpk00202
>>808
200円からDEcrease it TO to 10%だと、20円になる(元値が10%になるまで減らす)。

200円からDEcrease it BY 10%だと、180円になる(10%の差分を元値から削る)。

INcrease/raiseならその逆。

0.25 percentageの後にPOINTSとあるように、その数値分を現行の数値(金利)にプラスする。今が0.25ならそれに0.25足して0.5になる。
0811名無しさん@英語勉強中 (アタマイタイーW 8a8a-FK61)
垢版 |
2023/02/02(木) 17:46:13.43ID:A+XkJXlE00202
>>792
俺はthoughtless だと思う
stereotypedが入るとくどい感じがする。直後で同じことをまた言うことになるから
0812名無しさん@英語勉強中 (アタマイタイーW 8a8a-FK61)
垢版 |
2023/02/02(木) 18:00:44.95ID:A+XkJXlE00202
正直どっちが入っても良いというか、どっちが入ってももう片方も完全に間違ってるとは言えないあんまり良くない問題だと思うけど
0815名無しさん@英語勉強中 (アタマイタイー a796-fO7+)
垢版 |
2023/02/02(木) 20:41:47.13ID:5rPdE8cA00202
>>811
「thoughtlessなキャラ」がいない漫画なんて、つまり、みんな立派なキャラばかりの漫画なんて面白くないよね。

「漫画にはthoughtlessなキャラが出てくる」という理由で漫画を批判する人がいたらアホでしょw
0816名無しさん@英語勉強中 (アタマイタイー a796-fO7+)
垢版 |
2023/02/02(木) 20:47:59.02ID:5rPdE8cA00202
>>806
いくら自分が文法苦手だからといって、そういう言い方をしちゃだめだよ。
0817名無しさん@英語勉強中 (アタマイタイー a796-fO7+)
垢版 |
2023/02/02(木) 20:58:02.23ID:5rPdE8cA00202
>>806
thoughtless(軽率な)がなぜダメかというと、「軽率なキャラが出てくるという理由で漫画を批判する人などいない」から。
天才バカボンを見て、「あんなバカを出すな。みんな聖人キャラにしろ」っていう人がいるのか?
そんなことを言う人はいないのだから、thoughtlessは、明らかに間違い。

それを、後ろにある分詞構文が~と文法とか英文にこだわって理由を考えるから、人を納得させる答えが出てこないのだ。
もっと上位概念である「常識」で判断すれば、答えは簡単に出てくる。
0818名無しさん@英語勉強中 (アタマイタイー a796-fO7+)
垢版 |
2023/02/02(木) 21:01:51.94ID:5rPdE8cA00202
今日の教訓
聖人君子しか出てこない漫画なんて絶対に売れない

※「漫画には聖人君子だけ出せ」「thougtlessなキャラは出すな」などという人などいない。
0819名無しさん@英語勉強中 (アタマイタイー Sdba-Y2Y+)
垢版 |
2023/02/02(木) 21:24:37.77ID:IieTxH5bd0202
>>806
うーん、僕はshowing以降はtheyに掛かってるとハナから思ってるんですけど意図したことが伝わってないようですね
日本語の読解は苦手か?
0820名無しさん@英語勉強中 (ワッチョイ a796-fO7+)
垢版 |
2023/02/02(木) 22:27:47.89ID:5rPdE8cA0
>>792の文をよく読んでみた。そして、なぜ、「thoughtless」が間違いなのかを真剣に解説したい。

その文は、「アニメは教育的な効果がなく時に暴力的であったり( )であったりして有害ですらある」と主張している。

そして、(  )には「軽率なキャラ」が入るのか「固定観念にとらわれたキャラ」が入るのかが問われている。

「軽率なキャラ」は教育的な寓話にも出てくるのであるから「軽率なキャラが出てくること」が教育的でないことの理由にはならない。
しかし、「固定観念にとらわれたキャラ」は、この世の偏見を助長する悪という意味しか持たない。よって、批評家が批判しているのは、「偏見的キャラ」である。
0821名無しさん@英語勉強中 (ワッチョイW 8a8a-FK61)
垢版 |
2023/02/02(木) 22:31:57.10ID:A+XkJXlE0
>>815>>817
それはviolence にも同じことが言えるのでそれは理由にならない
いやいやviolence は問題だろうと言うのなら、thoughtless にも同じことが言える
でもじゃあstereotypedだと絶対ダメな文かと言うとそういうわけでもないので、これは問題が良くない
要するに問題作成者が無理やり紛らわしくしようとしてどっちでも通るような単語を用意して選択肢にしただけなので、あまり深く考える必要自体が無い
0822名無しさん@英語勉強中 (ワッチョイW 8a8a-FK61)
垢版 |
2023/02/02(木) 22:33:01.62ID:A+XkJXlE0
>>820
真面目モードになったところ悪いんだが、君はね、thoughtless って単語の理解自体が間違ってるのよw
0823名無しさん@英語勉強中 (ワッチョイ a796-fO7+)
垢版 |
2023/02/02(木) 22:51:30.83ID:5rPdE8cA0
>>822
いやいや、「軽率な」でも「思いやりのない」でも同じ。教育的な寓話にも、善悪の対比のために、「思いやりのないキャラ」が出てくるし。
「思いやりのないキャラ」「軽率なキャラ」が出てくることが教育的でない理由にはならない。

「固定観念にとらわれたキャラ」と「思いやりのないキャラ・軽率なキャラ」、どちらが、より悪質で批評家が批判しそうかと言えば、前者である。
0824名無しさん@英語勉強中 (スップー Sd5a-Y2Y+)
垢版 |
2023/02/02(木) 22:58:54.81ID:IJSlfUNPd
>>823
そんな君の「予想」は誰も聞いてないんだよ
0825名無しさん@英語勉強中 (ワッチョイ a796-fO7+)
垢版 |
2023/02/02(木) 23:01:33.24ID:5rPdE8cA0
「固定観念にとらわれたキャラ」と「思いやりのないキャラ・軽率なキャラ」、どちらが、より悪質で批評家が批判しそうかと言えば、前者である。

これが感じ取れない人がこの世にいるなんて驚きだよw


教育的な寓話には固定観念に捕らわれたキャラは出てこない。(人種的偏見など)
教育的な寓話には、生々しい暴力的表現は出てこない。
しかし、教育的な寓話には「思いやりのないキャラ・軽率なキャラ」も、善にスポットライトを与える対照物としてたくさん出てくる。

絶対的に排除すべきなのはどっちなの?「固定観念」?「思いやりのないキャラ」? まだ、わからんかな~?
0826名無しさん@英語勉強中 (ワッチョイ a796-fO7+)
垢版 |
2023/02/02(木) 23:04:42.95ID:5rPdE8cA0
>>824
そんな小学生レベルの反論しないで、ちゃんと理由を述べなさいよ。
0827名無しさん@英語勉強中 (ワッチョイ a796-fO7+)
垢版 |
2023/02/02(木) 23:07:58.32ID:5rPdE8cA0
頭のいい人なら、>>825を読んだら、「固定観念にとらわれたキャラ」が正解だと簡単に理解できる事柄
0828名無しさん@英語勉強中 (ワッチョイW 5a26-Xl9m)
垢版 |
2023/02/02(木) 23:13:17.88ID:tWQ0F1N50
>>792
批評家たち(中略)が指摘するのは、マンガが暴力や(固定観念的に作成された)キャラを含んでいて、
社会的・民族的な集団を偏見に満ちた形で描写することだ。

thoughtlessだと「周りの人に気を配らない」キャラ。これだと批判材料として微妙かな?
0830ブタ耳 (ワッチョイW 3ee3-FGvc)
垢版 |
2023/02/03(金) 00:26:22.16ID:I3thS4tc0
「固定観念にとらわれた」というのは、人物の内面的性格に関わる言い方だが、stereotypedというのはそんな内面的なものではなくて、外部にある定型を押し付けられた「型にはまった」人物造形のことだろう。
いわゆる勧善懲悪的な、いかにも正義の味方、いかにも悪者的な、使い古された陳腐な人物造形。
最近のアニメは人物造形も複雑で、そんな単純なstereotypeはあんまり見かけないような気がする。
一体いつの時代のどんなアニメを念頭に置いているのだろうか?
0831名無しさん@英語勉強中 (ワッチョイW 8a8a-FK61)
垢版 |
2023/02/03(金) 00:39:56.91ID:EV0ezwNf0
>>825
君のはとにかく全部
「批評家は〜と思いそうだと俺は個人的に思っている。なぜなら教育的寓話は〜な感じだと俺は個人的に思っているからだ」というだけの話なので、
それだと他の人が「いや俺は逆だと個人的に思う」の一言で全部ひっくり返っちゃうのよ
小学生の国語の解答根拠みたいな感じなので、そもそも話になってない
英語的にはさっきも言ったけどthoughtlessがよくわかってないのでそこが理解の進まない原因
そもそもさっきも言ったようにこれどっちが入っても別に良いからどっちでもいいのよ
0832名無しさん@英語勉強中 (スップー Sd5a-Y2Y+)
垢版 |
2023/02/03(金) 00:44:30.57ID:2DZWQs/Nd
>>826
お前の回答の根拠が小学生並みだよバカ
俺はこう思うから!しか言ってないだろ
0833名無しさん@英語勉強中 (ワッチョイ a796-fO7+)
垢版 |
2023/02/03(金) 00:44:37.86ID:UmJGH9oa0
>>830
そういう意味じゃないよ。日本人を出っ歯でカメラを首からぶらさげて集団で行動するように描く類いのことだよ。ChatGPTは、この件では、正しい答えを出してるよ。

私 「What does "stereotyped cartoon character" mean?」

ChatGPTの答え
A "stereotyped cartoon character" refers to a character in animation that is designed to conform to a widely recognized, oversimplified representation of a particular group, often based on a specific trait or personality. Such characters are usually used as comedic devices and often perpetuate negative and stereotypical images of the group they represent.
「固定観念に捕らわれた漫画のキャラクター」とは、広く認識されている特定の集団の過度な表現に適合するように作られたアニメーションのキャラクターで、しばしば特定の特徴や性格に基づいているものを指します。このようなキャラクターは通常、コメディーの道具として使われ、しばしば、代表する集団の否定的でステレオタイプなイメージを永続させるものです。
0834名無しさん@英語勉強中 (スップー Sd5a-Y2Y+)
垢版 |
2023/02/03(金) 00:48:47.39ID:gcl6Fy0kd
>>831
めちゃくちゃ賢そう
0835名無しさん@英語勉強中 (ワッチョイ a796-fO7+)
垢版 |
2023/02/03(金) 00:55:50.44ID:UmJGH9oa0
>>831
では、thoughtlessがどういう意味か言ってみな。君のほうこそ、わかってないから、なにも言えないんだろ?
俺がわかってない理由を述べてくれる? 理由もなく批判するのは、愚か者のすることだぞ

>>832
同じ時間に出てきて、同じ論調、自演っぽい・・・まあ・・・いいか
この問題の正解がわからない時点で、おまえのほうがバカなのは確定してるんだよw
くやしかったら、なぜ、thoughtlessのほうが優勢なのか説明してみろ。「~と思う」でいいからさ。
それも、説明できないなら小学生レベルの反論と言われても仕方ないだろ
0836名無しさん@英語勉強中 (ワッチョイ a796-fO7+)
垢版 |
2023/02/03(金) 00:56:44.84ID:UmJGH9oa0
>>834
もう完璧に自演www 
0837名無しさん@英語勉強中 (アウアウウー Sa47-BLd4)
垢版 |
2023/02/03(金) 02:08:31.56ID:hAIcQEF/a
どっちがより適切かといえば、stereotypedで決まりでしょ👩‍🏫 thoughtlessを使うんだったらもっと適切な形容詞があるはず。例えばwickedとか。だから、thoughtlessは不自然きわまりない
0838名無しさん@英語勉強中 (スフッ Sdba-GjX+)
垢版 |
2023/02/03(金) 05:19:01.56ID:KEOkkMgSd
>>836
ライフラインは和製英語だと思ってそう
0839The OED Loves Me Not (ワッチョイ 4e89-2biX)
垢版 |
2023/02/03(金) 05:38:58.08ID:MlNt48rD0
普段は借りて来た猫みたいにずっと押し黙って震えている連中が、今回みたいに本当は英語そのものの
問題なのに日本語だけの問題にすり替えてしまって、日本語の感覚だけで物を言い続けているな。

sterreotyped を日本語に訳して、その日本語についての感覚で物を言ってはいけないのだ。
あくまでこれは英語についての問題だ。再び英語原文を示す。

>>791-792
原文(文脈):
Critics complained that cartoons had no educational value
and pointed out that they often included violence
and ●stereotyped / thoughtless● characters,
showing a social or ethnic group in a prejudiced way.

最後に showing a social or ethnic group in a prejudiced way と
書いてあるだろ?そうなるとその前には反射的に stereotyped が
来るに決まってるんだよ。

stereotype, stereotyped, stereotyping と言えば、それは
gender, race, sexual orientation などが絡むに決まっている。
本当はもっといろんなことが関わるんだろうけど、特に
racial stereotyping や gender stereotyping や
sexual orientation に関わる stereotyping に対して、
少なくとも英米では常に喧嘩が起こる。

ただそれだけの問題だ。こんな文脈で thoughtless なんていう
間抜けの単語を選ぶ奴の気が知れん。それくらいに、
英米では stereotyping という言葉にインパクトがある。
それを日本語に訳したらダメなんだ。あくまで英語としての
その言葉のインパクトの問題を言っている。
0840The OED Loves Me Not (ワッチョイ 4e89-2biX)
垢版 |
2023/02/03(金) 05:46:30.15ID:MlNt48rD0
stereotype, stereotyping, stereotyped という言葉がどういう
ふうな文脈で使われているかを知りたければ、
その言葉が使われている「英文の」新聞雑誌の記事を
たくさん読み流したらいい。

それを日本語に訳した記事なんて読んだって仕方がない。
日本語の「型にはまった」などの訳語には、そんなニュアンスはない。

少なくとも大多数の日本国籍を持った人たちのあいだでは、
日本に住んでいる限り、少なくとも racial stereotyping によって苦しんだり
怒ったりする場面はないに等しい。だから自然に
stereotype という言葉を racial な文脈で使うことは少ない。

gender の問題にしても、日本では英米に比べるとはるかに静かだ。
だから日本語で stereotiype, stereotyped, stereotyping の
語感について論議したって無駄だ。
0841名無しさん@英語勉強中 (ワッチョイW 8a8a-FK61)
垢版 |
2023/02/03(金) 07:25:42.68ID:EV0ezwNf0
あーーー、わかった
やっとすっきりした。
答えはstereotyped。
ただしその理由は、これまでの回答で誰も気づいていない理由。
0842名無しさん@英語勉強中 (ワッチョイ a796-fO7+)
垢版 |
2023/02/03(金) 07:52:15.14ID:UmJGH9oa0
>>841
理由言ってみなよ。どうせ、突っ込みが入るような理由だろうけど

いずれにしても、stereotypedとthoughtlessのどちらでもいいと言ってる人たちが理解力が高くない人たちであることが判明してよかった
0843名無しさん@英語勉強中 (ワッチョイ 1af0-lklL)
垢版 |
2023/02/03(金) 09:03:18.73ID:VlKFarin0
a social or ethnic group がなかったら、どっちも意味はつながるが、
偏見とのかかわりだから、ステレオタイプの方だろうな。
0844名無しさん@英語勉強中 (ワッチョイW 8a8a-FK61)
垢版 |
2023/02/03(金) 09:15:41.34ID:EV0ezwNf0
そもそも今んとこ誰も理由言えてないからな
「多分こっち!」って言ってるだけだから解答として話にならない
0845名無しさん@英語勉強中 (ワッチョイ a796-fO7+)
垢版 |
2023/02/03(金) 09:24:14.68ID:UmJGH9oa0
>>844
だから理由言ってみなよ。どうせ、また、突っ込みが入るような理由だろうけどw

理由を示さないなら、お話にならないね
0846名無しさん@英語勉強中 (ワッチョイ a796-fO7+)
垢版 |
2023/02/03(金) 09:51:08.76ID:UmJGH9oa0
俺は、最初から「stereotyped」が正解であると少しの疑いもなく理解できていた。「thoughtless」なんてありえんよ
0847名無しさん@英語勉強中 (ワッチョイW 8a8a-FK61)
垢版 |
2023/02/03(金) 15:00:32.14ID:EV0ezwNf0
解答根拠が教育的な寓話は普通に間違いですww
0848名無しさん@英語勉強中 (ワッチョイ a796-fO7+)
垢版 |
2023/02/03(金) 15:09:29.02ID:UmJGH9oa0
>>847
理由がないので却下

一般的に言って、理由を示さないで反論をする人たち = 頭のよくない人たちwww
0850名無しさん@英語勉強中 (スーップ Sdba-Y2Y+)
垢版 |
2023/02/03(金) 21:17:22.34ID:bOZBR0LDd
馬鹿vs馬鹿
0853名無しさん@英語勉強中 (ワッチョイ 4f96-EOzK)
垢版 |
2023/02/04(土) 12:37:13.14ID:+W0dx1T90
「我々の製品を10000個以上お買い上げいただくことに同意いただける場合は、お値引きのご要望をお受けします。」
上記の内容を英語で表すのに、以下の文を考えました。
In the case that you agree to purchase our product 10,000 pcs or more, we would accept your request to discount the product.

お尋ねしたいのは、文頭の「In the case that」という表現です。
上の例文では、In the case thatのthat以下は、もしあなたが我々の製品を10000個以上買うのなら、という単なる条件を表しているつもりですが、
このような意味でIn the case thatという表現は使用できるのでしょうか?
(Ifを使えば良いのかもしれませんが、Ifは文の他の部分で使用することが多く、できればIfを使用せずに表現したいと考えています)
また、ネットで検索するとIn the case whereという表現も見かけるのですがこれはIn the case thatとどう違うのでしょうか?

以上、是非ご教示いただきたく、よろしくお願いいたします。
0854名無しさん@英語勉強中 (ワッチョイ 3ff0-4osW)
垢版 |
2023/02/04(土) 13:24:07.04ID:XqD7FMmu0
>>852
生徒がミーティングのためディレクターのオフィスを使わしてもらう
0855名無しさん@英語勉強中 (ワッチョイ 3ff0-4osW)
垢版 |
2023/02/04(土) 13:53:03.80ID:XqD7FMmu0
Providing you purchace 10,000 pieces or more of the product, we will discount it as you requested.
0857ブタ耳 (ワッチョイW 7fe3-unDJ)
垢版 |
2023/02/04(土) 16:42:56.31ID:wfUUdyLc0
>>856
自動翻訳で有名なDeepL が最近始めたサービスにDeepL writeというのがある。
https://www.deepl.com/write-mobile
テキストを入力すると瞬時に添削してくれる。
色々なパターンを提示してくれる場合もあり、推敲するには非常に便利。

>>853の例文を入力してみたところ、4つの添削例が提示された。
その中では次のものが一番分かりやすいと思った。

We will accept your request for product discount if you agree to purchase 10,000 pieces or more of our product.
0858ブタ耳 (ワッチョイW 7fe3-unDJ)
垢版 |
2023/02/04(土) 17:05:58.29ID:wfUUdyLc0
In the case where, In the case that
の違いについて言えば、whereが関係副詞、thatはその代用と考えるのが妥当なところだろう。
thatの方がより口語的なのかもしれないが、in case ~ という表現自体が文語的なので、in case に続くものとしてはwhereの方が好まれるのではないかと思う。

GoogleのNgramで比較すると、やはりwhereの方が多いようだ。
https://imgur.com/a/gCDT4bm
https://books.google.com/ngrams/
(この結果には、thatが関係代名詞の主格である場合なども含まれるので、実際には関係副詞的thatの用例はもっと少ないはず)
0859ブタ耳 (ワッチョイW 7fe3-unDJ)
垢版 |
2023/02/04(土) 17:30:10.74ID:wfUUdyLc0
Rewrite this sentence without using the conjunction “if”.
Present several examples.
とChatGPTに指示して得られた出力結果。

1. By purchasing 10,000 or more pieces of our product, you'll qualify for a discounted price.

2. We're pleased to offer a special price on our product for orders of 10,000 pieces or more.

3. When you commit to buying 10,000 or more pieces of our product, a discount will be applied.

4. A discounted rate will be provided for orders of 10,000 or more pieces of our product.

5. You'll receive a lower price on our product for orders of 10,000 pieces or more.
0860名無しさん@英語勉強中 (ワッチョイ 8f32-G/L/)
垢版 |
2023/02/04(土) 23:05:33.55ID:azGreof+0
last night's soccer game は the soccer game last night と
the を付けて前後の語順を変えることが可能だと思いますが
この名詞の語順変更は日時や時間帯を表す単語の場合だけでしょうか?
0861名無しさん@英語勉強中 (ワッチョイ 3f69-zLlH)
垢版 |
2023/02/05(日) 00:17:24.58ID:NfYhg8510
There is nothing more important to maintaining dental health than brushing your teeth twice a day.
この文章ですが不自然に感じます
There is nothing important than
の文の応用だと思うのですが、important とthan の間にto maintaining dental health の文が入っているのが納得いきません
ネットで調べても例文が出てこず
There is nothing more important than brushing your teeth twice a day to maintaining dental health

だったら納得するんですが、間に入っているのは倒置とか何か文法名があるんでしょうか?
文法的にどう解釈すればいいのでしょうか?
0862名無しさん@英語勉強中 (ワッチョイ 4f96-EOzK)
垢版 |
2023/02/05(日) 00:36:53.18ID:HJAjakCb0
>>857,858,859
レスありがとうございます。
「~の場合」、という意味で使用するのなら、in the case whereと表現する事が多いということでしょうか?
それにしてもChatGPTというのはすごいですね。
”Ifを使わずに”などとも指定できるんですね。
私も試してみたいです。
また、よろしくお願いします!
0864名無しさん@英語勉強中 (ワッチョイ 3ff0-4osW)
垢版 |
2023/02/05(日) 04:46:23.10ID:Bb4zrRVl0
>>861
なんで後ろに持っていったら納得するの?
0867名無しさん@英語勉強中 (アウアウウー Sa93-7XYW)
垢版 |
2023/02/05(日) 08:41:34.65ID:mr/vvCXDa
be posted to B Bに赴任する、はbe promoted to BBに昇進する、と異なり余り使われない表現なのでしょうか?
文法的にはpost A to Bの受け身として誤りではないと思うのですが。
Tex was posted to Moscow. テックスはモスクワに赴任した。
英辞郎に載ってなかったのと、ネット検索しても「〜に投稿された○○」のような過去分詞の表現ばかり出てきたので質問しました。
0868名無しさん@英語勉強中 (ワッチョイ 7f84-uzHC)
垢版 |
2023/02/05(日) 08:52:50.48ID:xt5uEpHO0
>>861
新情報旧情報の語順で、一般論→個別論となったと思われます。
0870The OED Loves Me Not (ワッチョイ 7f89-EOzK)
垢版 |
2023/02/05(日) 09:05:12.06ID:AuKSsDY90
>>867
(1) When ★he was posted to London★ in 2004, he was disappointed to find British bankers didn’t do that sort of thing.
https://www.theguardian.com/lifeandstyle/2015/sep/13/britains-takeaway-revolution
(The Guardian という有名なな新聞サイト)

(2) ★He was posted to the firm’s London office★ in 1873, where he began collecting English prints and illustrations.
https://blog.oup.com/2015/07/vincent-van-gogh-life-death/
(The Oxford University Press によるサイト)

この他、いくらでも用例は見つかると思う。
0871名無しさん@英語勉強中 (ワッチョイ 0f96-F1up)
垢版 |
2023/02/05(日) 10:58:04.15ID:ASeKZCB00
>>861
https://eow.alc.co.jp/search?q=better+at+sports+than

Jane is better at sports than Mike.
「be good at」(~が得意だ)の比較級

この文で、at sportsを後ろに持っていったら分かりにくいでしょ。それと同じ。
0872名無しさん@英語勉強中 (ワッチョイW 3f26-3Qno)
垢版 |
2023/02/05(日) 11:45:51.82ID:p2oaYt8N0
>>861
比較級を作る場合、(比較の相手)は最後に残して、場所やら時間やらの入った主語述語を書き切るのが普通

①今日の東京は(熊谷より)暑かった。
→It was hotter in Tokyo today than Kumagaya.

②私は(去年のアメリカでより)今年の日本での方がずっと沢山の本を買った。
→I bought many more books in Japan this year than in America last year.

歯の健康を保つには(1日2回磨くことより)大切なことはない。
↑主語述語をルール通り優先してるだけ
0873名無しさん@英語勉強中 (オイコラミネオ MM03-jaVN)
垢版 |
2023/02/05(日) 16:26:37.02ID:wcAoXz94M
オンライン英会話、なるだけ柔軟に対応できるところを探しています。
例えば空いた時間(30分くらい前)でもすぐ予約など取ってレッスンができるようなところとか。
質も含めて候補として考えるべきところを教えて下さい。
0874名無しさん@英語勉強中 (アウアウウー Sa93-7XYW)
垢版 |
2023/02/05(日) 18:58:40.36ID:LW6JhQQ+a
flatterお世辞をいう flutter羽ばたきする、がゴッチャになるのですが、どっちがどっちか分かりやすい覚え方は有りませんか?
語源系よりはできればaやuに着目した覚え方がイイです。
0876The OED Loves Me Not (ワッチョイ 7f89-EOzK)
垢版 |
2023/02/05(日) 19:33:52.00ID:AuKSsDY90
>>874
そんなのは、適当に自分でシャレを作ればいいと思う。たとえば

flatter (お世辞を言う) --- fl A tter
--- ふらっと (flat --- A) 彼女の家に立ち寄って、お世辞を言ってやったら、
あいつは怒ってしまい、俺は振られ (flare -- A) ちゃって、どうしようもなかった。


flutter (羽ばたきする)
--- きれいな蝶々(ちょうちょ)がいたから、俺はフルート (flute -- U) を取り出して
必死できれいな音楽を演奏してやったら、あいつは羽ばたきして
喜んで夕焼け空へと帰っていったから、俺の心は震えた (fl U eta)。
0877The OED Loves Me Not (ワッチョイ 7f89-EOzK)
垢版 |
2023/02/05(日) 19:38:44.18ID:AuKSsDY90
>>861 について

(1) >>868
>>新情報旧情報の語順で、一般論→個別論となったと思われます。

(2) >>872
>>比較級を作る場合、(比較の相手)は最後に残して、

この二人はまったく違ったことを言っているように見えるけど、
実はこの二つは同じコインの裏表でしかない。

(2) の「比較の相手は最後に残す」のはなぜかというと、
「比較の相手」が新情報であるのが普通であり、
新情報だから最後に残すのは当然。

というわけで、質問者はこの二人の意見をすんなり頭に
入れられるはずだ。
0878The OED Loves Me Not (ワッチョイ 7f89-EOzK)
垢版 |
2023/02/05(日) 19:42:29.26ID:AuKSsDY90
つまり、
>>861
(1) >>There is nothing more important to maintaining dental health than brushing your teeth twice a day.

と言っているとき、"to maintaining dental health" というのは、ここでは旧情報であるはずだ。
「葉の健康」に関してすでに前の方で議論していたに違いない。そしてそのあとで
今回の原文 (1) が出て来たはずなのだ。

しかし比較の相手である「1日に二回、歯を磨く」というのは、新情報、つまり聞き手からすれば
ここで初めて聞く話であるはずだ。(この文の前ではまだそんな話をしていなかったはずだ。)
新情報だから、文の最後に持ってきた方がわかりやすくなる。
0879The OED Loves Me Not (ワッチョイ 7f89-EOzK)
垢版 |
2023/02/05(日) 19:47:52.14ID:AuKSsDY90
>>861
そして仮に今回の (1) の原文が単独で現れていて、その前にはまるで文脈がなかったとしても、
「歯の健康」というのは普通はみんなの念頭にあると考えるのが社会通念だ。だからこれは
新情報化旧情報かどっちだ、と詰問されれば、「どちらかと言えば旧情報」と答える
人が多くなる。

しかし「1日に二回、歯を磨く」というのは、葉の磨き方にもいろんな形態があり、1日に1回だけの人も
いれば、歯を磨かないで薬品を口に含んでうがいみたいにぶくぶくするだけ、という人もいるし、
いろいろあるはずだ。その中で「1日に二回、歯を磨く」というやり方が新情報なのか旧情報なのかと
詰問されれば、やはり普通の人は「どちらかと言うと新情報」と言うだろう。

そういうわけで、brushing your teeth twice a day が後ろに、そして maintaining dental health が
前に持って来られることが多い、ということになる。
0882名無しさん@英語勉強中 (ワッチョイ 0f96-F1up)
垢版 |
2023/02/05(日) 21:17:46.20ID:ASeKZCB00
>>878
新情報とか旧情報とか関係ないから。

There is nothing more important to maintaining dental health than brushing your teeth twice a day.は、省略しないで書いたら、
「There is nothing more important to maintaining dental health than brushing your teeth twice a day is important to maintaining dental health.」となる。

「There is nothing more important to maintaining dental health than brushing your teeth twice a day (is important to maintaining dental health.)」の(  )内が重複しているから省略されてるだけ。
そして、この手の重複では、後者のほうを省略する。
0883名無しさん@英語勉強中 (ワッチョイ 0f96-F1up)
垢版 |
2023/02/05(日) 21:20:06.52ID:ASeKZCB00
>>861

その文は、>>882で示したように、重複部の省略にすぎない。
0885名無しさん@英語勉強中 (ワッチョイ 8f32-G/L/)
垢版 |
2023/02/06(月) 00:04:45.35ID:bu7TFUXo0
先月の会議:the meeting last month の last month は
後置修飾、つまり「形容詞」として理解していますが
ならば言い換えでは last month meeting とは言わず last month's meeting と
なるのはどうしてでしょうか?
またどうして the last month's meeting とは言わないのでしょうか?
0886名無しさん@英語勉強中 (オッペケ Sr63-ax4o)
垢版 |
2023/02/06(月) 00:20:11.94ID:e/QyV2cur
この筆記体が読めないのですが、何で書いてありますか?
0887名無しさん@英語勉強中 (オッペケ Sr63-ax4o)
垢版 |
2023/02/06(月) 00:21:06.54ID:e/QyV2cur
https://i.imgur.com/yM7Gcrr.jpg

すみません画像貼り忘れました
0888名無しさん@英語勉強中 (ワッチョイ 8f32-G/L/)
垢版 |
2023/02/06(月) 00:44:44.55ID:bu7TFUXo0
>>885
失礼しました、限定詞が2つでしたね
最後の質問は無しです
0892The OED Loves Me Not (ワッチョイ 7f89-EOzK)
垢版 |
2023/02/06(月) 04:54:01.09ID:QRwtEfCN0
>>885
last month が後ろに来ているのは、形容詞の後置修飾じゃなくて
副詞が名詞の後ろからかかっているんだろうよ。

Young people today are....
People these days are....
みたいに。

それはともかく、「なぜ、なぜ」ばかり言っていないで、
自分で何年もかけて考えたらいいんだよ。疑問を持ちながら
新しい英文をどんどん読んでいるうちに、少しずつ解決していく。
0894The OED Loves Me Not (ワッチョイ 7f89-EOzK)
垢版 |
2023/02/06(月) 05:38:31.04ID:QRwtEfCN0
>>893
restroom
Originally: a room (usually in a public building or workplace) set aside for rest and relaxation (now rare). In later use (U.S.): a lavatory in a public building or workplace.

出典:
This entry has been updated (OED Third Edition, March 2010; most recently modified version published online March 2022).

上のように書いてあるから、たぶんアメリカでは restroom と言えばトイレなんだろうよ。アメリカ以外では restroom は
おそらくは今でも休憩室という意味で使われているんだろう。

だからアメリカでは、休憩室のことを lounge とか言っておけばいいだろうと思う。
0895The OED Loves Me Not (ワッチョイ 7f89-EOzK)
垢版 |
2023/02/06(月) 05:43:39.78ID:QRwtEfCN0
>>893
アメリカ以外でも restroom を休憩室(本当に休憩するための部屋)という意味では、今ではあまり使われていないらしいな。
だからやっぱり、アメリカであろうとそれ以外の国であろうと、休憩室としては lounge としておいた方がいいな。
0896名無しさん@英語勉強中 (ワッチョイW cff5-ax4o)
垢版 |
2023/02/06(月) 08:44:43.33ID:Dy7dF5B40
>>890
ありがとうございます♪
0897名無しさん@英語勉強中 (ワッチョイ 8f32-G/L/)
垢版 |
2023/02/06(月) 17:57:37.87ID:bu7TFUXo0
>>892
どうもすいません
街で Today's special menu とか見ると「ん?」とか思っちゃう性分なんですが
むしろそういうのはあまり文法を持ち込まない方がいいのか、そうでもないのか、
どっちか分らなくなる時がありまして
まぁ、Today's special が普通なんでしょうけど
the special menu today は文法的に正しそうだけど、言わないさそうなので
そのへんのことが調べてもなかなか分らないもので、ここで訊いてみました
0898😉三年英太郎🌈 ◆3CZBjOt3.Y (ワッチョイW 3f86-du88)
垢版 |
2023/02/06(月) 18:32:25.40ID:x2G8sWGd0
>>897
💙the special menu [today]

[...] は付加部であり、主要部 menu にとってあってもなくてもいい要素である

💙[today's] special menu

[...] 主要部の指定部(限定詞)であり、結構重要な要素

つまり today は位置によって、主要部との結びつきの度合いが変わるということ
0899The OED Loves Me Not (ワッチョイ 7f89-EOzK)
垢版 |
2023/02/06(月) 18:48:43.99ID:QRwtEfCN0
>>897
the special menu today は、ネット上に次のように一応は出てくる。
本当にこれが正しいと断言はできないけど、間違いだとは
言えないと僕は思っている。

(1) ★The special menu today★ is Crappy Asparagus Soup, Ma’am.
https://pdfcoffee.com/soal-uas-2016b-inggriskls-xiik13-pdf-free.html

(2) ★The special menu today★ is braised lamb shank with mashed potato.
https://www.mail-archive.com/fairfieldlife@yahoogroups.com/msg368420.html

(3) ★The special menu today★ is fried rice with cream, fried chicken, mmm….
chrome-extension://efaidnbmnnnibpcajpcglclefindmkaj/http://repository.ukwms.ac.id/id/eprint/2588/7/Lampiran.pdf
0901The OED Loves Me Not (ワッチョイ 7f89-EOzK)
垢版 |
2023/02/06(月) 18:59:41.07ID:QRwtEfCN0
>>897
the special guest today の guest の部分だけを別の単語に
入れ替えると、これもまたいろいろと用例が出てくる。
この下の表現は、すべてネット上で見つけたもの。

(1) The special guest today is...
(2) The Special Thematic Session today is....
(3) The special freebie of today is.... (ここでは of がついている)
(4) Oiur menu today is....
(5) The menu today is ....
(6) My menu today is....
(7) Our World Food Menu today is....
(8) The food truck menu today is....
(9) The selection today is....
(10) The choice today is....
(11) The dish today is....
(12) The burger today is....
0902The OED Loves Me Not (ワッチョイ 7f89-EOzK)
垢版 |
2023/02/06(月) 19:11:30.74ID:QRwtEfCN0
>>897
他に、これに似た言い回しを探してみる。すべてネット上で見つけたもの。

(13) Our guest today is....
(14) Our song today is....
(15) The guest today is....
(16) Our book today is....
(17) Our choice today is....
(18) Our whiskey today is....
(19) Our cocktail today is....
(20) The cocktail today is....
(21) Our star today is....

どうもこの
Our [名詞] today is....
という言い回しは、例えばテレビのインタビュー番組とか
バラエティ番組で「今日のスペシャルゲストは(今日の特別な~は)
~さんです(~という料理です、~という飲み物です)」とかいうふうに
視聴者に対して紹介するときの決まり文句のように見える。
0903The OED Loves Me Not (ワッチョイ 7f89-EOzK)
垢版 |
2023/02/06(月) 19:16:46.25ID:QRwtEfCN0
>>897
今までの
Our * today is...
という表現の today を tonight に変えても、
いろんな用例が見つかる。これもたぶん
テレビ番組の司会者の常套句なんだろうな。・

(22) Our star tonight is....
(23) Our book tonight is....
(24) Our menu tonivght is....
(25) Our dish tonight is....
(26) Our burger tonight is....
(27) Our choice tonight is....
(28) Our cocktail tonight is....
0904名無しさん@英語勉強中 (ワッチョイ 8f32-G/L/)
垢版 |
2023/02/06(月) 19:18:06.99ID:bu7TFUXo0
ありがとうございます 大変勉強になります 感謝です
0905名無しさん@英語勉強中 (ワッチョイ 4f96-qdck)
垢版 |
2023/02/07(火) 12:44:53.32ID:8H2KD84n0
i am wrong that ~ て表現て間違いかな
何件かコーパスにあるけど
This does not prove that I am wrong that this tends to be a systematic problem.
I am wrong that Contessa is hatenomor or eap.
You show me where I am wrong that less people were working after 8 years of Bush.
0906The OED Loves Me Not (ワッチョイ 7f89-EOzK)
垢版 |
2023/02/07(火) 13:04:22.78ID:sE14Wq3W0
>>905
I am wrong that S V. というのは、僕自身は見たことがない。しかし次のような
言い回しなら見るし、辞書にもそういう用例は見つかる。

(1) I am wrong in thinking that S V.

(2) It is wrong for me to feel that S V.

(3) I thought that he was a teacher. But I was wrong.

(3) のような言い回しが最も頻繁に見聞きするから、それがたぶん
自然なんだろうと思う。だから、なるべくそのような文章を書いておいた方がいいと思う。
0907名無しさん@英語勉強中 (ワッチョイ 7f89-gpJN)
垢版 |
2023/02/07(火) 13:22:38.73ID:PwvAUcYX0
>>905
やはり間違いではないでしょうか。
ナン・ネイティブは使わない方がいいと思う。

自分が定期的に覗いているサイトでフレーズ検索しても I am wrong that は見当たりません。

似た感じの言い回しを参考までに2つ挙げておきます。

・I hope that I am wrong about my sense that Biden will not defy
the national security consensus on such a sensitive issue by trying
to ...
これは前置詞の about を使い、その後で同格の接続詞の that で説明した例。

・It would be great if I am wrong about peace advocates being
listened to but nothing that has happened in the last 24 years
of nonstop ...
これも前置詞の about を使い、その後を動名詞で名詞語句のかたまりとした例。

つまり、いずれも I am wrong that と、いきなり接続詞の that を使って
内容を説明するという形ではありません。
0909The OED Loves Me Not (ワッチョイ 7f89-EOzK)
垢版 |
2023/02/08(水) 14:10:50.13ID:yWGIY94T0
>>908
僕はその方面に明るいわけではないけど、

pc computer temporary freeze

という keywords を放り込んで検索すると、次のような言葉が並んでいるのを
見かけた。

(1) Computer ★randomly freezes★? Learn the reasons and the solutions for the issue on Windows
It happens, that the computer randomly freezes for a few seconds.

(2) Why does my computer ●keep freezing●?
0910名無しさん@英語勉強中 (ワッチョイ 8f32-G/L/)
垢版 |
2023/02/09(木) 19:11:41.87ID:LH0XhegJ0
We will soon make a brief stop at ... .
新幹線なんかの社内アナウンスでよく耳にしますが、
We will soon be making a brief stop at ... . と進行形にすると違和感あるものでしょうか?
ニュアンスの違いを教えてくださればありがたいです。
0911The OED Loves Me Not (ワッチョイ 7f89-EOzK)
垢版 |
2023/02/09(木) 19:21:32.16ID:c2kRZa5O0
>>910
それはいわゆる frequently asked question というものだけど、
最近の辞書にはちゃんとそれについて詳しい解説がある。
たとえば「ジーニアス英和、第6版(2023)」の will の項目の、
「未来進行形(will be doing)の項目において、20行くらい
解説している。他の辞書にも載っているだろう。
0912名無しさん@英語勉強中 (ワッチョイ 8f33-zLlH)
垢版 |
2023/02/09(木) 19:38:14.62ID:S4TyCu1z0
宿題教えてください。できれば理由もお願いします。
There are too many people who get lung cancer(  )too much.
⑴ by smoking ⑵for smoking
0913名無しさん@英語勉強中 (ワッチョイ 7f89-gpJN)
垢版 |
2023/02/09(木) 20:06:55.58ID:0SPtBb7Z0
>>909
ありがとうございます。
ふと思い立って、ChatGPTに質問してみました。

角度を変えて質問を続けた結果、最終的には「brief not responding」を提案されました。
途中、他にも検索に使えそうな単語出てきたんで、これは便利ですね。
最初は "Petit Frère" とか言われてダメか、と思いましたがそんなことなかったです。
0915The OED Loves Me Not (ワッチョイ 7f89-EOzK)
垢版 |
2023/02/09(木) 21:31:21.20ID:c2kRZa5O0
>>914
工場で「製作中(製造中)の製品」のことを

products in process
とか
products in progress
って言うらしい。それを適用して、

files in process
files in progress
と言えば「作成中のファイル」っていう意味になるはず。そうすると、

to do (または todo)
done
に対応する言葉としては
in process
または
in progress
でいいんじゃないかという気がする。
0916名無しさん@英語勉強中 (ワッチョイ 0f96-F1up)
垢版 |
2023/02/09(木) 21:44:25.50ID:sn6/ZXHX0
>>912
その2つから選ぶなら、「by -ing」(~することによって)が正解。
「for -ing」は「~する目的で」。例 The knife is used for cutting bread. そのナイフはパンを切るために使われる。

ただし、実際の英語では、from smokingがベスト。fromは「(病気等の)原因」を表す。
0921名無しさん@英語勉強中 (ワッチョイ 8f33-zLlH)
垢版 |
2023/02/09(木) 22:29:29.79ID:S4TyCu1z0
>>916
ありがとうございます
0923名無しさん@英語勉強中 (ワッチョイ 3f69-zLlH)
垢版 |
2023/02/10(金) 20:29:50.87ID:bYAzTtf20
The other day, I posted a YouTube video in which I spoke in English.
これなんですが、 which の前に in がありますが、必要なんですかね?もしくは文法的にどう解釈すればこのinを正当化できるのでしょうか?
0927923 (ワッチョイ 3f69-zLlH)
垢版 |
2023/02/10(金) 23:04:28.97ID:bYAzTtf20
This is the apartment in which we used to live.
This is the apartment which we used to live in.
これならわかりますよ。
ただ
The other day, I posted a YouTube video in which I spoke in English.
これが分かりません
だったら
The other day, I posted a YouTube video which I spoke in English in.
出れでもいいという理屈になりますが、この文章ではinが必要ないと思うんですよね

それとも何か理由があるのでしょうか?
0928名無しさん@英語勉強中 (ワッチョイ 3f69-zLlH)
垢版 |
2023/02/10(金) 23:08:36.88ID:bYAzTtf20
I posted a YouTube video which I spoke in English.
これなら、しっくりくるんですが、無駄なinの意味が解らないんです
0931The OED Loves Me Not (ワッチョイ 1e89-8lmq)
垢版 |
2023/02/11(土) 04:08:39.75ID:MpO5UwNC0
>>928
(1) ★She talks about it in this video★ and you can purchase it below if you want to try it out.
https://dogtime.com/dog-health/flea-tick/49641-spot-flea-drop-treatments-killing-dog

上のようにニュースサイトに書いてある。この
She talks avbout it in this video.
の部分の this video を先行詞にしたら、
this video, ●in which● she talks about it
になる。

(2) He explains it in his book “The Optimistic Child“.
https://exploringyourmind.com/martin-seligman-positive-psychology/

上の文の his book "..." を先行詞にすると、

his book "The Optimistic Child," in which he explains it
になる。

なお、上の (1), (2) のそれぞれは最近の英語ではよく
in which の代わりに where になる。口語では特にそうだ。
0932The OED Loves Me Not (ワッチョイ 1e89-8lmq)
垢版 |
2023/02/11(土) 04:15:49.07ID:MpO5UwNC0
>>928
(*) I posted a YouTube video which I spoke in English.
もしこの英文が正しいとしたら、

(*) I spoke a YouTube video.
なんていう英語も正しいということになってしまう。

I speak English.
He speaks many languages.
これは正しい。でも
(*) I spoke a video.
これは間違い。

I spoke in a video.
これなら正しい。だから
a video in which I spoke
a video I spoke in
これも正しい。

そもそも、in a video とは「ビデオという場所にて」という意味だ。
それは、in New York とか in a bar と同じで、「そういう場所にて(そういう場所の中で)」
という意味だ。だから in とか at が常に必要。
0933名無しさん@英語勉強中 (ワッチョイW 0f96-w6cE)
垢版 |
2023/02/11(土) 09:11:16.16ID:PwG/grkH0
I spoke in a video. あ、ビデオ出て喋ったんね
I spoke English.  お、英語喋れるのか
I spoke in English. ふむ、英語で喋ったんね

I spoke a video. 意味不明。もしかしてビデオを喋ったのか、機械かよ
0934名無しさん@英語勉強中 (ワッチョイ 1e4b-lJcr)
垢版 |
2023/02/11(土) 09:54:36.48ID:e59vV0/T0
She threw her weight behind the candidate in hopes it will pay off if the candidate wins.
彼女は、候補者が勝ったときの見返りを期待して、職権を乱用して支援しました。

彼女は自分の力を利用した・振りかざしたのは確かなのですが、職権・地位を乱用したことを上の英文は含意しているのでしょうか。
それとも、前後関係から彼女がロクでもない女であることが前提な上での訳なのでしょうか。
0935名無しさん@英語勉強中 (ワッチョイ 0f96-t1ev)
垢版 |
2023/02/11(土) 13:15:54.92ID:PwG/grkH0
私も学びたい

throw weight は強権的なやり方で押し付けるみたいな意味があるようなので、
自分の力を利用した・振りかざしたのほうが正確なような気がします。

法や決まりを踏み外したかどうかはわかりませんが、
そこは訳者の好みの範囲で入れていいのではないでしょうか

というか、訳って正解とか不正解とかがないですよね。
一対一で日本語が定まるわけでもありませんし。
0936名無しさん@英語勉強中 (ワッチョイ 0af0-D0vN)
垢版 |
2023/02/11(土) 13:57:51.14ID:IUifIlOn0
権能・影響力を行使して・フル活用して
0937名無しさん@英語勉強中 (ワッチョイ fa69-AGaU)
垢版 |
2023/02/11(土) 14:12:23.16ID:JRWOiiEV0
>>930
何となく意味が解りました、ありがとうございます
0938名無しさん@英語勉強中 (ワッチョイ 1e89-vQqS)
垢版 |
2023/02/11(土) 14:21:49.39ID:lm6IRaqA0
>>934
>職権・地位を乱用したことを上の英文は含意しているのでしょうか

たぶん、言い過ぎになるかと思います。

例えば、オンライン辞書の Merriam-Webster をひくと、put/throw one's weight/influence behind の定義は、

to use one's influence to support (something)
Lobbyists are throwing their weight/influence behind the legislation.

と書いてあり、「自らの影響力を行使する」という説明だけです。

この言い回しは「職権・地位を乱用した」事件の文章に登場することが多い
でしょうが、必ずそういう事情が存在したとは限らないでしょう。
0939名無しさん@英語勉強中 (ワッチョイW 2b89-6rCu)
垢版 |
2023/02/11(土) 20:55:17.62ID:/tA90tFF0
エベレストに登れないのと同じように、彼にリーダーはできないでしょう。
He could no more be a leader than climb Mount Everest.

なぜ、He could be no more a leader
じゃないのかがわかりません。
なぜ動詞のbeがno moreの後ろへ行くんですか?
0941名無しさん@英語勉強中 (ワッチョイW 2b89-6rCu)
垢版 |
2023/02/11(土) 21:33:11.22ID:/tA90tFF0
文法的には間違ってるにしても、通常使われてる形は、助動詞が入る場合はno more の後ろにbeが来る、ってことでしょうか?
0944The OED Loves Me Not (ワッチョイ 1e89-8lmq)
垢版 |
2023/02/11(土) 21:56:58.14ID:MpO5UwNC0
>>939
He could

no more ★be a leader★
than ●climb Mount Everest●.

===

上のような構造になっている。つまり

no more 動詞 (1)
than 動詞 (2)

というわけだ。だからこの語順をそのまま踏襲するのが、
一番きれいなわけだ。

もし be no more a leader にしたら、

He could be
no more ★a leader★
than ●climb Mount Everest★.

となってしまって、きれいじゃないだろ?
0948名無しさん@英語勉強中 (ワッチョイ 0732-wyQN)
垢版 |
2023/02/12(日) 01:48:31.64ID:lzJkIogF0
>>910
を質問したものです 
新幹線の社内アナウンスで、We will soon be making a brief stop at ... . でなく
We will soon make a brief stop at ... としているのは、
定時刻発着をうたう新幹線運営側の矜持を表しているものと捉えてよいでしょうか?
0949The OED Loves Me Not (ワッチョイ 1e89-8lmq)
垢版 |
2023/02/12(日) 04:34:25.74ID:gOb/cMu00
>>948
いや、どうせ単なる日本人の書いた英文に過ぎなくて、外国人にしてはそこそこ上手な英語だ
という程度のことでしかないのだろう。1,000 人のうち 999 人の nonnatives の使う英語なんて、
本気で解釈しようなんて考えない方がいいと思う。nonnnatives の書く(しゃべる)英語なんて、
要は、通じればいいというレベルなんだ。その微妙な意味の違いなんて、意識して使えるほどの
レベルにはない。

もしごく普通の英語ネイティブが新幹線の正社員として、しかも注意深くこの英文を作っていたら、
We will soon be making... のように最初から書いていただろうよ。
0951名無しさん@英語勉強中 (ワッチョイ 0f96-t1ev)
垢版 |
2023/02/12(日) 10:11:18.46ID:2+fIZESa0
>>948
>>950

多分だけど
making a stop だと、一時停車するという動作に焦点が当たってしまうんじゃないかな
アナウンスが伝えたいのは、そこに「停まる予定」であるので、
「駅に近づいて人がいないことを確認し安全に停車し人の乗り降りを確認しまた安全を確認して出発する」
とかいう一連の動作を「生き生きと実行する予定」ではない。

make a stopは
 a stop なのでいくつかある中のちょっとした一つをしますよ、ってことで
have a break とかと一緒でしょう
stopでもいいけど完全停止じゃなくて一時停止ですよ、ということで。
終点近くでのアナウンスはおそらく違うはず

一方で、出発時には we will be stopping at A,B,and C というアナウンスがある。
この場合、停車を繰り返す予定は決まっているから be stopping が使われる


アナウンスしている人の動画。面白い。
https://youtu.be/XlX_8SdsW30
0957The OED Loves Me Not (ワッチョイ 1e89-8lmq)
垢版 |
2023/02/13(月) 10:47:39.69ID:MHIKcmsr0
別のサイトで出た質問だけど、久しぶりに見るような、
少しばかり tricky な問題を見た。

I am, actually. The essence of the African American experience is we’re hybrid people,
because the concept of race in America is not just genetic; it’s cultural.
●It●’s this notion of a people who look different than the mainstream, suffering discrimination
and for many decades terrible oppression, but somehow being able to make out of that a music
and a language and a faith and a patriotism and a belief in this project we call America ★that★ is unique.
https://okwave.jp/qa/q10102570.html

印をつけた It が何を指すのか、そして that の品詞は何で、それがこの文章の中で
何を指しているか(どういう機能を持っているのかを考えながら、印をつけた It からあとの
全文を和訳してみてほしい、というような趣旨の質問だ。

今のところ回答は2つついている。このスレッドの人たちは、どう考えるか?
考えるまでもなく、議論するまでもなく、実際には答えは一つだけなんだけど、
そんなに簡単ではないらしい。
0958名無しさん@英語勉強中 (ワッチョイW fa8a-CFQd)
垢版 |
2023/02/13(月) 12:39:51.11ID:FxoTmNQW0
>>957
強調構文ではないの?
最初からitとthatに印ついてるから余計にそう見えるがw
でも普通に読んでも強調構文だと思うだろうな
それで意味も通るし逆に他にあるか?という感じ
むしろmake out of that a music… のthatの方がよくわからん
0959The OED Loves Me Not (ワッチョイ 1e89-8lmq)
垢版 |
2023/02/13(月) 14:53:19.88ID:MHIKcmsr0
>>958
強調構文だと一発でわかってくれたのは、うれしいね。リンク先の2人目の回答者が、とんでもない回答をしてたので、びっくりした。

to make out of that a music hが、次のように続いていて

to make out of that a music and a language and a faith and a patriotism and a belief in this project we call America that is unique.

それを僕は、リンク先のサイトにて、次のように訳しておいた。

「その中からこのアメリカというプロジェクトの中で一種の(独自の)音楽と言語と宗教と愛国心と心情を創り上げる」

だから、make out of that a music だけを取り上げるなら、

「その中(つまり、他の主流の人々とは見かけが違っていて、差別に苦しみ、しかも何十年にもわたって恐ろしい抑圧に苦しんできたこと)
から、一種の(独自の)音楽を創り上げる」

という感じ。
0960名無しさん@英語勉強中 (ワッチョイW fa8a-CFQd)
垢版 |
2023/02/13(月) 16:53:41.79ID:FxoTmNQW0
>>959
回答者の一人OEDさん本人でワロタ
あーーー、やっとわかった、ありがとう!
実は959を読んでもそのthatの使い方がわからなくて、そのまま質問しようかとする直前にやっとわかった
いや俺make out of で「〜を生み出す」的な意味の熟語的なやつなんだろうなと思って読んでたから、このthatがなんでこんなところにポツンとあるのかわからなかったんですよね
なんなら誤植かと思ってた
out of thatね、こんな初歩的なことが今やっとわかってスッキリしました、ありがとうw
0962名無しさん@英語勉強中 (ワッチョイ 0af0-D0vN)
垢版 |
2023/02/13(月) 17:45:13.16ID:8jHL2f7W0
もう一人の回答の方が合ってるな
ただし唯一のアメリカでなくて
アメリカというユニークなプロジェクトだけど
0968名無しさん@英語勉強中 (ワッチョイ 835e-sQVB)
垢版 |
2023/02/13(月) 20:00:21.77ID:sBwyUJN40
英作文の質問です。問題集で「良い子を育てたかったら、周りの大人が良い言葉、心のこもった言葉を使わなくてはならない。」の答えが、If you want to raise a goog child, the adults surround them must use a good and thoughtful language.なのですが、なぜadultsにtheがついているのでしょう?
0971The OED Loves Me Not (ワッチョイ 1e89-8lmq)
垢版 |
2023/02/13(月) 20:19:42.57ID:MHIKcmsr0
a good child が不特定でも、その周りにいる人とか、
その子供が特定のときに一緒に遊んでいる
(あるいはしゃべっている)相手となると、それは特定されて
the がつく。

というか、そんなもん、普通に英文を読んでいたらそうなってること
がすぐにわかりそうなもんだが、英文をろくに読まずに、単に文法書だけを
読んでるんじゃないか、と思ってしまう。
0972968 (ワッチョイ 835e-sQVB)
垢版 |
2023/02/13(月) 20:33:35.77ID:sBwyUJN40
>>969-971

ありがとうございます。私が間違えました。the adults ★who★surround themとなってました。
どうも>>970さんのように思えたんですよね。

a goog child, adults who surround themだとバツですか?
0974968 (ワッチョイ 835e-sQVB)
垢版 |
2023/02/13(月) 20:53:00.69ID:sBwyUJN40
もうひとつお願いします。入試やばいです。

来週で午前中が空いているのは、火曜と金曜だけです。

正解はThe only morinigs I am free next week are Tuesday and Friday.ですが、なぜwill beじゃなくてareなんでしょう?未来のことならwillなのでは?
0976名無しさん@英語勉強中 (中止 8afb-AGaU)
垢版 |
2023/02/14(火) 09:30:25.93ID:CDS7hXlb0St.V
>>968 surroundの後はhim or herじゃなくてthemでもいいんだな(a childを受けて)
英語には男女兼用の人用3人称代名詞がないからか
0977The OED Loves Me Not (中止 1e89-8lmq)
垢版 |
2023/02/14(火) 09:43:19.85ID:KiVYhCcB0St.V
"singular they" (単数の they) が、women's liberation や、
それ以外のいろんな少数派の人たちの運動と共に、少なくとも50年前から喧々諤々と
論議されてきたのに、今ごろになってやっと気づく人もいたとはびっくりする。
0978The OED Loves Me Not (中止 1e89-8lmq)
垢版 |
2023/02/14(火) 09:47:39.49ID:KiVYhCcB0St.V
>>974
これについても、あっちこっちで論議されているけど、
もしどうしても文法書の中で確かめたいんなら、
安藤貞雄「現代英文法の講義」の中で
現在時制の使い方について説明してあるページを
1ページか2ページほど流し読みすればいい。
この程度のことは、高校生用の文法書にも載っているだろう。

もしそんなものを開く気がないのなら、差し当たって
「こういうときには、こうなるんだな」という程度に流しておけばいい。
たかが受験なんだから、適当にマークシートに印をつけてれば、
合格点くらいは取れる。別に満点じゃなくても、6割か7割ほど点数を
稼げば誰でも受かるんだから。
0981ブタ耳 (中止W 8ee3-zJF7)
垢版 |
2023/02/14(火) 11:41:23.34ID:NdnTtRAs0St.V
Imagine a car twenty meters away, coming at you at 160 km/h. That is the distance and speed ( ) a fastball can come from a pitcher to a batter.

括弧内に関係詞を補うとすれば何が適切だと思いますか?
前置詞付きも可。
先行詞は”the distance and speed”です。

悩んでいます。
0983名無しさん@英語勉強中 (中止 8ae2-FlRM)
垢版 |
2023/02/14(火) 12:53:09.99ID:uo5wCEk70St.V
>>981
at which
distanceは先行詞に含まれない
0984The OED Loves Me Not (中止 1e89-8lmq)
垢版 |
2023/02/14(火) 12:54:42.03ID:KiVYhCcB0St.V
>>981
(1) "the distance and speed with which a"

(2) "the distance and speed with which the"

上のような phrase を検索すると、それぞれいくつか
用例を出してくれる。そのうち、信頼性が高そうなものとしては、次のものがある。

(i) by DC Houston ・ 1972 ・ Cited by 36
... therefore presumably depends upon ★the distance and speed with which★ the birds are able to travel to the herds, ...

(ii) I. T. L. Education Solutions Limited, ?Itl ・ 2005
Direct control joysticks have the added ability to respond to ★the distance and speed with which★ the user moves the stick.

(続く)
0985The OED Loves Me Not (中止 1e89-8lmq)
垢版 |
2023/02/14(火) 12:55:19.22ID:KiVYhCcB0St.V
>>981 続き
(3) "the distance and speed at which a"

(4) "the distaancde and speed at which the"

上のような (3), (4) の用例も、一応は見つかる。そうかと思うと、

(5) "the distance and speed a" (つまり、関係代名詞も前置詞もなし)

(6) "the distancde and speed the" (同じく、関係代名詞も前置詞もなし)

というものも見つかる。ただし (5), (6) の場合は、最後の方に travels とか moves が来る。
こういう動詞の場合、the object travels the distance などというふうに、まるで
他動詞のようにして the distance を受けたりもするみたいだから、これについては除外した方がいいかな?

あくまで原案であえる
That is the distance and speed ( ) a fastball can come from a pitcher to a batter.
これにこだわるとすると、(1) から (4) のうちのいずれかがいいだろうな。
僕がその用例をちらほらと眺めた感触で言えば、(1), (2) の場合には
信頼性の高そうなサイトがいくらか見つかっているという感じがする。
だから僕ならば、with which にしておこうかな、と思う。
0986The OED Loves Me Not (中止 1e89-8lmq)
垢版 |
2023/02/14(火) 13:04:33.37ID:KiVYhCcB0St.V
>>983 の言うような理由で、確かに at which を使う用例の方が
勝っているようだな。とはいえ、with which が使われていないというわけでもなさそうだ。

In total, the researchers analyzed more than 21,000 wagging bouts, including ★the distance and speed with which★ their tails moved.
https://www.earth.com/news/dogs-wag-their-tails-to-the-right-for-people-they-love/
0987ブタ耳 (中止W 8ee3-zJF7)
垢版 |
2023/02/14(火) 13:16:42.96ID:NdnTtRAs0St.V
distance and speed with which,
distance and speed at which,
distance and speed where,
distance and speed that,
をNgranで検索すると、where以外は使用例が見つかりました。どれが多いかは一概に言えない結果です。
theを付けて検索してみたいところですが、あいにくNgramの語数制限に引っかかってしまいます。

theがdistanceの前にだけあるということは、”distance and speed”を一つのまとまりとして捉えているという解釈で良いですよね?
とすれば、at whichは不自然では?
0989名無しさん@英語勉強中 (中止 8ae2-FlRM)
垢版 |
2023/02/14(火) 13:23:32.72ID:uo5wCEk70St.V
A fast call can come with the distance from a pitcher to a batter.なんて文あるの?

travel the distanceとかならともかく。

そもそもマウンドからホームまでの距離は固定(18.44メートル)なんだからcanも何もないけど。
0991The OED Loves Me Not (中止 1e89-8lmq)
垢版 |
2023/02/14(火) 13:26:24.10ID:KiVYhCcB0St.V
>>987
atw which と with which との両方が一応は使われているように見えるから、
おそらく英語で物を書いている人たちは、
the distance and speed * which [subject] [verb]
と来た時に、最初の先行詞である distance に注目して with を使いたくなる人と、
"[前置詞} + which" の直前にある speed に引きずられて at にしたくなる人とが
混在しているということではないかという気がする。

たとえば Either of them is っていうのが正式でも、Either of them are っていう
人たちもいるのと同じような現象ではないかという気がする。
0993The OED Loves Me Not (中止 1e89-8lmq)
垢版 |
2023/02/14(火) 18:40:47.65ID:KiVYhCcB0St.V
2か月ほど前だと思うが、このスレにて
"as I write" という挿入句の意味を知りたい、
どの辞書や解説書に載っているか、という
質問があった。

それは本の著者がよく使うもので、「私がこれを書いている現在」
というような意味だ、と答えておいたが、その表現の意味や用法を解説書や
辞書の中で見つけるのは難しいだろうといつも思っていた。

第一、日本語でも「私がこれを書いている今では」とかいう
言い回しを著者は使うと思うが、そんなものを日本語の文法書や解説書や
辞書に載せているはずがないではないか。

そんなものは、普段からの読書の中で偶然に見つけていき、
その意味を考えながら読者は少しずつ覚えていくものだと思う。

それに似た言い回しを、久しぶりに見つけたが、それを
この下のリンク先で紹介しておいた。
https://lavender.5ch.net/test/read.cgi/english/1672871348/701
0995名無しさん@英語勉強中 (ワッチョイW fa8a-CFQd)
垢版 |
2023/02/14(火) 22:35:38.54ID:G9ZHX3In0
>>994
これ何て問題集?色々勉強だよこれ
0996名無しさん@英語勉強中 (ワッチョイW fa8a-CFQd)
垢版 |
2023/02/14(火) 22:36:16.57ID:G9ZHX3In0
打ち間違い
勉強だよ→変だよ
10011001
垢版 |
Over 1000Thread
このスレッドは1000を超えました。
新しいスレッドを立ててください。
life time: 46日 20時間 4分 39秒
10021002
垢版 |
Over 1000Thread
5ちゃんねるの運営はプレミアム会員の皆さまに支えられています。
運営にご協力お願いいたします。


───────────────────
《プレミアム会員の主な特典》
★ 5ちゃんねる専用ブラウザからの広告除去
★ 5ちゃんねるの過去ログを取得
★ 書き込み規制の緩和
───────────────────

会員登録には個人情報は一切必要ありません。
月300円から匿名でご購入いただけます。

▼ プレミアム会員登録はこちら ▼
https://premium.5ch.net/

▼ 浪人ログインはこちら ▼
https://login.5ch.net/login.php
レス数が1000を超えています。これ以上書き込みはできません。

ニューススポーツなんでも実況